User login
Caring for women with pelvic floor disorders during pregnancy and postpartum: Expert guidance
Pelvic floor disorders (PFDs) affect many pregnant and newly postpartum women. These conditions, including urinary incontinence, anal incontinence, and pelvic organ prolapse (POP), can be overshadowed by common pregnancy and postpartum concerns (TABLE 1).1 With the use of a few quick screening questions, however, PFDs easily can be identified in this at-risk population. Active management need not be delayed until after delivery for women experiencing bother, as options exist for women with PFDs during pregnancy as well as postpartum.
In this article, we discuss the common PFDs that obstetric clinicians face in the context of case scenarios and review how you can be better equipped to care for affected individuals.
CASE 1 Screening
A 30-year-old woman (G1P1) presents for her routine postpartum visit after an operative vaginal delivery with a second-degree laceration.
How would you screen this patient for PFDs?
Why screening for PFDs matters
While there are no validated PFD screening tools for this patient population, clinicians can ask a series of brief open-ended questions as part of the review of systems to efficiently evaluate for the common PFDs in peripartum patients (see “Screening questions to evaluate patients for peripartum pelvic floor disorders” below).
Pelvic floor disorders in the peripartum period can have a significant negative impact. In pregnancy, nearly half of women report psychological strain due to the presence of bowel, bladder, prolapse, or sexual dysfunction symptoms.2 Postpartum, PFDs have negative effects on overall health, well-being, and self-esteem, with significantly increased rates of postpartum depression in women who experience urinary incontinence.3,4 Proactively inquiring about PFD symptoms, providing anticipatory guidance, and recommending treatment options can positively impact a patient in multiple domains.
Sometimes during pregnancy or after having a baby, a woman experiences pelvic floor symptoms. Do you have any of the following?
- leakage with coughing, laughing, sneezing, or physical activity
- urgency to urinate or leakage due to urgency
- bulging or pressure within the vagina
- pain with intercourse
- accidental bowel leakage of stool or flatus
CASE 2 Stress urinary incontinence
A 27-year-old woman (G1P1) presents 2 months following spontaneous vaginal delivery with symptoms of urine leakage with laughing and running. Her urinary incontinence has been improving since delivery, but it continues to be bothersome.
What would you recommend for this patient?
Conservative SUI management strategies in pregnancy
Urinary tract symptoms are common in pregnancy, with up to 41.8% of women reporting urinary symptom distress in the third trimester.5 During pregnancy, estrogen and progesterone decrease urethral pressure that, together with increased intra-abdominal pressure from the gravid uterus, can cause or worsen stress urinary incontinence (SUI).6
During pregnancy, women should be offered conservative therapies for SUI. For women who can perform a pelvic floor contraction (a Kegel exercise), self-guided pelvic floor muscle exercises (PFMEs) may be helpful (see “Pelvic floor muscle exercises” below). We recommend that women start with 1 to 2 sets of 10 Kegel exercises per day and that they hold the squeeze for 2 to 3 seconds, working up to holding for 10 seconds. The goal is to strengthen and improve muscle control so that the Kegel squeeze can be paired with activities that cause SUI.
For women who are unable to perform a Kegel exercise or are not improving with a home PFME regimen, referral to pelvic floor physical therapy (PFPT) can be considered. While data support the efficacy of PFPT for SUI treatment in nonpregnant women,7 data are lacking on PFME in pregnancy.
In women without urinary incontinence, PFME in early pregnancy can prevent the onset of incontinence in late pregnancy and the postpartum period.8 By contrast, the same 2020 Cochrane Review found no evidence that antenatal pelvic floor muscle therapy in incontinent women decreases incontinence in mid- or late-pregnancy or in the postpartum period.8 As the quality of this evidence is very low and there is no evidence of harm with PFME, we continue to recommend it for women with bothersome SUI.
Incontinence pessaries or vaginal inserts (such as Poise Impressa bladder supports) can be helpful for SUI treatment. An incontinence pessary can be fitted in the office, and fitting kits are available for both. Pessaries can safely be used in pregnancy, but there are no data on the efficacy of pessaries for treating SUI in pregnancy. In nonpregnant women, evidence demonstrates 63% satisfaction 3 months post–pessary placement for SUI.7
We do not recommend invasive procedures for the treatment of SUI during pregnancy or in the first 6 months following delivery. There is no evidence that elective cesarean delivery prevents persistent SUI postpartum.9
To identify and engage the proper pelvic floor muscles:
- Insert a finger in the vagina and squeeze the vaginal muscles around your finger.
- Imagine you are sitting on a marble and have to pick it up with the vaginal muscles.
- Squeeze the muscles you would use to stop the flow of urine or hold back flatulence.
Perform sets of 10, 2 to 3 times per day as follows:
- Squeeze: Engage the pelvic floor muscles as described above; avoid performing Kegels while voiding.
- Hold: For 2 to 10 seconds; increase the duration to 10 seconds as able.
- Relax: Completely relax muscles before initating the next squeeze.
Reference
1. UpToDate. Patient education: pelvic muscle (Kegel) exercises (the basics). 2018. https://uptodatefree.ir/topic.htm?path=pelvic-muscle-kegel-exercises-the-basics. Accessed February 24, 2021.
Continue to: Managing SUI in the postpartum period...
Managing SUI in the postpartum period
After the first 6 months postpartum and exhaustion of conservative measures, we offer surgical interventions for women with persistent, bothersome incontinence. Surgery for SUI typically is not recommended until childbearing is complete, but it can be considered if the patient’s bother is significant.
For women with bothersome SUI who still desire future pregnancy, management options include periurethral bulking, a retropubic urethropexy (Burch procedure), or a midurethral sling procedure. Women who undergo an anti-incontinence procedure have an increased risk for urinary retention during a subsequent pregnancy.10 Most women with a midurethral sling will continue to be continent following an obstetric delivery.
Anticipatory guidance
At 3 months postpartum, the incidence of urinary incontinence is 6% to 40%, depending on parity and delivery type. Postpartum urinary incontinence is most common after instrumented vaginal delivery (32%) followed by spontaneous vaginal delivery (28%) and cesarean delivery (15%). The mean prevalence of any type of urinary incontinence is 33% at 3 months postpartum, and only small changes in the rate of urinary incontinence occur over the first postpartum year.11 While urinary incontinence is common postpartum, it should not be considered normal. We counsel that symptoms may improve spontaneously, but treatment can be initiated if the patient experiences significant bother.
A longitudinal cohort study that followed women from 3 months to 12 years postpartum found that, of women with urinary incontinence at 3 months postpartum, 76% continued to report incontinence at 12 years postpartum.12 We recommend that women be counseled that, even when symptoms resolve, they remain at increased risk for urinary incontinence in the future. Invasive therapies should be used to treat bothersome urinary incontinence, not to prevent future incontinence.
CASE 3 Fecal incontinence
A 24-year-old woman (G1P1) presents 3 weeks postpartum following a forceps-assisted vaginal delivery complicated by a 3c laceration. She reports fecal urgency, inability to control flatus, and once-daily fecal incontinence.
How would you evaluate these symptoms?
Steps in evaluation
The initial evaluation should include an inquiry regarding the patient’s stool consistency and bowel regimen. The Bristol stool form scale can be used to help patients describe their typical bowel movements (TABLE 2).13 During healing, the goal is to achieve a Bristol type 4 stool, both to avoid straining and to improve continence, as loose stool is the most difficult to control.
A physical examination can evaluate healing and sphincter integrity; it should include inspection of the distal vagina and perineal body and a digital rectal exam. Anal canal resting tone and squeeze strength should be evaluated, and the digital rectal examination scoring system (DRESS) can be useful for quantification (TABLE 3).14 Lack of tone at rest in the anterolateral portion of the sphincter complex can indicate an internal anal sphincter defect, as 80% of the resting tone comes from this muscle (FIGURE).15
The rectovaginal septum should be assessed given the increased risk of rectovaginal fistula in women with obstetric anal sphincter injury (OASI). The patient should be instructed to contract the anal sphincter, allowing evaluation of muscular contraction. Lack of contraction anteriolaterally may indicate external anal sphincter separation.
Continue to: Conservative options for improving fecal incontinence symptoms...
Conservative options for improving fecal incontinence symptoms
The patient can be counseled regarding stool bulking, first with insoluble fiber supplementation and cessation of stool softeners if she is incontinent of liquid stool. If these measures are not effective, use of a constipating agent, such as loperamide, can improve stool consistency and thereby decrease incontinence episodes. PFPT with biofeedback can be offered as well. While typically we do not recommend initiating PFPT before 6 weeks postpartum, so the initial phases of healing can occur, early referral enables the patient to avoid a delay in access to care.
The patient also can be counseled about a referral to a pelvic floor specialist for further evaluation. A variety of peripartum pelvic floor disorder clinics are being established by Female Pelvic Medicine and Reconstructive Surgery (FPMRS) physicians. These clinics provide the benefit of comprehensive care for pelvic floor disorders in this unique population.
When conservative measures fail. If a patient has persistent bowel control issues despite conservative measures, a referral to an FPMRS physician should be initiated.
Delivery route in future pregnancies
The risk of a subsequent OASI is low. While this means that many women can safely pursue a future vaginal delivery, a scheduled cesarean delivery is indicated for women with persistent bowel control issues, wound healing complications, and those who experienced psychological trauma from their delivery.16 We recommend a shared-decision making approach, reviewing modifiable and nonmodifiable risk factors to help determine whether or not a future vaginal birth is appropriate. It is important to highlight that a cesarean delivery does not protect against fecal incontinence in women with a history of OASI; however, there is benefit in preventing worsening of anal incontinence, if present.17
CASE 4 Uterovaginal prolapse
A 36-year-old woman (G3P3) presents for her routine postpartum visit at 6 weeks after a spontaneous vaginal delivery without lacerations. She reports a persistent feeling of vaginal pressure and fullness. She thinks she felt a bulge with wiping after a bowel movement.
What options are available for this patient?
Prolapse in the peripartum population
Previous studies have revealed an increased prevalence of POP in pregnant women on examination compared with their nulligravid counterparts (47.6% vs 0%).18 With the changes in the hormonal milieu in pregnancy, as well as the weight of the gravid uterus on the pelvic floor, it is not surprising that pregnancy may be the inciting event to expose even transient defects in pelvic organ support.19
It is well established that increasing parity and, to a lesser extent, larger babies are associated with increased risk for future POP and surgery for prolapse. In the first year postpartum, nearly one-third of women have stage 2 or greater prolapse on exam, with studies demonstrating an increased prevalence of postpartum POP in women who delivered vaginally compared with those who delivered by cesarean.20,21
Initial evaluation
Diagnosis can be made during a routine pelvic exam by having the patient perform a Valsalva maneuver while in the lithotomy position. Using half of a speculum permits evaluation of the anterior and posterior vaginal walls separately, and Valsalva during a bimanual exam can aid in evaluating descensus of the uterus and cervix.
Excellent free patient education resources available online through the American Urogynecologic Society and the International Urogynecological Association can be used to direct counseling.
Continue to: Treatments you can offer for POP...
Treatments you can offer for POP
For pregnant or postpartum patients with bothersome prolapse, initial management options include pessary fitting and/or PFPT referral. In pregnancy, women often can be successfully fitted with a pessary for POP; however, as expulsion is a common issue, selection of a stiffer or space-occupying device may be more efficacious.
Often, early onset POP in pregnancy resolves as the gravid uterus lifts out of the pelvis in the second trimester, at which time the pessary can be discontinued. In the postpartum period, a pessary fitting can be undertaken similarly to that in nonpregnant patients. While data are lacking in the peripartum population, evidence supports the positive impact of PFPT on improving POP symptom bother.22 Additionally, for postpartum women who experience OASI, PFPT can produce significant improvement in subjective POP and associated bother.23
Impact of future childbearing wishes on treatment
The desire for future childbearing does not preclude treatment of patients experiencing bother from POP after conservative management options have failed. Both vaginal native tissue and mesh-augmented uterine-sparing repairs are performed by many FPMRS specialists and are associated with good outcomes. As with SUI, we do not recommend invasive treatment for POP during pregnancy or before 6 months postpartum.
In conclusion
Obstetric specialists play an essential role in caring for women with PFDs in the peripartum period. Basic evaluation, counseling, and management can be initiated using many of the resources already available in an obstetric ambulatory practice. Important adjunctive resources include those available for both providers and patients through the American Urogynecologic Society and the International Urogynecological Association. In addition, clinicians can partner with pelvic floor specialists through the growing number of FPMRS-run peripartum pelvic floor disorder clinics across the country and pelvic floor physical therapists.
If these specialty clinics and therapists are not available in your area, FPMRS specialists, urologists, gastroenterologists, and/or colorectal surgeons can aid in patient diagnosis and management to reach the ultimate goal of improving PFDs in this at-risk population. ●
- Madsen AM, Hickman LC, Propst K. Recognition and management of pelvic floor disorders in pregnancy and the postpartum period. Obstet Gynecol Clin North Am. Forthcoming 2021.
- Bodner-Adler B, Kimberger O, Laml T, et al. Prevalence and risk factors for pelvic floor disorders during early and late pregnancy in a cohort of Austrian women. Arch Gynecol Obstet. 2019;300:1325-1330.
- Swenson CW, DePorre JA, Haefner JK, et al. Postpartum depression screening and pelvic floor symptoms among women referred to a specialty postpartum perineal clinic. Am J Obstet Gynecol. 2018;218:335.e1-335.e6.
- Skinner EM, Dietz HP. Psychological and somatic sequelae of traumatic vaginal delivery: a literature review. Aust N Z J Obstet Gynaecol. 2015;55:309-314.
- Yohay D, Weintraub AY, Mauer-Perry N, et al. Prevalence and trends of pelvic floor disorders in late pregnancy and after delivery in a cohort of Israeli women using the PFDI-20. Eur J Obstet Gynecol Reprod Biol. 2016;200:35-39.
- Gregory WT, Sibai BM. Obstetrics and pelvic floor disorders. In: Walters M, Karram M, eds. Urogynecology and Reconstructive Pelvic Surgery. 4th ed. Philadelphia, PA: Saunders; 2015:224-237.
- Richter HE, Burgio KL, Brubaker L, et al; Pelvic Floor Disorders Network. Continence pessary compared with behavioral therapy or combined therapy for stress incontinence: a randomized controlled trial. Obstet Gynecol. 2010;115:609-617.
- Woodley SJ, Lawrenson P, Boyle R, et al. Pelvic floor muscle training for preventing and treating urinary and faecal incontinence in antenatal and postnatal women. Cochrane Database Syst Rev. 2020;6:CD007471.
- Foldspang A, Hvidman L, Mommsen S, et al. Risk of postpartum urinary incontinence associated with pregnancy and mode of delivery. Acta Obstet Gynecol Scand. 2004;83:923-927.
- Wieslander CK, Weinstein MM, Handa V, et al. Pregnancy in women with prior treatments for pelvic floor disorders. Female Pelvic Med Reconstr Surg. 2020;26:299-305.
- Thom DH, Rortveit G. Prevalence of postpartum urinary incontinence: a systematic review. Acta Obstet Gynecol Scand. 2010;89:1511-1522.
- MacArthur C, Wilson D, Herbison P, et al; Prolong Study Group. Urinary incontinence persisting after childbirth: extent, delivery history, and effects in a 12-year longitudinal cohort study. BJOG. 2016;123:1022-1029.
- Blake MR, Raker JM, Whelan K. Validity and reliability of the Bristol Stool Form Scale in healthy adults and patients with diarrhoea-predominant irritable bowel syndrome. Aliment Pharmacol Ther. 2016;44:693-703
- Orkin BA, Sinykin SB, Lloyd PC. The digital rectal examination scoring system (DRESS). Dis Colon Rectum. 2010;53:1656-1660.
- UpToDate. Repair of episiotomy and perineal lacerations associated with childbirth. 2020. https://www-uptodate-com .ccmain.ohionet.org/contents/repair-of-perineal-and-other -lacerations-associated-with-childbirth?search=repair%20 episiotomy&source=search_result&selectedTitle=1~150&usa ge_type=default&display_rank=1. Accessed February 28, 2021.
- Committee on Practice Bulletins–Obstetrics. ACOG practice bulletin no. 198: prevention and management of obstetric lacerations at vaginal delivery. Obstet Gynecol. 2018;132:e87-e102.
- Jangö H, Langhoff-Roos J, Rosthøj S, et al. Long-term anal incontinence after obstetric anal sphincter injury—does grade of tear matter? Am J Obstet Gynecol. 2018;218:232.e1-232.e10.
- O’Boyle AL, Woodman PJ, O’Boyle JD, et al. Pelvic organ support in nulliparous pregnant and nonpregnant women: a case control study. Am J Obstet Gynecol. 2002;187:99-102.
- Handa VL, Blomquist JL, McDermott KC, et al. Pelvic floor disorders after vaginal birth. Obstet Gynecol. 2012;119 (2, pt 1):233-239.
- Handa VL, Nygaard I, Kenton K, et al; Pelvic Floor Disorders Network. Pelvic organ support among primiparous women in the first year after childbirth. Int Urogynecol J Pelvic Floor Dysfunct. 2009;20:1407-1411.
- O’Boyle AL, O’Boyle JD, Calhoun B, et al. Pelvic organ support in pregnancy and postpartum. Int Urogynecol J Pelvic Floor Dysfunct. 2005;16:69-72.
- Hagen S, Stark D, Glazener C, et al; POPPY Trial Collaborators. Individualised pelvic floor muscle training in women with pelvic organ prolapse (POPPY): a multicentre randomised controlled trial. Lancet. 2014;383:796-806.
- Von Bargen E, Haviland MJ, Chang OH, et al. Evaluation of postpartum pelvic floor physical therapy on obstetrical anal sphincter injury: a randomized controlled trial. Female Pelvic Med Reconstr Surg. 2020. doi: 10.1097/SPV.0000000000000849.
Pelvic floor disorders (PFDs) affect many pregnant and newly postpartum women. These conditions, including urinary incontinence, anal incontinence, and pelvic organ prolapse (POP), can be overshadowed by common pregnancy and postpartum concerns (TABLE 1).1 With the use of a few quick screening questions, however, PFDs easily can be identified in this at-risk population. Active management need not be delayed until after delivery for women experiencing bother, as options exist for women with PFDs during pregnancy as well as postpartum.
In this article, we discuss the common PFDs that obstetric clinicians face in the context of case scenarios and review how you can be better equipped to care for affected individuals.
CASE 1 Screening
A 30-year-old woman (G1P1) presents for her routine postpartum visit after an operative vaginal delivery with a second-degree laceration.
How would you screen this patient for PFDs?
Why screening for PFDs matters
While there are no validated PFD screening tools for this patient population, clinicians can ask a series of brief open-ended questions as part of the review of systems to efficiently evaluate for the common PFDs in peripartum patients (see “Screening questions to evaluate patients for peripartum pelvic floor disorders” below).
Pelvic floor disorders in the peripartum period can have a significant negative impact. In pregnancy, nearly half of women report psychological strain due to the presence of bowel, bladder, prolapse, or sexual dysfunction symptoms.2 Postpartum, PFDs have negative effects on overall health, well-being, and self-esteem, with significantly increased rates of postpartum depression in women who experience urinary incontinence.3,4 Proactively inquiring about PFD symptoms, providing anticipatory guidance, and recommending treatment options can positively impact a patient in multiple domains.
Sometimes during pregnancy or after having a baby, a woman experiences pelvic floor symptoms. Do you have any of the following?
- leakage with coughing, laughing, sneezing, or physical activity
- urgency to urinate or leakage due to urgency
- bulging or pressure within the vagina
- pain with intercourse
- accidental bowel leakage of stool or flatus
CASE 2 Stress urinary incontinence
A 27-year-old woman (G1P1) presents 2 months following spontaneous vaginal delivery with symptoms of urine leakage with laughing and running. Her urinary incontinence has been improving since delivery, but it continues to be bothersome.
What would you recommend for this patient?
Conservative SUI management strategies in pregnancy
Urinary tract symptoms are common in pregnancy, with up to 41.8% of women reporting urinary symptom distress in the third trimester.5 During pregnancy, estrogen and progesterone decrease urethral pressure that, together with increased intra-abdominal pressure from the gravid uterus, can cause or worsen stress urinary incontinence (SUI).6
During pregnancy, women should be offered conservative therapies for SUI. For women who can perform a pelvic floor contraction (a Kegel exercise), self-guided pelvic floor muscle exercises (PFMEs) may be helpful (see “Pelvic floor muscle exercises” below). We recommend that women start with 1 to 2 sets of 10 Kegel exercises per day and that they hold the squeeze for 2 to 3 seconds, working up to holding for 10 seconds. The goal is to strengthen and improve muscle control so that the Kegel squeeze can be paired with activities that cause SUI.
For women who are unable to perform a Kegel exercise or are not improving with a home PFME regimen, referral to pelvic floor physical therapy (PFPT) can be considered. While data support the efficacy of PFPT for SUI treatment in nonpregnant women,7 data are lacking on PFME in pregnancy.
In women without urinary incontinence, PFME in early pregnancy can prevent the onset of incontinence in late pregnancy and the postpartum period.8 By contrast, the same 2020 Cochrane Review found no evidence that antenatal pelvic floor muscle therapy in incontinent women decreases incontinence in mid- or late-pregnancy or in the postpartum period.8 As the quality of this evidence is very low and there is no evidence of harm with PFME, we continue to recommend it for women with bothersome SUI.
Incontinence pessaries or vaginal inserts (such as Poise Impressa bladder supports) can be helpful for SUI treatment. An incontinence pessary can be fitted in the office, and fitting kits are available for both. Pessaries can safely be used in pregnancy, but there are no data on the efficacy of pessaries for treating SUI in pregnancy. In nonpregnant women, evidence demonstrates 63% satisfaction 3 months post–pessary placement for SUI.7
We do not recommend invasive procedures for the treatment of SUI during pregnancy or in the first 6 months following delivery. There is no evidence that elective cesarean delivery prevents persistent SUI postpartum.9
To identify and engage the proper pelvic floor muscles:
- Insert a finger in the vagina and squeeze the vaginal muscles around your finger.
- Imagine you are sitting on a marble and have to pick it up with the vaginal muscles.
- Squeeze the muscles you would use to stop the flow of urine or hold back flatulence.
Perform sets of 10, 2 to 3 times per day as follows:
- Squeeze: Engage the pelvic floor muscles as described above; avoid performing Kegels while voiding.
- Hold: For 2 to 10 seconds; increase the duration to 10 seconds as able.
- Relax: Completely relax muscles before initating the next squeeze.
Reference
1. UpToDate. Patient education: pelvic muscle (Kegel) exercises (the basics). 2018. https://uptodatefree.ir/topic.htm?path=pelvic-muscle-kegel-exercises-the-basics. Accessed February 24, 2021.
Continue to: Managing SUI in the postpartum period...
Managing SUI in the postpartum period
After the first 6 months postpartum and exhaustion of conservative measures, we offer surgical interventions for women with persistent, bothersome incontinence. Surgery for SUI typically is not recommended until childbearing is complete, but it can be considered if the patient’s bother is significant.
For women with bothersome SUI who still desire future pregnancy, management options include periurethral bulking, a retropubic urethropexy (Burch procedure), or a midurethral sling procedure. Women who undergo an anti-incontinence procedure have an increased risk for urinary retention during a subsequent pregnancy.10 Most women with a midurethral sling will continue to be continent following an obstetric delivery.
Anticipatory guidance
At 3 months postpartum, the incidence of urinary incontinence is 6% to 40%, depending on parity and delivery type. Postpartum urinary incontinence is most common after instrumented vaginal delivery (32%) followed by spontaneous vaginal delivery (28%) and cesarean delivery (15%). The mean prevalence of any type of urinary incontinence is 33% at 3 months postpartum, and only small changes in the rate of urinary incontinence occur over the first postpartum year.11 While urinary incontinence is common postpartum, it should not be considered normal. We counsel that symptoms may improve spontaneously, but treatment can be initiated if the patient experiences significant bother.
A longitudinal cohort study that followed women from 3 months to 12 years postpartum found that, of women with urinary incontinence at 3 months postpartum, 76% continued to report incontinence at 12 years postpartum.12 We recommend that women be counseled that, even when symptoms resolve, they remain at increased risk for urinary incontinence in the future. Invasive therapies should be used to treat bothersome urinary incontinence, not to prevent future incontinence.
CASE 3 Fecal incontinence
A 24-year-old woman (G1P1) presents 3 weeks postpartum following a forceps-assisted vaginal delivery complicated by a 3c laceration. She reports fecal urgency, inability to control flatus, and once-daily fecal incontinence.
How would you evaluate these symptoms?
Steps in evaluation
The initial evaluation should include an inquiry regarding the patient’s stool consistency and bowel regimen. The Bristol stool form scale can be used to help patients describe their typical bowel movements (TABLE 2).13 During healing, the goal is to achieve a Bristol type 4 stool, both to avoid straining and to improve continence, as loose stool is the most difficult to control.
A physical examination can evaluate healing and sphincter integrity; it should include inspection of the distal vagina and perineal body and a digital rectal exam. Anal canal resting tone and squeeze strength should be evaluated, and the digital rectal examination scoring system (DRESS) can be useful for quantification (TABLE 3).14 Lack of tone at rest in the anterolateral portion of the sphincter complex can indicate an internal anal sphincter defect, as 80% of the resting tone comes from this muscle (FIGURE).15
The rectovaginal septum should be assessed given the increased risk of rectovaginal fistula in women with obstetric anal sphincter injury (OASI). The patient should be instructed to contract the anal sphincter, allowing evaluation of muscular contraction. Lack of contraction anteriolaterally may indicate external anal sphincter separation.
Continue to: Conservative options for improving fecal incontinence symptoms...
Conservative options for improving fecal incontinence symptoms
The patient can be counseled regarding stool bulking, first with insoluble fiber supplementation and cessation of stool softeners if she is incontinent of liquid stool. If these measures are not effective, use of a constipating agent, such as loperamide, can improve stool consistency and thereby decrease incontinence episodes. PFPT with biofeedback can be offered as well. While typically we do not recommend initiating PFPT before 6 weeks postpartum, so the initial phases of healing can occur, early referral enables the patient to avoid a delay in access to care.
The patient also can be counseled about a referral to a pelvic floor specialist for further evaluation. A variety of peripartum pelvic floor disorder clinics are being established by Female Pelvic Medicine and Reconstructive Surgery (FPMRS) physicians. These clinics provide the benefit of comprehensive care for pelvic floor disorders in this unique population.
When conservative measures fail. If a patient has persistent bowel control issues despite conservative measures, a referral to an FPMRS physician should be initiated.
Delivery route in future pregnancies
The risk of a subsequent OASI is low. While this means that many women can safely pursue a future vaginal delivery, a scheduled cesarean delivery is indicated for women with persistent bowel control issues, wound healing complications, and those who experienced psychological trauma from their delivery.16 We recommend a shared-decision making approach, reviewing modifiable and nonmodifiable risk factors to help determine whether or not a future vaginal birth is appropriate. It is important to highlight that a cesarean delivery does not protect against fecal incontinence in women with a history of OASI; however, there is benefit in preventing worsening of anal incontinence, if present.17
CASE 4 Uterovaginal prolapse
A 36-year-old woman (G3P3) presents for her routine postpartum visit at 6 weeks after a spontaneous vaginal delivery without lacerations. She reports a persistent feeling of vaginal pressure and fullness. She thinks she felt a bulge with wiping after a bowel movement.
What options are available for this patient?
Prolapse in the peripartum population
Previous studies have revealed an increased prevalence of POP in pregnant women on examination compared with their nulligravid counterparts (47.6% vs 0%).18 With the changes in the hormonal milieu in pregnancy, as well as the weight of the gravid uterus on the pelvic floor, it is not surprising that pregnancy may be the inciting event to expose even transient defects in pelvic organ support.19
It is well established that increasing parity and, to a lesser extent, larger babies are associated with increased risk for future POP and surgery for prolapse. In the first year postpartum, nearly one-third of women have stage 2 or greater prolapse on exam, with studies demonstrating an increased prevalence of postpartum POP in women who delivered vaginally compared with those who delivered by cesarean.20,21
Initial evaluation
Diagnosis can be made during a routine pelvic exam by having the patient perform a Valsalva maneuver while in the lithotomy position. Using half of a speculum permits evaluation of the anterior and posterior vaginal walls separately, and Valsalva during a bimanual exam can aid in evaluating descensus of the uterus and cervix.
Excellent free patient education resources available online through the American Urogynecologic Society and the International Urogynecological Association can be used to direct counseling.
Continue to: Treatments you can offer for POP...
Treatments you can offer for POP
For pregnant or postpartum patients with bothersome prolapse, initial management options include pessary fitting and/or PFPT referral. In pregnancy, women often can be successfully fitted with a pessary for POP; however, as expulsion is a common issue, selection of a stiffer or space-occupying device may be more efficacious.
Often, early onset POP in pregnancy resolves as the gravid uterus lifts out of the pelvis in the second trimester, at which time the pessary can be discontinued. In the postpartum period, a pessary fitting can be undertaken similarly to that in nonpregnant patients. While data are lacking in the peripartum population, evidence supports the positive impact of PFPT on improving POP symptom bother.22 Additionally, for postpartum women who experience OASI, PFPT can produce significant improvement in subjective POP and associated bother.23
Impact of future childbearing wishes on treatment
The desire for future childbearing does not preclude treatment of patients experiencing bother from POP after conservative management options have failed. Both vaginal native tissue and mesh-augmented uterine-sparing repairs are performed by many FPMRS specialists and are associated with good outcomes. As with SUI, we do not recommend invasive treatment for POP during pregnancy or before 6 months postpartum.
In conclusion
Obstetric specialists play an essential role in caring for women with PFDs in the peripartum period. Basic evaluation, counseling, and management can be initiated using many of the resources already available in an obstetric ambulatory practice. Important adjunctive resources include those available for both providers and patients through the American Urogynecologic Society and the International Urogynecological Association. In addition, clinicians can partner with pelvic floor specialists through the growing number of FPMRS-run peripartum pelvic floor disorder clinics across the country and pelvic floor physical therapists.
If these specialty clinics and therapists are not available in your area, FPMRS specialists, urologists, gastroenterologists, and/or colorectal surgeons can aid in patient diagnosis and management to reach the ultimate goal of improving PFDs in this at-risk population. ●
Pelvic floor disorders (PFDs) affect many pregnant and newly postpartum women. These conditions, including urinary incontinence, anal incontinence, and pelvic organ prolapse (POP), can be overshadowed by common pregnancy and postpartum concerns (TABLE 1).1 With the use of a few quick screening questions, however, PFDs easily can be identified in this at-risk population. Active management need not be delayed until after delivery for women experiencing bother, as options exist for women with PFDs during pregnancy as well as postpartum.
In this article, we discuss the common PFDs that obstetric clinicians face in the context of case scenarios and review how you can be better equipped to care for affected individuals.
CASE 1 Screening
A 30-year-old woman (G1P1) presents for her routine postpartum visit after an operative vaginal delivery with a second-degree laceration.
How would you screen this patient for PFDs?
Why screening for PFDs matters
While there are no validated PFD screening tools for this patient population, clinicians can ask a series of brief open-ended questions as part of the review of systems to efficiently evaluate for the common PFDs in peripartum patients (see “Screening questions to evaluate patients for peripartum pelvic floor disorders” below).
Pelvic floor disorders in the peripartum period can have a significant negative impact. In pregnancy, nearly half of women report psychological strain due to the presence of bowel, bladder, prolapse, or sexual dysfunction symptoms.2 Postpartum, PFDs have negative effects on overall health, well-being, and self-esteem, with significantly increased rates of postpartum depression in women who experience urinary incontinence.3,4 Proactively inquiring about PFD symptoms, providing anticipatory guidance, and recommending treatment options can positively impact a patient in multiple domains.
Sometimes during pregnancy or after having a baby, a woman experiences pelvic floor symptoms. Do you have any of the following?
- leakage with coughing, laughing, sneezing, or physical activity
- urgency to urinate or leakage due to urgency
- bulging or pressure within the vagina
- pain with intercourse
- accidental bowel leakage of stool or flatus
CASE 2 Stress urinary incontinence
A 27-year-old woman (G1P1) presents 2 months following spontaneous vaginal delivery with symptoms of urine leakage with laughing and running. Her urinary incontinence has been improving since delivery, but it continues to be bothersome.
What would you recommend for this patient?
Conservative SUI management strategies in pregnancy
Urinary tract symptoms are common in pregnancy, with up to 41.8% of women reporting urinary symptom distress in the third trimester.5 During pregnancy, estrogen and progesterone decrease urethral pressure that, together with increased intra-abdominal pressure from the gravid uterus, can cause or worsen stress urinary incontinence (SUI).6
During pregnancy, women should be offered conservative therapies for SUI. For women who can perform a pelvic floor contraction (a Kegel exercise), self-guided pelvic floor muscle exercises (PFMEs) may be helpful (see “Pelvic floor muscle exercises” below). We recommend that women start with 1 to 2 sets of 10 Kegel exercises per day and that they hold the squeeze for 2 to 3 seconds, working up to holding for 10 seconds. The goal is to strengthen and improve muscle control so that the Kegel squeeze can be paired with activities that cause SUI.
For women who are unable to perform a Kegel exercise or are not improving with a home PFME regimen, referral to pelvic floor physical therapy (PFPT) can be considered. While data support the efficacy of PFPT for SUI treatment in nonpregnant women,7 data are lacking on PFME in pregnancy.
In women without urinary incontinence, PFME in early pregnancy can prevent the onset of incontinence in late pregnancy and the postpartum period.8 By contrast, the same 2020 Cochrane Review found no evidence that antenatal pelvic floor muscle therapy in incontinent women decreases incontinence in mid- or late-pregnancy or in the postpartum period.8 As the quality of this evidence is very low and there is no evidence of harm with PFME, we continue to recommend it for women with bothersome SUI.
Incontinence pessaries or vaginal inserts (such as Poise Impressa bladder supports) can be helpful for SUI treatment. An incontinence pessary can be fitted in the office, and fitting kits are available for both. Pessaries can safely be used in pregnancy, but there are no data on the efficacy of pessaries for treating SUI in pregnancy. In nonpregnant women, evidence demonstrates 63% satisfaction 3 months post–pessary placement for SUI.7
We do not recommend invasive procedures for the treatment of SUI during pregnancy or in the first 6 months following delivery. There is no evidence that elective cesarean delivery prevents persistent SUI postpartum.9
To identify and engage the proper pelvic floor muscles:
- Insert a finger in the vagina and squeeze the vaginal muscles around your finger.
- Imagine you are sitting on a marble and have to pick it up with the vaginal muscles.
- Squeeze the muscles you would use to stop the flow of urine or hold back flatulence.
Perform sets of 10, 2 to 3 times per day as follows:
- Squeeze: Engage the pelvic floor muscles as described above; avoid performing Kegels while voiding.
- Hold: For 2 to 10 seconds; increase the duration to 10 seconds as able.
- Relax: Completely relax muscles before initating the next squeeze.
Reference
1. UpToDate. Patient education: pelvic muscle (Kegel) exercises (the basics). 2018. https://uptodatefree.ir/topic.htm?path=pelvic-muscle-kegel-exercises-the-basics. Accessed February 24, 2021.
Continue to: Managing SUI in the postpartum period...
Managing SUI in the postpartum period
After the first 6 months postpartum and exhaustion of conservative measures, we offer surgical interventions for women with persistent, bothersome incontinence. Surgery for SUI typically is not recommended until childbearing is complete, but it can be considered if the patient’s bother is significant.
For women with bothersome SUI who still desire future pregnancy, management options include periurethral bulking, a retropubic urethropexy (Burch procedure), or a midurethral sling procedure. Women who undergo an anti-incontinence procedure have an increased risk for urinary retention during a subsequent pregnancy.10 Most women with a midurethral sling will continue to be continent following an obstetric delivery.
Anticipatory guidance
At 3 months postpartum, the incidence of urinary incontinence is 6% to 40%, depending on parity and delivery type. Postpartum urinary incontinence is most common after instrumented vaginal delivery (32%) followed by spontaneous vaginal delivery (28%) and cesarean delivery (15%). The mean prevalence of any type of urinary incontinence is 33% at 3 months postpartum, and only small changes in the rate of urinary incontinence occur over the first postpartum year.11 While urinary incontinence is common postpartum, it should not be considered normal. We counsel that symptoms may improve spontaneously, but treatment can be initiated if the patient experiences significant bother.
A longitudinal cohort study that followed women from 3 months to 12 years postpartum found that, of women with urinary incontinence at 3 months postpartum, 76% continued to report incontinence at 12 years postpartum.12 We recommend that women be counseled that, even when symptoms resolve, they remain at increased risk for urinary incontinence in the future. Invasive therapies should be used to treat bothersome urinary incontinence, not to prevent future incontinence.
CASE 3 Fecal incontinence
A 24-year-old woman (G1P1) presents 3 weeks postpartum following a forceps-assisted vaginal delivery complicated by a 3c laceration. She reports fecal urgency, inability to control flatus, and once-daily fecal incontinence.
How would you evaluate these symptoms?
Steps in evaluation
The initial evaluation should include an inquiry regarding the patient’s stool consistency and bowel regimen. The Bristol stool form scale can be used to help patients describe their typical bowel movements (TABLE 2).13 During healing, the goal is to achieve a Bristol type 4 stool, both to avoid straining and to improve continence, as loose stool is the most difficult to control.
A physical examination can evaluate healing and sphincter integrity; it should include inspection of the distal vagina and perineal body and a digital rectal exam. Anal canal resting tone and squeeze strength should be evaluated, and the digital rectal examination scoring system (DRESS) can be useful for quantification (TABLE 3).14 Lack of tone at rest in the anterolateral portion of the sphincter complex can indicate an internal anal sphincter defect, as 80% of the resting tone comes from this muscle (FIGURE).15
The rectovaginal septum should be assessed given the increased risk of rectovaginal fistula in women with obstetric anal sphincter injury (OASI). The patient should be instructed to contract the anal sphincter, allowing evaluation of muscular contraction. Lack of contraction anteriolaterally may indicate external anal sphincter separation.
Continue to: Conservative options for improving fecal incontinence symptoms...
Conservative options for improving fecal incontinence symptoms
The patient can be counseled regarding stool bulking, first with insoluble fiber supplementation and cessation of stool softeners if she is incontinent of liquid stool. If these measures are not effective, use of a constipating agent, such as loperamide, can improve stool consistency and thereby decrease incontinence episodes. PFPT with biofeedback can be offered as well. While typically we do not recommend initiating PFPT before 6 weeks postpartum, so the initial phases of healing can occur, early referral enables the patient to avoid a delay in access to care.
The patient also can be counseled about a referral to a pelvic floor specialist for further evaluation. A variety of peripartum pelvic floor disorder clinics are being established by Female Pelvic Medicine and Reconstructive Surgery (FPMRS) physicians. These clinics provide the benefit of comprehensive care for pelvic floor disorders in this unique population.
When conservative measures fail. If a patient has persistent bowel control issues despite conservative measures, a referral to an FPMRS physician should be initiated.
Delivery route in future pregnancies
The risk of a subsequent OASI is low. While this means that many women can safely pursue a future vaginal delivery, a scheduled cesarean delivery is indicated for women with persistent bowel control issues, wound healing complications, and those who experienced psychological trauma from their delivery.16 We recommend a shared-decision making approach, reviewing modifiable and nonmodifiable risk factors to help determine whether or not a future vaginal birth is appropriate. It is important to highlight that a cesarean delivery does not protect against fecal incontinence in women with a history of OASI; however, there is benefit in preventing worsening of anal incontinence, if present.17
CASE 4 Uterovaginal prolapse
A 36-year-old woman (G3P3) presents for her routine postpartum visit at 6 weeks after a spontaneous vaginal delivery without lacerations. She reports a persistent feeling of vaginal pressure and fullness. She thinks she felt a bulge with wiping after a bowel movement.
What options are available for this patient?
Prolapse in the peripartum population
Previous studies have revealed an increased prevalence of POP in pregnant women on examination compared with their nulligravid counterparts (47.6% vs 0%).18 With the changes in the hormonal milieu in pregnancy, as well as the weight of the gravid uterus on the pelvic floor, it is not surprising that pregnancy may be the inciting event to expose even transient defects in pelvic organ support.19
It is well established that increasing parity and, to a lesser extent, larger babies are associated with increased risk for future POP and surgery for prolapse. In the first year postpartum, nearly one-third of women have stage 2 or greater prolapse on exam, with studies demonstrating an increased prevalence of postpartum POP in women who delivered vaginally compared with those who delivered by cesarean.20,21
Initial evaluation
Diagnosis can be made during a routine pelvic exam by having the patient perform a Valsalva maneuver while in the lithotomy position. Using half of a speculum permits evaluation of the anterior and posterior vaginal walls separately, and Valsalva during a bimanual exam can aid in evaluating descensus of the uterus and cervix.
Excellent free patient education resources available online through the American Urogynecologic Society and the International Urogynecological Association can be used to direct counseling.
Continue to: Treatments you can offer for POP...
Treatments you can offer for POP
For pregnant or postpartum patients with bothersome prolapse, initial management options include pessary fitting and/or PFPT referral. In pregnancy, women often can be successfully fitted with a pessary for POP; however, as expulsion is a common issue, selection of a stiffer or space-occupying device may be more efficacious.
Often, early onset POP in pregnancy resolves as the gravid uterus lifts out of the pelvis in the second trimester, at which time the pessary can be discontinued. In the postpartum period, a pessary fitting can be undertaken similarly to that in nonpregnant patients. While data are lacking in the peripartum population, evidence supports the positive impact of PFPT on improving POP symptom bother.22 Additionally, for postpartum women who experience OASI, PFPT can produce significant improvement in subjective POP and associated bother.23
Impact of future childbearing wishes on treatment
The desire for future childbearing does not preclude treatment of patients experiencing bother from POP after conservative management options have failed. Both vaginal native tissue and mesh-augmented uterine-sparing repairs are performed by many FPMRS specialists and are associated with good outcomes. As with SUI, we do not recommend invasive treatment for POP during pregnancy or before 6 months postpartum.
In conclusion
Obstetric specialists play an essential role in caring for women with PFDs in the peripartum period. Basic evaluation, counseling, and management can be initiated using many of the resources already available in an obstetric ambulatory practice. Important adjunctive resources include those available for both providers and patients through the American Urogynecologic Society and the International Urogynecological Association. In addition, clinicians can partner with pelvic floor specialists through the growing number of FPMRS-run peripartum pelvic floor disorder clinics across the country and pelvic floor physical therapists.
If these specialty clinics and therapists are not available in your area, FPMRS specialists, urologists, gastroenterologists, and/or colorectal surgeons can aid in patient diagnosis and management to reach the ultimate goal of improving PFDs in this at-risk population. ●
- Madsen AM, Hickman LC, Propst K. Recognition and management of pelvic floor disorders in pregnancy and the postpartum period. Obstet Gynecol Clin North Am. Forthcoming 2021.
- Bodner-Adler B, Kimberger O, Laml T, et al. Prevalence and risk factors for pelvic floor disorders during early and late pregnancy in a cohort of Austrian women. Arch Gynecol Obstet. 2019;300:1325-1330.
- Swenson CW, DePorre JA, Haefner JK, et al. Postpartum depression screening and pelvic floor symptoms among women referred to a specialty postpartum perineal clinic. Am J Obstet Gynecol. 2018;218:335.e1-335.e6.
- Skinner EM, Dietz HP. Psychological and somatic sequelae of traumatic vaginal delivery: a literature review. Aust N Z J Obstet Gynaecol. 2015;55:309-314.
- Yohay D, Weintraub AY, Mauer-Perry N, et al. Prevalence and trends of pelvic floor disorders in late pregnancy and after delivery in a cohort of Israeli women using the PFDI-20. Eur J Obstet Gynecol Reprod Biol. 2016;200:35-39.
- Gregory WT, Sibai BM. Obstetrics and pelvic floor disorders. In: Walters M, Karram M, eds. Urogynecology and Reconstructive Pelvic Surgery. 4th ed. Philadelphia, PA: Saunders; 2015:224-237.
- Richter HE, Burgio KL, Brubaker L, et al; Pelvic Floor Disorders Network. Continence pessary compared with behavioral therapy or combined therapy for stress incontinence: a randomized controlled trial. Obstet Gynecol. 2010;115:609-617.
- Woodley SJ, Lawrenson P, Boyle R, et al. Pelvic floor muscle training for preventing and treating urinary and faecal incontinence in antenatal and postnatal women. Cochrane Database Syst Rev. 2020;6:CD007471.
- Foldspang A, Hvidman L, Mommsen S, et al. Risk of postpartum urinary incontinence associated with pregnancy and mode of delivery. Acta Obstet Gynecol Scand. 2004;83:923-927.
- Wieslander CK, Weinstein MM, Handa V, et al. Pregnancy in women with prior treatments for pelvic floor disorders. Female Pelvic Med Reconstr Surg. 2020;26:299-305.
- Thom DH, Rortveit G. Prevalence of postpartum urinary incontinence: a systematic review. Acta Obstet Gynecol Scand. 2010;89:1511-1522.
- MacArthur C, Wilson D, Herbison P, et al; Prolong Study Group. Urinary incontinence persisting after childbirth: extent, delivery history, and effects in a 12-year longitudinal cohort study. BJOG. 2016;123:1022-1029.
- Blake MR, Raker JM, Whelan K. Validity and reliability of the Bristol Stool Form Scale in healthy adults and patients with diarrhoea-predominant irritable bowel syndrome. Aliment Pharmacol Ther. 2016;44:693-703
- Orkin BA, Sinykin SB, Lloyd PC. The digital rectal examination scoring system (DRESS). Dis Colon Rectum. 2010;53:1656-1660.
- UpToDate. Repair of episiotomy and perineal lacerations associated with childbirth. 2020. https://www-uptodate-com .ccmain.ohionet.org/contents/repair-of-perineal-and-other -lacerations-associated-with-childbirth?search=repair%20 episiotomy&source=search_result&selectedTitle=1~150&usa ge_type=default&display_rank=1. Accessed February 28, 2021.
- Committee on Practice Bulletins–Obstetrics. ACOG practice bulletin no. 198: prevention and management of obstetric lacerations at vaginal delivery. Obstet Gynecol. 2018;132:e87-e102.
- Jangö H, Langhoff-Roos J, Rosthøj S, et al. Long-term anal incontinence after obstetric anal sphincter injury—does grade of tear matter? Am J Obstet Gynecol. 2018;218:232.e1-232.e10.
- O’Boyle AL, Woodman PJ, O’Boyle JD, et al. Pelvic organ support in nulliparous pregnant and nonpregnant women: a case control study. Am J Obstet Gynecol. 2002;187:99-102.
- Handa VL, Blomquist JL, McDermott KC, et al. Pelvic floor disorders after vaginal birth. Obstet Gynecol. 2012;119 (2, pt 1):233-239.
- Handa VL, Nygaard I, Kenton K, et al; Pelvic Floor Disorders Network. Pelvic organ support among primiparous women in the first year after childbirth. Int Urogynecol J Pelvic Floor Dysfunct. 2009;20:1407-1411.
- O’Boyle AL, O’Boyle JD, Calhoun B, et al. Pelvic organ support in pregnancy and postpartum. Int Urogynecol J Pelvic Floor Dysfunct. 2005;16:69-72.
- Hagen S, Stark D, Glazener C, et al; POPPY Trial Collaborators. Individualised pelvic floor muscle training in women with pelvic organ prolapse (POPPY): a multicentre randomised controlled trial. Lancet. 2014;383:796-806.
- Von Bargen E, Haviland MJ, Chang OH, et al. Evaluation of postpartum pelvic floor physical therapy on obstetrical anal sphincter injury: a randomized controlled trial. Female Pelvic Med Reconstr Surg. 2020. doi: 10.1097/SPV.0000000000000849.
- Madsen AM, Hickman LC, Propst K. Recognition and management of pelvic floor disorders in pregnancy and the postpartum period. Obstet Gynecol Clin North Am. Forthcoming 2021.
- Bodner-Adler B, Kimberger O, Laml T, et al. Prevalence and risk factors for pelvic floor disorders during early and late pregnancy in a cohort of Austrian women. Arch Gynecol Obstet. 2019;300:1325-1330.
- Swenson CW, DePorre JA, Haefner JK, et al. Postpartum depression screening and pelvic floor symptoms among women referred to a specialty postpartum perineal clinic. Am J Obstet Gynecol. 2018;218:335.e1-335.e6.
- Skinner EM, Dietz HP. Psychological and somatic sequelae of traumatic vaginal delivery: a literature review. Aust N Z J Obstet Gynaecol. 2015;55:309-314.
- Yohay D, Weintraub AY, Mauer-Perry N, et al. Prevalence and trends of pelvic floor disorders in late pregnancy and after delivery in a cohort of Israeli women using the PFDI-20. Eur J Obstet Gynecol Reprod Biol. 2016;200:35-39.
- Gregory WT, Sibai BM. Obstetrics and pelvic floor disorders. In: Walters M, Karram M, eds. Urogynecology and Reconstructive Pelvic Surgery. 4th ed. Philadelphia, PA: Saunders; 2015:224-237.
- Richter HE, Burgio KL, Brubaker L, et al; Pelvic Floor Disorders Network. Continence pessary compared with behavioral therapy or combined therapy for stress incontinence: a randomized controlled trial. Obstet Gynecol. 2010;115:609-617.
- Woodley SJ, Lawrenson P, Boyle R, et al. Pelvic floor muscle training for preventing and treating urinary and faecal incontinence in antenatal and postnatal women. Cochrane Database Syst Rev. 2020;6:CD007471.
- Foldspang A, Hvidman L, Mommsen S, et al. Risk of postpartum urinary incontinence associated with pregnancy and mode of delivery. Acta Obstet Gynecol Scand. 2004;83:923-927.
- Wieslander CK, Weinstein MM, Handa V, et al. Pregnancy in women with prior treatments for pelvic floor disorders. Female Pelvic Med Reconstr Surg. 2020;26:299-305.
- Thom DH, Rortveit G. Prevalence of postpartum urinary incontinence: a systematic review. Acta Obstet Gynecol Scand. 2010;89:1511-1522.
- MacArthur C, Wilson D, Herbison P, et al; Prolong Study Group. Urinary incontinence persisting after childbirth: extent, delivery history, and effects in a 12-year longitudinal cohort study. BJOG. 2016;123:1022-1029.
- Blake MR, Raker JM, Whelan K. Validity and reliability of the Bristol Stool Form Scale in healthy adults and patients with diarrhoea-predominant irritable bowel syndrome. Aliment Pharmacol Ther. 2016;44:693-703
- Orkin BA, Sinykin SB, Lloyd PC. The digital rectal examination scoring system (DRESS). Dis Colon Rectum. 2010;53:1656-1660.
- UpToDate. Repair of episiotomy and perineal lacerations associated with childbirth. 2020. https://www-uptodate-com .ccmain.ohionet.org/contents/repair-of-perineal-and-other -lacerations-associated-with-childbirth?search=repair%20 episiotomy&source=search_result&selectedTitle=1~150&usa ge_type=default&display_rank=1. Accessed February 28, 2021.
- Committee on Practice Bulletins–Obstetrics. ACOG practice bulletin no. 198: prevention and management of obstetric lacerations at vaginal delivery. Obstet Gynecol. 2018;132:e87-e102.
- Jangö H, Langhoff-Roos J, Rosthøj S, et al. Long-term anal incontinence after obstetric anal sphincter injury—does grade of tear matter? Am J Obstet Gynecol. 2018;218:232.e1-232.e10.
- O’Boyle AL, Woodman PJ, O’Boyle JD, et al. Pelvic organ support in nulliparous pregnant and nonpregnant women: a case control study. Am J Obstet Gynecol. 2002;187:99-102.
- Handa VL, Blomquist JL, McDermott KC, et al. Pelvic floor disorders after vaginal birth. Obstet Gynecol. 2012;119 (2, pt 1):233-239.
- Handa VL, Nygaard I, Kenton K, et al; Pelvic Floor Disorders Network. Pelvic organ support among primiparous women in the first year after childbirth. Int Urogynecol J Pelvic Floor Dysfunct. 2009;20:1407-1411.
- O’Boyle AL, O’Boyle JD, Calhoun B, et al. Pelvic organ support in pregnancy and postpartum. Int Urogynecol J Pelvic Floor Dysfunct. 2005;16:69-72.
- Hagen S, Stark D, Glazener C, et al; POPPY Trial Collaborators. Individualised pelvic floor muscle training in women with pelvic organ prolapse (POPPY): a multicentre randomised controlled trial. Lancet. 2014;383:796-806.
- Von Bargen E, Haviland MJ, Chang OH, et al. Evaluation of postpartum pelvic floor physical therapy on obstetrical anal sphincter injury: a randomized controlled trial. Female Pelvic Med Reconstr Surg. 2020. doi: 10.1097/SPV.0000000000000849.
Management of a Child vs an Adult Presenting With Acral Lesions During the COVID-19 Pandemic: A Practical Review
There has been a rise in the prevalence of perniolike lesions—erythematous to violaceous, edematous papules or nodules on the fingers or toes—during the coronavirus disease 2019 (COVID-19) pandemic. These lesions are referred to as “COVID toes.” Although several studies have suggested an association with these lesions and COVID-19, and coronavirus particles have been identified in endothelial cells of biopsies of pernio lesions, questions remain on the management, pathophysiology, and implications of these lesions.1 We provide a practical review for primary care clinicians and dermatologists on the current management, recommendations, and remaining questions, with particular attention to the distinctions for children vs adults presenting with pernio lesions.
Hypothetical Case of a Child Presenting With Acral Lesions
A 7-year-old boy presents with acute-onset, violaceous, mildly painful and pruritic macules on the distal toes that began 3 days earlier and have progressed to involve more toes and appear more purpuric. A review of symptoms reveals no fever, cough, fatigue, or viral symptoms. He has been staying at home for the last few weeks with his brother, mother, and father. His father is working in delivery services and is social distancing at work but not at home. His mother is concerned about the lesions, if they could be COVID toes, and if testing is needed for the patient or family. In your assessment and management of this patient, you consider the following questions.
What Is the Relationship Between These Clinical Findings and COVID-19?
Despite negative polymerase chain reaction (PCR) tests reported in cases of chilblains during the COVID-19 pandemic as well as the possibility that these lesions are an indirect result of environmental factors or behavioral changes during quarantine, the majority of studies favor an association between these chilblains lesions and COVID-19 infection.2,3 Most compellingly, COVID-19 viral particles have been identified by immunohistochemistry and electron microscopy in the endothelial cells of biopsies of these lesions.1 Additionally, there is evidence for possible associations of other viruses, including Epstein-Barr virus and parvovirus B19, with chilblains lesions.4,5 In sum, with the lack of any large prospective study, the weight of current evidence suggests that these perniolike skin lesions are not specific markers of infection with severe acute respiratory syndrome coronavirus 2 (SARS-CoV-2).6
Published studies differ in reporting the coincidence of perniolike lesions with typical COVID-19 symptoms, including fever, dyspnea, cough, fatigue, myalgia, headache, and anosmia, among others. Some studies have reported that up to 63% of patients with reported perniolike lesions developed typical COVID-19 symptoms, but other studies found that no patients with these lesions developed symptoms.6-11 Studies with younger cohorts tended to report lower prevalence of COVID-19 symptoms, and within cohorts, younger patients tended to have less severe symptoms. For example, 78.8% of patients in a cohort (n=58) with an average age of 14 years did not experience COVID-19–related symptoms.6 Based on these data, it has been hypothesized that patients with chilblainslike lesions may represent a subpopulation who will have a robust interferon response that is protective from more symptomatic and severe COVID-19.12-14
Current evidence suggests that these lesions are most likely to occur between 9 days and 2 months after the onset of COVID-19 symptoms.4,9,10 Most cases have been only mildly symptomatic, with an overall favorable prognosis of both lesions and any viral symptoms.8,10 The lesions typically resolve without treatment within a few days of initial onset.15,16
What Should Be the Workup and Management of These Lesions?
Given the currently available information and favorable prognosis, usually no further workup specific to the perniolike lesions is required in the case of an asymptomatic child presenting with acral lesions, and the majority of management will center around patient and parent/guardian education and reassurance. When asked by the patient’s parent, “What does it mean that my child has these lesions?”, clinicians can provide information on the possible association with COVID-19 and the excellent, self-resolving prognosis. An example of honest and reasonable phrasing with current understanding might be, “We are currently not certain if COVID-19 causes these lesions, although there are data to suggest that they are associated. There are a lot of data showing that children with these lesions either do not have any symptoms or have very mild symptoms that resolve without treatment.”
For management, important considerations include how painful the lesions are to the individual patient and how they affect quality of life. If less severe, clinicians can reassure patients and parents/guardians that the lesions will likely self-resolve without treatment. If worsening or symptomatic, clinicians can try typical treatments for chilblains, such as topical steroids, whole-body warming, and nifedipine.17-19 Obtaining a review of symptoms, including COVID-19 symptoms and general viral symptoms, is important given the rare cases of children with severe COVID-19.20,21
The question of COVID-19 testing as related to these lesions remains controversial, and currently there are still differing perspectives on the need for biopsy, PCR for COVID-19, or serologies for COVID-19 in patients presenting with these lesions. Some experts report that additional testing is not needed in the pediatric population because of the high frequency of negative testing reported to date.22,23 However, these children may be silent carriers, and until more is known about their potential to transmit the virus, testing may be considered if resources allow, particularly if the patient has a known exposure.10,12,16,24 The ultimate decision to pursue biopsy or serologic workup for COVID-19 remains up to clinical discretion with consideration of symptoms, severity, and immunocompromised household contacts. If lesions developed after infection, PCR likely will result negative, whereas serologic testing may reveal antibodies.
Hypothetical Case of an Adult Presenting With Acral Lesions and COVID-19 Symptoms
A 50-year-old man presents with acute-onset, violaceous, painful, edematous plaques on the distal toes that began 3 days earlier and have progressed to include the soles. A review of symptoms reveals fever (temperature, 38.4 °C [101 °F]), cough, dyspnea, diarrhea, and severe asthenia. He has had interactions with a coworker who recently tested positive for COVID-19.
How Should You Consider These Lesions in the Context of the Other Symptoms Concerning for COVID-19?
In contrast to the asymptomatic child above, this adult has chilblainslike lesions and viral symptoms. In adults, chilblainslike lesions have been associated with relatively mild COVID-19, and patients with these lesions who are otherwise asymptomatic have largely tested negative for COVID-19 by PCR and serologic antibody testing.11,25,26
True acral ischemia, which is more severe and should be differentiated from chilblains, has been reported in critically ill patients.9 Additionally, studies have found that retiform purpura is the most common cutaneous finding in patients with severe COVID-19.27 For this patient, who has an examination consistent with progressive and severe chilblainslike lesions and suspicion for COVID-19 infection, it is important to observe and monitor these lesions, as clinical progression suggestive of acral ischemia or retiform purpura should be taken seriously and may indicate worsening of the underlying disease. Early intervention with anticoagulation might be considered, though there currently is no evidence of successful treatment.28
What Causes These Lesions in a Patient With COVID-19?
The underlying pathophysiology has been proposed to be a monocytic-macrophage–induced hyperinflammatory systemic state that damages the lungs, as well as the gastrointestinal, renal, and endothelial systems. The activation of the innate immune system triggers a cytokine storm that creates a hypercoagulable state that ultimately can manifest as superficial thromboses, leading to gangrene of the extremities. Additionally, interferon response and resulting hypercytokinemia may cause direct cytopathic damage to the endothelium of arterioles and capillaries, causing the development of papulovesicular lesions that resemble the chilblainslike lesions observed in children.29 In contrast to children, who typically have no or mild COVID-19 symptoms, adults may have a delayed interferon response, which has been proposed to allow for more severe manifestations of infection.12,30
How Should an Adult With Perniolike Lesions Be Managed?
Adults with chilblainslike lesions and no other signs or symptoms of COVID-19 infection do not necessarily need be tested for COVID-19, given the reports demonstrating most patients in this clinical situation will have negative PCRs and serologies for antibodies. However, there have been several reports of adults with acro-ischemic skin findings who also had severe COVID-19, with an observed incidence of 23% in intensive care unit patients with COVID-19.27,28,31,32 If there is suspicion of infection with COVID-19, it is advisable to first obtain workup for COVID-19 and other viruses that can cause acral lesions, including Epstein-Barr virus and parvovirus. Other pertinent laboratory tests may include D-dimer, fibrinogen, prothrombin time, activated partial thromboplastin time, antithrombin activity, platelet count, neutrophil count, procalcitonin, triglycerides, ferritin, C-reactive protein, and hemoglobin. For patients with evidence of worsening acro-ischemia, regular monitoring of these values up to several times per week can allow for initiation of vascular intervention, including angiontensin-converting enzyme inhibitors, statins, or antiplatelet drugs.32 The presence of antiphospholipid antibodies also has been associated with critically ill patients who develop digit ischemia as part of the sequelae of COVID-19 infection and therefore may act as an important marker for the potential to develop disseminated intravascular coagulation in this patient.33 Even if COVID-19 infection is not suspected, a thorough review of systems is important to look for an underlying connective tissue disease, such as systemic lupus erythematosus, which is associated with pernio. Associated symptoms may warrant workup with antinuclear antibodies and other appropriate autoimmune serologies.
If there is any doubt of the diagnosis, the patient is experiencing symptoms from the lesion, or the patient is experiencing other viral symptoms, it is appropriate to biopsy immediately to confirm the diagnosis. Prior studies have identified fibrin clots, angiocentric and eccrinotropic lymphocytic infiltrates, lymphocytic vasculopathy, and papillary dermal edema as the most common features in chilblainslike lesions during the COVID-19 pandemic.9
For COVID-19 testing, many studies have revealed adult patients with an acute hypercoagulable state testing positive by SARS-CoV-2 PCR. These same patients also experienced thromboembolic events shortly after testing positive for COVID-19, which suggests that patients with elevated D-dimer and fibrinogen likely will have a viral load that is sufficient to test positive for COVID-19.32,34-36 It is appropriate to test all patients with suspected COVID-19, especially adults who are more likely to experience adverse complications secondary to infection.
This patient experiencing COVID-19 symptoms with signs of acral ischemia is likely to test positive by PCR, and additional testing for serologic antibodies is unlikely to be clinically meaningful in this patient’s state. Furthermore, there is little evidence that serology is reliable because of the markedly high levels of both false-negative and false-positive results when using the available antibody testing kits.37 The latter evidence makes serology testing of little value for the general population, but particularly for patients with acute COVID-19.
Conclusion and Outstanding Questions
There is evidence suggesting an association between chilblainslike lesions and COVID-19.11,22,38,39 Children presenting with these lesions have an excellent prognosis and only need a workup or treatment if there are other symptoms, as the lesions self-resolve in the majority of reported cases.7-9 Adults presenting with these lesions and without symptoms likewise are unlikely to test positive for COVID-19, and the lesions typically resolve spontaneously or with first-line treatment. However, adults presenting with these lesions and COVID-19 symptoms should raise clinical concern for evolving skin manifestations of acro-ischemia. If the diagnosis is uncertain or systemic symptoms are concerning, biopsy, COVID-19 PCR, and other appropriate laboratory workup should be obtained.
There remains controversy and uncertainty over the relationship between these skin findings and SARS-CoV-2 infection, with clinical evidence to support both a direct relationship representing convalescent-phase cutaneous reaction as well as an indirect epiphenomenon. If there was a direct relationship, we would have expected to see a rise in the incidence of acral lesions proportionate to the rising caseload of COVID-19 after the reopening of many states in the summer of 2020. Similarly, because young adults represent the largest demographic of increasing cases and as some schools have remained open for in-person instruction during the current academic year, we also would have expected the incidence of chilblains-like lesions presenting to dermatologists and pediatricians to increase alongside these cases. Continued evaluation of emerging literature and ongoing efforts to understand the cause of this observed phenomenon will hopefully help us arrive at a future understanding of the pathophysiology of this puzzling skin manifestation.40
- Colmenero I, Santonja C, Alonso-Riaño M, et al. SARS-CoV-2 endothelial infection causes COVID-19 chilblains: histopathological, immunohistochemical and ultrastructural study of seven paediatric cases. Br J Dermatol. 2020;183:729-737. doi:10.1111/bjd.19327
- Neri I, Virdi A, Corsini I, et al. Major cluster of paediatric “true” primary chilblains during the COVID-19 pandemic: a consequence of lifestyle changes due to lockdown. J Eur Acad Dermatol Venereol. 2020;34:2630-2635. doi:10.1111/jdv.16751
- Hubiche T, Le Duff F, Chiaverini C, et al. Negative SARS-CoV-2 PCR in patients with chilblain-like lesions [letter]. Lancet Infect Dis. June 18, 2020. doi:10.1016/S1473-3099(20)30518-1
- Pistorius MA, Blaise S, Le Hello C, et al. Chilblains and COVID19 infection: causality or coincidence? How to proceed? J Med Vasc. 2020;45:221-223. doi:10.1016/j.jdmv.2020.05.002
- Massey PR, Jones KM. Going viral: a brief history of Chilblain-like skin lesions (“COVID toes”) amidst the COVID-19 pandemic. Semin Oncol. 2020;47:330-334. doi:10.1053/j.seminoncol.2020.05.012
- Docampo-Simón A, Sánchez-Pujol MJ, Juan-Carpena G, et al. Are chilblain-like acral skin lesions really indicative of COVID-19? A prospective study and literature review [letter]. J Eur Acad Dermatol Venereol. 2020;34:e445-e446. doi:10.1111/jdv.16665
- El Hachem M, Diociaiuti A, Concato C, et al. A clinical, histopathological and laboratory study of 19 consecutive Italian paediatric patients with chilblain-like lesions: lights and shadows on the relationship with COVID-19 infection. J Eur Acad Dermatol Venereol. 2020;34:2620-2629. doi:10.1111/jdv.16682
- Recalcati S, Barbagallo T, Frasin LA, et al. Acral cutaneous lesions in the time of COVID-19. J Eur Acad Dermatol Venereol. 2020;34:e346-e347. doi:10.1111/jdv.16533
- Andina D, Noguera-Morel L, Bascuas-Arribas M, et al. Chilblains in children in the setting of COVID-19 pandemic. Pediatr Dermatol. 2020;37:406-411. doi:10.1111/pde.14215
- Casas CG, Català A, Hernández GC, et al. Classification of the cutaneous manifestations of COVID-19: a rapid prospective nationwide consensus study in Spain with 375 cases. Br J Dermatol. 2020;183:71-77. doi:10.1111/bjd.19163
- Freeman EE, McMahon DE, Lipoff JB, et al. Pernio-like skin lesions associated with COVID-19: a case series of 318 patients from 8 countries. J Am Acad Dermatol. 2020;83:486-492. doi:10.1016/j.jaad.2020.05.109
- Kolivras A, Dehavay F, Delplace D, et al. Coronavirus (COVID-19) infection–induced chilblains: a case report with histopathologic findings. JAAD Case Rep. 2020;6:489-492. doi:10.1016/j.jdcr.2020.04.011
- Damsky W, Peterson D, King B. When interferon tiptoes through COVID-19: pernio-like lesions and their prognostic implications during SARS-CoV-2 infection. J Am Acad Dermatol. 2020;83:E269-E270. doi:10.1016/j.jaad.2020.06.052
- Lipsker D. A chilblain epidemic during the COVID-19 pandemic. A sign of natural resistance to SARS-CoV-2? Med Hypotheses. 2020;144:109959. doi:10.1016/j.mehy.2020.109959
- Kaya G, Kaya A, Saurat J-H. Clinical and histopathological features and potential pathological mechanisms of skin lesions in COVID-19: review of the literature. Dermatopathology. 2020;7:3-16. doi:10.3390/dermatopathology7010002
- Pavone P, Marino S, Marino L, et al. Chilblains-like lesions and SARS-CoV-2 in children: An overview in therapeutic approach. Dermatol Ther. 2021;34:E14502. doi:https://doi.org/10.1111/dth.14502
- Dowd PM, Rustin MH, Lanigan S. Nifedipine in the treatment of chilblains. Br Med J (Clin Res Ed). 1986;293:923-924. doi:10.1136/bmj.293.6552.923-a
- Rustin MH, Newton JA, Smith NP, et al. The treatment of chilblains with nifedipine: the results of a pilot study, a double-blind placebo-controlled randomized study and a long-term open trial. Br J Dermatol. 1989;120:267-275. doi:10.1111/j.1365-2133.1989.tb07792.x
- Almahameed A, Pinto DS. Pernio (chilblains). Curr Treat Options Cardiovasc Med. 2008;10:128-135. doi:10.1007/s11936-008-0014-0
- Chen F, Liu ZS, Zhang FR, et al. First case of severe childhood novel coronavirus pneumonia in China [in Chinese]. Zhonghua Er Ke Za Zhi. 2020;58:179-182. doi:10.3760/cma.j.issn.0578-1310.2020.03.003
- Choi S-H, Kim HW, Kang J-M, et al. Epidemiology and clinical features of coronavirus disease 2019 in children. Clin Exp Pediatr. 2020;63:125-132. doi:10.3345/cep.2020.00535
- Piccolo V, Neri I, Manunza F, et al. Chilblain-like lesions during the COVID-19 pandemic: should we really worry? Int J Dermatol. 2020;59:1026-1027. doi:10.1111/ijd.1499
- Roca-Ginés J, Torres-Navarro I, Sánchez-Arráez J, et al. Assessment of acute acral lesions in a case series of children and adolescents during the COVID-19 pandemic. JAMA Dermatol. 2020;156:992-997. doi:10.1001/jamadermatol.2020.2340
- Landa N, Mendieta-Eckert M, Fonda-Pascual P, et al. Chilblain-like lesions on feet and hands during the COVID-19 pandemic. Int J Dermatol. 2020;59:739-743. doi:10.1111/ijd.14937
- Herman A, Peeters C, Verroken A, et al. Evaluation of chilblains as a manifestation of the COVID-19 pandemic. JAMA Dermatol. 2020;156:998-1003. doi:10.1001/jamadermatol.2020.2368
- Daneshjou R, Rana J, Dickman M, et al. Pernio-like eruption associated with COVID-19 in skin of color. JAAD Case Rep. 2020;6:892-897. doi:10.1016/j.jdcr.2020.07.009
- Freeman EE, McMahon DE, Lipoff JB, et al. The spectrum of COVID-19-associated dermatologic manifestations: an international registry of 716 patients from 31 countries. J Am Acad Dermatol. 2020;83:1118-1129. doi:10.1016/j.jaad.2020.06.1016
- Zhang Y, Cao W, Xiao M, et al. Clinical and coagulation characteristics of 7 patients with critical COVID-2019 pneumonia and acro-ischemia [in Chinese]. Zhonghua Xue Ye Xue Za Zhi. 2020;41:E006. doi:10.3760/cma.j.issn.0253-2727.2020.0006
- Criado PR, Abdalla BMZ, de Assis IC, et al. Are the cutaneous manifestations during or due to SARS-CoV-2 infection/COVID-19 frequent or not? revision of possible pathophysiologic mechanisms. Inflamm Res. 2020;69:745-756. doi:10.1007/s00011-020-01370-w
- Park A, Iwasaki A. Type I and type III interferons—induction, signaling, evasion, and application to combat COVID-19. Cell Host Microbe. 2020;27:870-878. doi:10.1016/j.chom.2020.05.008
- Wollina U, Karadag˘ AS, Rowland-Payne C, et al. Cutaneous signs in COVID-19 patients: a review. Dermatol Ther. 2020;33:E13549. doi:10.1111/dth.13549
- Alonso MN, Mata-Forte T, García-León N, et al. Incidence, characteristics, laboratory findings and outcomes in acro-ischemia in COVID-19 patients. Vasc Health Risk Manag. 2020;16:467-478. doi:10.2147/VHRM.S276530
- Zhang L, Yan X, Fan Q, et al. D-dimer levels on admission to predict in-hospital mortality in patients with COVID-19. J Thromb Haemost. 2020;18:1324-1329. doi:10.1111/jth.14859
- Helms J, Tacquard C, Severac F, et al. High risk of thrombosis in patients with severe SARS-CoV-2 infection: a multicenter prospective cohort study. Intensive Care Med. 2020;46:1089-1098. doi:10.1007/s00134-020-06062-x
- Barton LM, Duval EJ, Stroberg E, et al. COVID-19 autopsies, Oklahoma, USA. Am J Clin Pathol. 2020;153:725-733. doi:10.1093/ajcp/aqaa062
- Wichmann D, Sperhake J-P, Lütgehetmann M, et al. Autopsy findings and venous thromboembolism in patients with COVID-19. Ann Intern Med. 2020;173:268-277. doi:10.7326/M20-2003
- Bastos ML, Tavaziva G, Abidi SK, et al. Diagnostic accuracy of serological tests for COVID-19: systematic review and meta-analysis. BMJ. 2020;370:m2516. doi:10.1136/bmj.m2516
- Galván Casas C, Català A, Carretero Hernández G, et al. Classification of the cutaneous manifestations of
COVID -19: a rapid prospective nationwide consensus study in Spain with 375 cases. Br J Dermatol. 2020;183:71-77. doi:10.1111/bjd.19163 - Fernandez-Nieto D, Jimenez-Cauhe J, Suarez-Valle A, et al. Characterization of acute acral skin lesions in nonhospitalized patients: a case series of 132 patients during the COVID-19 outbreak. J Am Acad Dermatol. 2020;83:E61-E63. doi:10.1016/j.jaad.2020.04.093
- Deutsch A, Blasiak R, Keyes A, et al. COVID toes: phenomenon or epiphenomenon? J Am Acad Dermatol. 2020;83:E347-E348. doi:10.1016/j.jaad.2020.07.037
There has been a rise in the prevalence of perniolike lesions—erythematous to violaceous, edematous papules or nodules on the fingers or toes—during the coronavirus disease 2019 (COVID-19) pandemic. These lesions are referred to as “COVID toes.” Although several studies have suggested an association with these lesions and COVID-19, and coronavirus particles have been identified in endothelial cells of biopsies of pernio lesions, questions remain on the management, pathophysiology, and implications of these lesions.1 We provide a practical review for primary care clinicians and dermatologists on the current management, recommendations, and remaining questions, with particular attention to the distinctions for children vs adults presenting with pernio lesions.
Hypothetical Case of a Child Presenting With Acral Lesions
A 7-year-old boy presents with acute-onset, violaceous, mildly painful and pruritic macules on the distal toes that began 3 days earlier and have progressed to involve more toes and appear more purpuric. A review of symptoms reveals no fever, cough, fatigue, or viral symptoms. He has been staying at home for the last few weeks with his brother, mother, and father. His father is working in delivery services and is social distancing at work but not at home. His mother is concerned about the lesions, if they could be COVID toes, and if testing is needed for the patient or family. In your assessment and management of this patient, you consider the following questions.
What Is the Relationship Between These Clinical Findings and COVID-19?
Despite negative polymerase chain reaction (PCR) tests reported in cases of chilblains during the COVID-19 pandemic as well as the possibility that these lesions are an indirect result of environmental factors or behavioral changes during quarantine, the majority of studies favor an association between these chilblains lesions and COVID-19 infection.2,3 Most compellingly, COVID-19 viral particles have been identified by immunohistochemistry and electron microscopy in the endothelial cells of biopsies of these lesions.1 Additionally, there is evidence for possible associations of other viruses, including Epstein-Barr virus and parvovirus B19, with chilblains lesions.4,5 In sum, with the lack of any large prospective study, the weight of current evidence suggests that these perniolike skin lesions are not specific markers of infection with severe acute respiratory syndrome coronavirus 2 (SARS-CoV-2).6
Published studies differ in reporting the coincidence of perniolike lesions with typical COVID-19 symptoms, including fever, dyspnea, cough, fatigue, myalgia, headache, and anosmia, among others. Some studies have reported that up to 63% of patients with reported perniolike lesions developed typical COVID-19 symptoms, but other studies found that no patients with these lesions developed symptoms.6-11 Studies with younger cohorts tended to report lower prevalence of COVID-19 symptoms, and within cohorts, younger patients tended to have less severe symptoms. For example, 78.8% of patients in a cohort (n=58) with an average age of 14 years did not experience COVID-19–related symptoms.6 Based on these data, it has been hypothesized that patients with chilblainslike lesions may represent a subpopulation who will have a robust interferon response that is protective from more symptomatic and severe COVID-19.12-14
Current evidence suggests that these lesions are most likely to occur between 9 days and 2 months after the onset of COVID-19 symptoms.4,9,10 Most cases have been only mildly symptomatic, with an overall favorable prognosis of both lesions and any viral symptoms.8,10 The lesions typically resolve without treatment within a few days of initial onset.15,16
What Should Be the Workup and Management of These Lesions?
Given the currently available information and favorable prognosis, usually no further workup specific to the perniolike lesions is required in the case of an asymptomatic child presenting with acral lesions, and the majority of management will center around patient and parent/guardian education and reassurance. When asked by the patient’s parent, “What does it mean that my child has these lesions?”, clinicians can provide information on the possible association with COVID-19 and the excellent, self-resolving prognosis. An example of honest and reasonable phrasing with current understanding might be, “We are currently not certain if COVID-19 causes these lesions, although there are data to suggest that they are associated. There are a lot of data showing that children with these lesions either do not have any symptoms or have very mild symptoms that resolve without treatment.”
For management, important considerations include how painful the lesions are to the individual patient and how they affect quality of life. If less severe, clinicians can reassure patients and parents/guardians that the lesions will likely self-resolve without treatment. If worsening or symptomatic, clinicians can try typical treatments for chilblains, such as topical steroids, whole-body warming, and nifedipine.17-19 Obtaining a review of symptoms, including COVID-19 symptoms and general viral symptoms, is important given the rare cases of children with severe COVID-19.20,21
The question of COVID-19 testing as related to these lesions remains controversial, and currently there are still differing perspectives on the need for biopsy, PCR for COVID-19, or serologies for COVID-19 in patients presenting with these lesions. Some experts report that additional testing is not needed in the pediatric population because of the high frequency of negative testing reported to date.22,23 However, these children may be silent carriers, and until more is known about their potential to transmit the virus, testing may be considered if resources allow, particularly if the patient has a known exposure.10,12,16,24 The ultimate decision to pursue biopsy or serologic workup for COVID-19 remains up to clinical discretion with consideration of symptoms, severity, and immunocompromised household contacts. If lesions developed after infection, PCR likely will result negative, whereas serologic testing may reveal antibodies.
Hypothetical Case of an Adult Presenting With Acral Lesions and COVID-19 Symptoms
A 50-year-old man presents with acute-onset, violaceous, painful, edematous plaques on the distal toes that began 3 days earlier and have progressed to include the soles. A review of symptoms reveals fever (temperature, 38.4 °C [101 °F]), cough, dyspnea, diarrhea, and severe asthenia. He has had interactions with a coworker who recently tested positive for COVID-19.
How Should You Consider These Lesions in the Context of the Other Symptoms Concerning for COVID-19?
In contrast to the asymptomatic child above, this adult has chilblainslike lesions and viral symptoms. In adults, chilblainslike lesions have been associated with relatively mild COVID-19, and patients with these lesions who are otherwise asymptomatic have largely tested negative for COVID-19 by PCR and serologic antibody testing.11,25,26
True acral ischemia, which is more severe and should be differentiated from chilblains, has been reported in critically ill patients.9 Additionally, studies have found that retiform purpura is the most common cutaneous finding in patients with severe COVID-19.27 For this patient, who has an examination consistent with progressive and severe chilblainslike lesions and suspicion for COVID-19 infection, it is important to observe and monitor these lesions, as clinical progression suggestive of acral ischemia or retiform purpura should be taken seriously and may indicate worsening of the underlying disease. Early intervention with anticoagulation might be considered, though there currently is no evidence of successful treatment.28
What Causes These Lesions in a Patient With COVID-19?
The underlying pathophysiology has been proposed to be a monocytic-macrophage–induced hyperinflammatory systemic state that damages the lungs, as well as the gastrointestinal, renal, and endothelial systems. The activation of the innate immune system triggers a cytokine storm that creates a hypercoagulable state that ultimately can manifest as superficial thromboses, leading to gangrene of the extremities. Additionally, interferon response and resulting hypercytokinemia may cause direct cytopathic damage to the endothelium of arterioles and capillaries, causing the development of papulovesicular lesions that resemble the chilblainslike lesions observed in children.29 In contrast to children, who typically have no or mild COVID-19 symptoms, adults may have a delayed interferon response, which has been proposed to allow for more severe manifestations of infection.12,30
How Should an Adult With Perniolike Lesions Be Managed?
Adults with chilblainslike lesions and no other signs or symptoms of COVID-19 infection do not necessarily need be tested for COVID-19, given the reports demonstrating most patients in this clinical situation will have negative PCRs and serologies for antibodies. However, there have been several reports of adults with acro-ischemic skin findings who also had severe COVID-19, with an observed incidence of 23% in intensive care unit patients with COVID-19.27,28,31,32 If there is suspicion of infection with COVID-19, it is advisable to first obtain workup for COVID-19 and other viruses that can cause acral lesions, including Epstein-Barr virus and parvovirus. Other pertinent laboratory tests may include D-dimer, fibrinogen, prothrombin time, activated partial thromboplastin time, antithrombin activity, platelet count, neutrophil count, procalcitonin, triglycerides, ferritin, C-reactive protein, and hemoglobin. For patients with evidence of worsening acro-ischemia, regular monitoring of these values up to several times per week can allow for initiation of vascular intervention, including angiontensin-converting enzyme inhibitors, statins, or antiplatelet drugs.32 The presence of antiphospholipid antibodies also has been associated with critically ill patients who develop digit ischemia as part of the sequelae of COVID-19 infection and therefore may act as an important marker for the potential to develop disseminated intravascular coagulation in this patient.33 Even if COVID-19 infection is not suspected, a thorough review of systems is important to look for an underlying connective tissue disease, such as systemic lupus erythematosus, which is associated with pernio. Associated symptoms may warrant workup with antinuclear antibodies and other appropriate autoimmune serologies.
If there is any doubt of the diagnosis, the patient is experiencing symptoms from the lesion, or the patient is experiencing other viral symptoms, it is appropriate to biopsy immediately to confirm the diagnosis. Prior studies have identified fibrin clots, angiocentric and eccrinotropic lymphocytic infiltrates, lymphocytic vasculopathy, and papillary dermal edema as the most common features in chilblainslike lesions during the COVID-19 pandemic.9
For COVID-19 testing, many studies have revealed adult patients with an acute hypercoagulable state testing positive by SARS-CoV-2 PCR. These same patients also experienced thromboembolic events shortly after testing positive for COVID-19, which suggests that patients with elevated D-dimer and fibrinogen likely will have a viral load that is sufficient to test positive for COVID-19.32,34-36 It is appropriate to test all patients with suspected COVID-19, especially adults who are more likely to experience adverse complications secondary to infection.
This patient experiencing COVID-19 symptoms with signs of acral ischemia is likely to test positive by PCR, and additional testing for serologic antibodies is unlikely to be clinically meaningful in this patient’s state. Furthermore, there is little evidence that serology is reliable because of the markedly high levels of both false-negative and false-positive results when using the available antibody testing kits.37 The latter evidence makes serology testing of little value for the general population, but particularly for patients with acute COVID-19.
Conclusion and Outstanding Questions
There is evidence suggesting an association between chilblainslike lesions and COVID-19.11,22,38,39 Children presenting with these lesions have an excellent prognosis and only need a workup or treatment if there are other symptoms, as the lesions self-resolve in the majority of reported cases.7-9 Adults presenting with these lesions and without symptoms likewise are unlikely to test positive for COVID-19, and the lesions typically resolve spontaneously or with first-line treatment. However, adults presenting with these lesions and COVID-19 symptoms should raise clinical concern for evolving skin manifestations of acro-ischemia. If the diagnosis is uncertain or systemic symptoms are concerning, biopsy, COVID-19 PCR, and other appropriate laboratory workup should be obtained.
There remains controversy and uncertainty over the relationship between these skin findings and SARS-CoV-2 infection, with clinical evidence to support both a direct relationship representing convalescent-phase cutaneous reaction as well as an indirect epiphenomenon. If there was a direct relationship, we would have expected to see a rise in the incidence of acral lesions proportionate to the rising caseload of COVID-19 after the reopening of many states in the summer of 2020. Similarly, because young adults represent the largest demographic of increasing cases and as some schools have remained open for in-person instruction during the current academic year, we also would have expected the incidence of chilblains-like lesions presenting to dermatologists and pediatricians to increase alongside these cases. Continued evaluation of emerging literature and ongoing efforts to understand the cause of this observed phenomenon will hopefully help us arrive at a future understanding of the pathophysiology of this puzzling skin manifestation.40
There has been a rise in the prevalence of perniolike lesions—erythematous to violaceous, edematous papules or nodules on the fingers or toes—during the coronavirus disease 2019 (COVID-19) pandemic. These lesions are referred to as “COVID toes.” Although several studies have suggested an association with these lesions and COVID-19, and coronavirus particles have been identified in endothelial cells of biopsies of pernio lesions, questions remain on the management, pathophysiology, and implications of these lesions.1 We provide a practical review for primary care clinicians and dermatologists on the current management, recommendations, and remaining questions, with particular attention to the distinctions for children vs adults presenting with pernio lesions.
Hypothetical Case of a Child Presenting With Acral Lesions
A 7-year-old boy presents with acute-onset, violaceous, mildly painful and pruritic macules on the distal toes that began 3 days earlier and have progressed to involve more toes and appear more purpuric. A review of symptoms reveals no fever, cough, fatigue, or viral symptoms. He has been staying at home for the last few weeks with his brother, mother, and father. His father is working in delivery services and is social distancing at work but not at home. His mother is concerned about the lesions, if they could be COVID toes, and if testing is needed for the patient or family. In your assessment and management of this patient, you consider the following questions.
What Is the Relationship Between These Clinical Findings and COVID-19?
Despite negative polymerase chain reaction (PCR) tests reported in cases of chilblains during the COVID-19 pandemic as well as the possibility that these lesions are an indirect result of environmental factors or behavioral changes during quarantine, the majority of studies favor an association between these chilblains lesions and COVID-19 infection.2,3 Most compellingly, COVID-19 viral particles have been identified by immunohistochemistry and electron microscopy in the endothelial cells of biopsies of these lesions.1 Additionally, there is evidence for possible associations of other viruses, including Epstein-Barr virus and parvovirus B19, with chilblains lesions.4,5 In sum, with the lack of any large prospective study, the weight of current evidence suggests that these perniolike skin lesions are not specific markers of infection with severe acute respiratory syndrome coronavirus 2 (SARS-CoV-2).6
Published studies differ in reporting the coincidence of perniolike lesions with typical COVID-19 symptoms, including fever, dyspnea, cough, fatigue, myalgia, headache, and anosmia, among others. Some studies have reported that up to 63% of patients with reported perniolike lesions developed typical COVID-19 symptoms, but other studies found that no patients with these lesions developed symptoms.6-11 Studies with younger cohorts tended to report lower prevalence of COVID-19 symptoms, and within cohorts, younger patients tended to have less severe symptoms. For example, 78.8% of patients in a cohort (n=58) with an average age of 14 years did not experience COVID-19–related symptoms.6 Based on these data, it has been hypothesized that patients with chilblainslike lesions may represent a subpopulation who will have a robust interferon response that is protective from more symptomatic and severe COVID-19.12-14
Current evidence suggests that these lesions are most likely to occur between 9 days and 2 months after the onset of COVID-19 symptoms.4,9,10 Most cases have been only mildly symptomatic, with an overall favorable prognosis of both lesions and any viral symptoms.8,10 The lesions typically resolve without treatment within a few days of initial onset.15,16
What Should Be the Workup and Management of These Lesions?
Given the currently available information and favorable prognosis, usually no further workup specific to the perniolike lesions is required in the case of an asymptomatic child presenting with acral lesions, and the majority of management will center around patient and parent/guardian education and reassurance. When asked by the patient’s parent, “What does it mean that my child has these lesions?”, clinicians can provide information on the possible association with COVID-19 and the excellent, self-resolving prognosis. An example of honest and reasonable phrasing with current understanding might be, “We are currently not certain if COVID-19 causes these lesions, although there are data to suggest that they are associated. There are a lot of data showing that children with these lesions either do not have any symptoms or have very mild symptoms that resolve without treatment.”
For management, important considerations include how painful the lesions are to the individual patient and how they affect quality of life. If less severe, clinicians can reassure patients and parents/guardians that the lesions will likely self-resolve without treatment. If worsening or symptomatic, clinicians can try typical treatments for chilblains, such as topical steroids, whole-body warming, and nifedipine.17-19 Obtaining a review of symptoms, including COVID-19 symptoms and general viral symptoms, is important given the rare cases of children with severe COVID-19.20,21
The question of COVID-19 testing as related to these lesions remains controversial, and currently there are still differing perspectives on the need for biopsy, PCR for COVID-19, or serologies for COVID-19 in patients presenting with these lesions. Some experts report that additional testing is not needed in the pediatric population because of the high frequency of negative testing reported to date.22,23 However, these children may be silent carriers, and until more is known about their potential to transmit the virus, testing may be considered if resources allow, particularly if the patient has a known exposure.10,12,16,24 The ultimate decision to pursue biopsy or serologic workup for COVID-19 remains up to clinical discretion with consideration of symptoms, severity, and immunocompromised household contacts. If lesions developed after infection, PCR likely will result negative, whereas serologic testing may reveal antibodies.
Hypothetical Case of an Adult Presenting With Acral Lesions and COVID-19 Symptoms
A 50-year-old man presents with acute-onset, violaceous, painful, edematous plaques on the distal toes that began 3 days earlier and have progressed to include the soles. A review of symptoms reveals fever (temperature, 38.4 °C [101 °F]), cough, dyspnea, diarrhea, and severe asthenia. He has had interactions with a coworker who recently tested positive for COVID-19.
How Should You Consider These Lesions in the Context of the Other Symptoms Concerning for COVID-19?
In contrast to the asymptomatic child above, this adult has chilblainslike lesions and viral symptoms. In adults, chilblainslike lesions have been associated with relatively mild COVID-19, and patients with these lesions who are otherwise asymptomatic have largely tested negative for COVID-19 by PCR and serologic antibody testing.11,25,26
True acral ischemia, which is more severe and should be differentiated from chilblains, has been reported in critically ill patients.9 Additionally, studies have found that retiform purpura is the most common cutaneous finding in patients with severe COVID-19.27 For this patient, who has an examination consistent with progressive and severe chilblainslike lesions and suspicion for COVID-19 infection, it is important to observe and monitor these lesions, as clinical progression suggestive of acral ischemia or retiform purpura should be taken seriously and may indicate worsening of the underlying disease. Early intervention with anticoagulation might be considered, though there currently is no evidence of successful treatment.28
What Causes These Lesions in a Patient With COVID-19?
The underlying pathophysiology has been proposed to be a monocytic-macrophage–induced hyperinflammatory systemic state that damages the lungs, as well as the gastrointestinal, renal, and endothelial systems. The activation of the innate immune system triggers a cytokine storm that creates a hypercoagulable state that ultimately can manifest as superficial thromboses, leading to gangrene of the extremities. Additionally, interferon response and resulting hypercytokinemia may cause direct cytopathic damage to the endothelium of arterioles and capillaries, causing the development of papulovesicular lesions that resemble the chilblainslike lesions observed in children.29 In contrast to children, who typically have no or mild COVID-19 symptoms, adults may have a delayed interferon response, which has been proposed to allow for more severe manifestations of infection.12,30
How Should an Adult With Perniolike Lesions Be Managed?
Adults with chilblainslike lesions and no other signs or symptoms of COVID-19 infection do not necessarily need be tested for COVID-19, given the reports demonstrating most patients in this clinical situation will have negative PCRs and serologies for antibodies. However, there have been several reports of adults with acro-ischemic skin findings who also had severe COVID-19, with an observed incidence of 23% in intensive care unit patients with COVID-19.27,28,31,32 If there is suspicion of infection with COVID-19, it is advisable to first obtain workup for COVID-19 and other viruses that can cause acral lesions, including Epstein-Barr virus and parvovirus. Other pertinent laboratory tests may include D-dimer, fibrinogen, prothrombin time, activated partial thromboplastin time, antithrombin activity, platelet count, neutrophil count, procalcitonin, triglycerides, ferritin, C-reactive protein, and hemoglobin. For patients with evidence of worsening acro-ischemia, regular monitoring of these values up to several times per week can allow for initiation of vascular intervention, including angiontensin-converting enzyme inhibitors, statins, or antiplatelet drugs.32 The presence of antiphospholipid antibodies also has been associated with critically ill patients who develop digit ischemia as part of the sequelae of COVID-19 infection and therefore may act as an important marker for the potential to develop disseminated intravascular coagulation in this patient.33 Even if COVID-19 infection is not suspected, a thorough review of systems is important to look for an underlying connective tissue disease, such as systemic lupus erythematosus, which is associated with pernio. Associated symptoms may warrant workup with antinuclear antibodies and other appropriate autoimmune serologies.
If there is any doubt of the diagnosis, the patient is experiencing symptoms from the lesion, or the patient is experiencing other viral symptoms, it is appropriate to biopsy immediately to confirm the diagnosis. Prior studies have identified fibrin clots, angiocentric and eccrinotropic lymphocytic infiltrates, lymphocytic vasculopathy, and papillary dermal edema as the most common features in chilblainslike lesions during the COVID-19 pandemic.9
For COVID-19 testing, many studies have revealed adult patients with an acute hypercoagulable state testing positive by SARS-CoV-2 PCR. These same patients also experienced thromboembolic events shortly after testing positive for COVID-19, which suggests that patients with elevated D-dimer and fibrinogen likely will have a viral load that is sufficient to test positive for COVID-19.32,34-36 It is appropriate to test all patients with suspected COVID-19, especially adults who are more likely to experience adverse complications secondary to infection.
This patient experiencing COVID-19 symptoms with signs of acral ischemia is likely to test positive by PCR, and additional testing for serologic antibodies is unlikely to be clinically meaningful in this patient’s state. Furthermore, there is little evidence that serology is reliable because of the markedly high levels of both false-negative and false-positive results when using the available antibody testing kits.37 The latter evidence makes serology testing of little value for the general population, but particularly for patients with acute COVID-19.
Conclusion and Outstanding Questions
There is evidence suggesting an association between chilblainslike lesions and COVID-19.11,22,38,39 Children presenting with these lesions have an excellent prognosis and only need a workup or treatment if there are other symptoms, as the lesions self-resolve in the majority of reported cases.7-9 Adults presenting with these lesions and without symptoms likewise are unlikely to test positive for COVID-19, and the lesions typically resolve spontaneously or with first-line treatment. However, adults presenting with these lesions and COVID-19 symptoms should raise clinical concern for evolving skin manifestations of acro-ischemia. If the diagnosis is uncertain or systemic symptoms are concerning, biopsy, COVID-19 PCR, and other appropriate laboratory workup should be obtained.
There remains controversy and uncertainty over the relationship between these skin findings and SARS-CoV-2 infection, with clinical evidence to support both a direct relationship representing convalescent-phase cutaneous reaction as well as an indirect epiphenomenon. If there was a direct relationship, we would have expected to see a rise in the incidence of acral lesions proportionate to the rising caseload of COVID-19 after the reopening of many states in the summer of 2020. Similarly, because young adults represent the largest demographic of increasing cases and as some schools have remained open for in-person instruction during the current academic year, we also would have expected the incidence of chilblains-like lesions presenting to dermatologists and pediatricians to increase alongside these cases. Continued evaluation of emerging literature and ongoing efforts to understand the cause of this observed phenomenon will hopefully help us arrive at a future understanding of the pathophysiology of this puzzling skin manifestation.40
- Colmenero I, Santonja C, Alonso-Riaño M, et al. SARS-CoV-2 endothelial infection causes COVID-19 chilblains: histopathological, immunohistochemical and ultrastructural study of seven paediatric cases. Br J Dermatol. 2020;183:729-737. doi:10.1111/bjd.19327
- Neri I, Virdi A, Corsini I, et al. Major cluster of paediatric “true” primary chilblains during the COVID-19 pandemic: a consequence of lifestyle changes due to lockdown. J Eur Acad Dermatol Venereol. 2020;34:2630-2635. doi:10.1111/jdv.16751
- Hubiche T, Le Duff F, Chiaverini C, et al. Negative SARS-CoV-2 PCR in patients with chilblain-like lesions [letter]. Lancet Infect Dis. June 18, 2020. doi:10.1016/S1473-3099(20)30518-1
- Pistorius MA, Blaise S, Le Hello C, et al. Chilblains and COVID19 infection: causality or coincidence? How to proceed? J Med Vasc. 2020;45:221-223. doi:10.1016/j.jdmv.2020.05.002
- Massey PR, Jones KM. Going viral: a brief history of Chilblain-like skin lesions (“COVID toes”) amidst the COVID-19 pandemic. Semin Oncol. 2020;47:330-334. doi:10.1053/j.seminoncol.2020.05.012
- Docampo-Simón A, Sánchez-Pujol MJ, Juan-Carpena G, et al. Are chilblain-like acral skin lesions really indicative of COVID-19? A prospective study and literature review [letter]. J Eur Acad Dermatol Venereol. 2020;34:e445-e446. doi:10.1111/jdv.16665
- El Hachem M, Diociaiuti A, Concato C, et al. A clinical, histopathological and laboratory study of 19 consecutive Italian paediatric patients with chilblain-like lesions: lights and shadows on the relationship with COVID-19 infection. J Eur Acad Dermatol Venereol. 2020;34:2620-2629. doi:10.1111/jdv.16682
- Recalcati S, Barbagallo T, Frasin LA, et al. Acral cutaneous lesions in the time of COVID-19. J Eur Acad Dermatol Venereol. 2020;34:e346-e347. doi:10.1111/jdv.16533
- Andina D, Noguera-Morel L, Bascuas-Arribas M, et al. Chilblains in children in the setting of COVID-19 pandemic. Pediatr Dermatol. 2020;37:406-411. doi:10.1111/pde.14215
- Casas CG, Català A, Hernández GC, et al. Classification of the cutaneous manifestations of COVID-19: a rapid prospective nationwide consensus study in Spain with 375 cases. Br J Dermatol. 2020;183:71-77. doi:10.1111/bjd.19163
- Freeman EE, McMahon DE, Lipoff JB, et al. Pernio-like skin lesions associated with COVID-19: a case series of 318 patients from 8 countries. J Am Acad Dermatol. 2020;83:486-492. doi:10.1016/j.jaad.2020.05.109
- Kolivras A, Dehavay F, Delplace D, et al. Coronavirus (COVID-19) infection–induced chilblains: a case report with histopathologic findings. JAAD Case Rep. 2020;6:489-492. doi:10.1016/j.jdcr.2020.04.011
- Damsky W, Peterson D, King B. When interferon tiptoes through COVID-19: pernio-like lesions and their prognostic implications during SARS-CoV-2 infection. J Am Acad Dermatol. 2020;83:E269-E270. doi:10.1016/j.jaad.2020.06.052
- Lipsker D. A chilblain epidemic during the COVID-19 pandemic. A sign of natural resistance to SARS-CoV-2? Med Hypotheses. 2020;144:109959. doi:10.1016/j.mehy.2020.109959
- Kaya G, Kaya A, Saurat J-H. Clinical and histopathological features and potential pathological mechanisms of skin lesions in COVID-19: review of the literature. Dermatopathology. 2020;7:3-16. doi:10.3390/dermatopathology7010002
- Pavone P, Marino S, Marino L, et al. Chilblains-like lesions and SARS-CoV-2 in children: An overview in therapeutic approach. Dermatol Ther. 2021;34:E14502. doi:https://doi.org/10.1111/dth.14502
- Dowd PM, Rustin MH, Lanigan S. Nifedipine in the treatment of chilblains. Br Med J (Clin Res Ed). 1986;293:923-924. doi:10.1136/bmj.293.6552.923-a
- Rustin MH, Newton JA, Smith NP, et al. The treatment of chilblains with nifedipine: the results of a pilot study, a double-blind placebo-controlled randomized study and a long-term open trial. Br J Dermatol. 1989;120:267-275. doi:10.1111/j.1365-2133.1989.tb07792.x
- Almahameed A, Pinto DS. Pernio (chilblains). Curr Treat Options Cardiovasc Med. 2008;10:128-135. doi:10.1007/s11936-008-0014-0
- Chen F, Liu ZS, Zhang FR, et al. First case of severe childhood novel coronavirus pneumonia in China [in Chinese]. Zhonghua Er Ke Za Zhi. 2020;58:179-182. doi:10.3760/cma.j.issn.0578-1310.2020.03.003
- Choi S-H, Kim HW, Kang J-M, et al. Epidemiology and clinical features of coronavirus disease 2019 in children. Clin Exp Pediatr. 2020;63:125-132. doi:10.3345/cep.2020.00535
- Piccolo V, Neri I, Manunza F, et al. Chilblain-like lesions during the COVID-19 pandemic: should we really worry? Int J Dermatol. 2020;59:1026-1027. doi:10.1111/ijd.1499
- Roca-Ginés J, Torres-Navarro I, Sánchez-Arráez J, et al. Assessment of acute acral lesions in a case series of children and adolescents during the COVID-19 pandemic. JAMA Dermatol. 2020;156:992-997. doi:10.1001/jamadermatol.2020.2340
- Landa N, Mendieta-Eckert M, Fonda-Pascual P, et al. Chilblain-like lesions on feet and hands during the COVID-19 pandemic. Int J Dermatol. 2020;59:739-743. doi:10.1111/ijd.14937
- Herman A, Peeters C, Verroken A, et al. Evaluation of chilblains as a manifestation of the COVID-19 pandemic. JAMA Dermatol. 2020;156:998-1003. doi:10.1001/jamadermatol.2020.2368
- Daneshjou R, Rana J, Dickman M, et al. Pernio-like eruption associated with COVID-19 in skin of color. JAAD Case Rep. 2020;6:892-897. doi:10.1016/j.jdcr.2020.07.009
- Freeman EE, McMahon DE, Lipoff JB, et al. The spectrum of COVID-19-associated dermatologic manifestations: an international registry of 716 patients from 31 countries. J Am Acad Dermatol. 2020;83:1118-1129. doi:10.1016/j.jaad.2020.06.1016
- Zhang Y, Cao W, Xiao M, et al. Clinical and coagulation characteristics of 7 patients with critical COVID-2019 pneumonia and acro-ischemia [in Chinese]. Zhonghua Xue Ye Xue Za Zhi. 2020;41:E006. doi:10.3760/cma.j.issn.0253-2727.2020.0006
- Criado PR, Abdalla BMZ, de Assis IC, et al. Are the cutaneous manifestations during or due to SARS-CoV-2 infection/COVID-19 frequent or not? revision of possible pathophysiologic mechanisms. Inflamm Res. 2020;69:745-756. doi:10.1007/s00011-020-01370-w
- Park A, Iwasaki A. Type I and type III interferons—induction, signaling, evasion, and application to combat COVID-19. Cell Host Microbe. 2020;27:870-878. doi:10.1016/j.chom.2020.05.008
- Wollina U, Karadag˘ AS, Rowland-Payne C, et al. Cutaneous signs in COVID-19 patients: a review. Dermatol Ther. 2020;33:E13549. doi:10.1111/dth.13549
- Alonso MN, Mata-Forte T, García-León N, et al. Incidence, characteristics, laboratory findings and outcomes in acro-ischemia in COVID-19 patients. Vasc Health Risk Manag. 2020;16:467-478. doi:10.2147/VHRM.S276530
- Zhang L, Yan X, Fan Q, et al. D-dimer levels on admission to predict in-hospital mortality in patients with COVID-19. J Thromb Haemost. 2020;18:1324-1329. doi:10.1111/jth.14859
- Helms J, Tacquard C, Severac F, et al. High risk of thrombosis in patients with severe SARS-CoV-2 infection: a multicenter prospective cohort study. Intensive Care Med. 2020;46:1089-1098. doi:10.1007/s00134-020-06062-x
- Barton LM, Duval EJ, Stroberg E, et al. COVID-19 autopsies, Oklahoma, USA. Am J Clin Pathol. 2020;153:725-733. doi:10.1093/ajcp/aqaa062
- Wichmann D, Sperhake J-P, Lütgehetmann M, et al. Autopsy findings and venous thromboembolism in patients with COVID-19. Ann Intern Med. 2020;173:268-277. doi:10.7326/M20-2003
- Bastos ML, Tavaziva G, Abidi SK, et al. Diagnostic accuracy of serological tests for COVID-19: systematic review and meta-analysis. BMJ. 2020;370:m2516. doi:10.1136/bmj.m2516
- Galván Casas C, Català A, Carretero Hernández G, et al. Classification of the cutaneous manifestations of
COVID -19: a rapid prospective nationwide consensus study in Spain with 375 cases. Br J Dermatol. 2020;183:71-77. doi:10.1111/bjd.19163 - Fernandez-Nieto D, Jimenez-Cauhe J, Suarez-Valle A, et al. Characterization of acute acral skin lesions in nonhospitalized patients: a case series of 132 patients during the COVID-19 outbreak. J Am Acad Dermatol. 2020;83:E61-E63. doi:10.1016/j.jaad.2020.04.093
- Deutsch A, Blasiak R, Keyes A, et al. COVID toes: phenomenon or epiphenomenon? J Am Acad Dermatol. 2020;83:E347-E348. doi:10.1016/j.jaad.2020.07.037
- Colmenero I, Santonja C, Alonso-Riaño M, et al. SARS-CoV-2 endothelial infection causes COVID-19 chilblains: histopathological, immunohistochemical and ultrastructural study of seven paediatric cases. Br J Dermatol. 2020;183:729-737. doi:10.1111/bjd.19327
- Neri I, Virdi A, Corsini I, et al. Major cluster of paediatric “true” primary chilblains during the COVID-19 pandemic: a consequence of lifestyle changes due to lockdown. J Eur Acad Dermatol Venereol. 2020;34:2630-2635. doi:10.1111/jdv.16751
- Hubiche T, Le Duff F, Chiaverini C, et al. Negative SARS-CoV-2 PCR in patients with chilblain-like lesions [letter]. Lancet Infect Dis. June 18, 2020. doi:10.1016/S1473-3099(20)30518-1
- Pistorius MA, Blaise S, Le Hello C, et al. Chilblains and COVID19 infection: causality or coincidence? How to proceed? J Med Vasc. 2020;45:221-223. doi:10.1016/j.jdmv.2020.05.002
- Massey PR, Jones KM. Going viral: a brief history of Chilblain-like skin lesions (“COVID toes”) amidst the COVID-19 pandemic. Semin Oncol. 2020;47:330-334. doi:10.1053/j.seminoncol.2020.05.012
- Docampo-Simón A, Sánchez-Pujol MJ, Juan-Carpena G, et al. Are chilblain-like acral skin lesions really indicative of COVID-19? A prospective study and literature review [letter]. J Eur Acad Dermatol Venereol. 2020;34:e445-e446. doi:10.1111/jdv.16665
- El Hachem M, Diociaiuti A, Concato C, et al. A clinical, histopathological and laboratory study of 19 consecutive Italian paediatric patients with chilblain-like lesions: lights and shadows on the relationship with COVID-19 infection. J Eur Acad Dermatol Venereol. 2020;34:2620-2629. doi:10.1111/jdv.16682
- Recalcati S, Barbagallo T, Frasin LA, et al. Acral cutaneous lesions in the time of COVID-19. J Eur Acad Dermatol Venereol. 2020;34:e346-e347. doi:10.1111/jdv.16533
- Andina D, Noguera-Morel L, Bascuas-Arribas M, et al. Chilblains in children in the setting of COVID-19 pandemic. Pediatr Dermatol. 2020;37:406-411. doi:10.1111/pde.14215
- Casas CG, Català A, Hernández GC, et al. Classification of the cutaneous manifestations of COVID-19: a rapid prospective nationwide consensus study in Spain with 375 cases. Br J Dermatol. 2020;183:71-77. doi:10.1111/bjd.19163
- Freeman EE, McMahon DE, Lipoff JB, et al. Pernio-like skin lesions associated with COVID-19: a case series of 318 patients from 8 countries. J Am Acad Dermatol. 2020;83:486-492. doi:10.1016/j.jaad.2020.05.109
- Kolivras A, Dehavay F, Delplace D, et al. Coronavirus (COVID-19) infection–induced chilblains: a case report with histopathologic findings. JAAD Case Rep. 2020;6:489-492. doi:10.1016/j.jdcr.2020.04.011
- Damsky W, Peterson D, King B. When interferon tiptoes through COVID-19: pernio-like lesions and their prognostic implications during SARS-CoV-2 infection. J Am Acad Dermatol. 2020;83:E269-E270. doi:10.1016/j.jaad.2020.06.052
- Lipsker D. A chilblain epidemic during the COVID-19 pandemic. A sign of natural resistance to SARS-CoV-2? Med Hypotheses. 2020;144:109959. doi:10.1016/j.mehy.2020.109959
- Kaya G, Kaya A, Saurat J-H. Clinical and histopathological features and potential pathological mechanisms of skin lesions in COVID-19: review of the literature. Dermatopathology. 2020;7:3-16. doi:10.3390/dermatopathology7010002
- Pavone P, Marino S, Marino L, et al. Chilblains-like lesions and SARS-CoV-2 in children: An overview in therapeutic approach. Dermatol Ther. 2021;34:E14502. doi:https://doi.org/10.1111/dth.14502
- Dowd PM, Rustin MH, Lanigan S. Nifedipine in the treatment of chilblains. Br Med J (Clin Res Ed). 1986;293:923-924. doi:10.1136/bmj.293.6552.923-a
- Rustin MH, Newton JA, Smith NP, et al. The treatment of chilblains with nifedipine: the results of a pilot study, a double-blind placebo-controlled randomized study and a long-term open trial. Br J Dermatol. 1989;120:267-275. doi:10.1111/j.1365-2133.1989.tb07792.x
- Almahameed A, Pinto DS. Pernio (chilblains). Curr Treat Options Cardiovasc Med. 2008;10:128-135. doi:10.1007/s11936-008-0014-0
- Chen F, Liu ZS, Zhang FR, et al. First case of severe childhood novel coronavirus pneumonia in China [in Chinese]. Zhonghua Er Ke Za Zhi. 2020;58:179-182. doi:10.3760/cma.j.issn.0578-1310.2020.03.003
- Choi S-H, Kim HW, Kang J-M, et al. Epidemiology and clinical features of coronavirus disease 2019 in children. Clin Exp Pediatr. 2020;63:125-132. doi:10.3345/cep.2020.00535
- Piccolo V, Neri I, Manunza F, et al. Chilblain-like lesions during the COVID-19 pandemic: should we really worry? Int J Dermatol. 2020;59:1026-1027. doi:10.1111/ijd.1499
- Roca-Ginés J, Torres-Navarro I, Sánchez-Arráez J, et al. Assessment of acute acral lesions in a case series of children and adolescents during the COVID-19 pandemic. JAMA Dermatol. 2020;156:992-997. doi:10.1001/jamadermatol.2020.2340
- Landa N, Mendieta-Eckert M, Fonda-Pascual P, et al. Chilblain-like lesions on feet and hands during the COVID-19 pandemic. Int J Dermatol. 2020;59:739-743. doi:10.1111/ijd.14937
- Herman A, Peeters C, Verroken A, et al. Evaluation of chilblains as a manifestation of the COVID-19 pandemic. JAMA Dermatol. 2020;156:998-1003. doi:10.1001/jamadermatol.2020.2368
- Daneshjou R, Rana J, Dickman M, et al. Pernio-like eruption associated with COVID-19 in skin of color. JAAD Case Rep. 2020;6:892-897. doi:10.1016/j.jdcr.2020.07.009
- Freeman EE, McMahon DE, Lipoff JB, et al. The spectrum of COVID-19-associated dermatologic manifestations: an international registry of 716 patients from 31 countries. J Am Acad Dermatol. 2020;83:1118-1129. doi:10.1016/j.jaad.2020.06.1016
- Zhang Y, Cao W, Xiao M, et al. Clinical and coagulation characteristics of 7 patients with critical COVID-2019 pneumonia and acro-ischemia [in Chinese]. Zhonghua Xue Ye Xue Za Zhi. 2020;41:E006. doi:10.3760/cma.j.issn.0253-2727.2020.0006
- Criado PR, Abdalla BMZ, de Assis IC, et al. Are the cutaneous manifestations during or due to SARS-CoV-2 infection/COVID-19 frequent or not? revision of possible pathophysiologic mechanisms. Inflamm Res. 2020;69:745-756. doi:10.1007/s00011-020-01370-w
- Park A, Iwasaki A. Type I and type III interferons—induction, signaling, evasion, and application to combat COVID-19. Cell Host Microbe. 2020;27:870-878. doi:10.1016/j.chom.2020.05.008
- Wollina U, Karadag˘ AS, Rowland-Payne C, et al. Cutaneous signs in COVID-19 patients: a review. Dermatol Ther. 2020;33:E13549. doi:10.1111/dth.13549
- Alonso MN, Mata-Forte T, García-León N, et al. Incidence, characteristics, laboratory findings and outcomes in acro-ischemia in COVID-19 patients. Vasc Health Risk Manag. 2020;16:467-478. doi:10.2147/VHRM.S276530
- Zhang L, Yan X, Fan Q, et al. D-dimer levels on admission to predict in-hospital mortality in patients with COVID-19. J Thromb Haemost. 2020;18:1324-1329. doi:10.1111/jth.14859
- Helms J, Tacquard C, Severac F, et al. High risk of thrombosis in patients with severe SARS-CoV-2 infection: a multicenter prospective cohort study. Intensive Care Med. 2020;46:1089-1098. doi:10.1007/s00134-020-06062-x
- Barton LM, Duval EJ, Stroberg E, et al. COVID-19 autopsies, Oklahoma, USA. Am J Clin Pathol. 2020;153:725-733. doi:10.1093/ajcp/aqaa062
- Wichmann D, Sperhake J-P, Lütgehetmann M, et al. Autopsy findings and venous thromboembolism in patients with COVID-19. Ann Intern Med. 2020;173:268-277. doi:10.7326/M20-2003
- Bastos ML, Tavaziva G, Abidi SK, et al. Diagnostic accuracy of serological tests for COVID-19: systematic review and meta-analysis. BMJ. 2020;370:m2516. doi:10.1136/bmj.m2516
- Galván Casas C, Català A, Carretero Hernández G, et al. Classification of the cutaneous manifestations of
COVID -19: a rapid prospective nationwide consensus study in Spain with 375 cases. Br J Dermatol. 2020;183:71-77. doi:10.1111/bjd.19163 - Fernandez-Nieto D, Jimenez-Cauhe J, Suarez-Valle A, et al. Characterization of acute acral skin lesions in nonhospitalized patients: a case series of 132 patients during the COVID-19 outbreak. J Am Acad Dermatol. 2020;83:E61-E63. doi:10.1016/j.jaad.2020.04.093
- Deutsch A, Blasiak R, Keyes A, et al. COVID toes: phenomenon or epiphenomenon? J Am Acad Dermatol. 2020;83:E347-E348. doi:10.1016/j.jaad.2020.07.037
Practice Points
- Children with chilblainslike lesions generally have a favorable prognosis. As lesions self-resolve, treatment should focus on symptom management and education.
- In children with chilblainslike lesions and no systemic symptoms, further workup for coronavirus disease 2019 (COVID-19) is not necessary for the care of the individual patient.
- In adults with acral lesions, it is important to distinguish between chilblainslike lesions, true acral ischemia, and retiform purpura. Chilblainslike lesions have been associated with mild COVID-19 disease, whereas acral ischemia and retiform purpura have been associated with severe and fatal disease.
- Biopsy and COVID-19 testing should be obtained in adults if there is diagnostic uncertainty or if there are worsening symptoms.
Comparison of Shave and Punch Biopsy Utilization Among Dermatology Practices
In 2019, the 2 Current Procedural Terminology (CPT) codes for skin biopsies (11100 and 11101) were replaced with 6 new CPT codes that specify biopsy technique and associated procedural complexity. 1,2 Prior to the coding changes, all biopsies were reimbursed at the same payment level, but a punch biopsy (11104; national nonfacility Medicare payment, $133.29) is now reimbursed more than a shave biopsy (11102; national nonfacility Medicare payment, $106.42). 3 We sought to evaluate whether the decrease in reimbursement for shave biopsies and concurrent increase in reimbursement for punch biopsies led to a shift from shave to punch biopsy utilization.
Methods
We examined shave and punch biopsies submitted for pathologic examination at Brigham and Women’s Hospital, Massachusetts General Hospital, and Massachusetts General Physician’s Organization (all in Boston, Massachusetts), and Penn Medicine, University of Pennsylvania Health System (Philadelphia, Pennsylvania), in May 2018 vs May 2019 (four months after new codes were implemented). This study was approved by Partners HealthCare (Boston, Massachusetts) and the University of Pennsylvania institutional review boards.
We included shave and punch biopsies of skin performed by physician dermatologists and mid-level providers (ie, physician assistants, nurse practitioners) at dermatology practices. All biopsies performed by a technique other than shave or punch, unspecified biopsy type, consultation cases, nonskin biopsies (eg, mucosa), and biopsies performed at nondermatology practices were excluded. We also excluded biopsies by providers who were not present during both study periods to account for provider mix.
Statistical Analysis
To evaluate for changes in the ratio of shave to punch biopsy utilization over time, we performed χ2 tests. Because care practices may differ between private and academic settings as well as between physicians and mid-level providers, we performed subgroup analyses by practice setting and provider type.4
Results
We identified 11,785 biopsies (12.11% of which were punch) submitted for pathologic examination in May 2018 compared to 11,291 biopsies (12.08% of which were punch) in May 2019 (Table). The overall use of punch biopsies relative to shave biopsies did not change between the years. There was a relative decrease in punch biopsy use among academic practices (−1.88%; P=.032) and a relative increase in punch biopsy use among private practices (+0.90%; P=.043). Provider type was not associated with differing utilization of biopsy type.
Comment
Overall, there was not a considerable shift in utilization behavior from shave to punch biopsies after the introduction of new coding changes. However, our study does demonstrate a small yet significant increase in punch biopsy utilization among private practices, and a decrease among academic practices. Although the change in biopsy utilization behavior is small in magnitude, it may have a substantial impact when extrapolated to behavior across the entire United States.
We were unable to assess additional factors, such as clinical diagnosis, body site, and cosmetic concerns, that may impact biopsy type selection in this study. Although we included multiple study sites to improve generalizability, our findings may not be representative of all biopsies performed in the dermatology setting. The baseline difference in relative punch biopsy use in academic vs private practices may reflect differences in patient populations and chief concerns, but assuming these features are stable over a 1-year time period, our findings should remain valid. Future studies should focus on qualitative evaluations of physician decision-making and evaluate whether similar trends persist over time.
Conclusion
Skin biopsy utilization trends among differing practice and provider types should continue to be monitored to assess for longitudinal trends in utilization within the context of updated billing codes and associated reimbursements.
- Grider D. 2019 CPT® coding for skin biopsies. ICD10 monitor website. September 17, 2018. Updated January 7, 2019. Accessed February 17, 2021. https://www.icd10monitor.com/2019-cpt-coding-for-skin-biopsies 2.
- Tongdee E, Siegel DM, Markowitz O. New diagnostic procedure codes and reimbursement. Cutis. 2019;103:208-211.
- Search the physician fee schedule. Centers for Medicare & Medicaid Services website. Updated January 20, 2021. Accessed February 17, 2021. https://www.cms.gov/medicare/physician-fee-schedule/search
- Zhang M, Zippin J, Kaffenberger B. Trends and scope of dermatology procedures billed by advanced practice professionals from 2012 through 2015. JAMA Dermatol. 2018;154:1040-1044.
In 2019, the 2 Current Procedural Terminology (CPT) codes for skin biopsies (11100 and 11101) were replaced with 6 new CPT codes that specify biopsy technique and associated procedural complexity. 1,2 Prior to the coding changes, all biopsies were reimbursed at the same payment level, but a punch biopsy (11104; national nonfacility Medicare payment, $133.29) is now reimbursed more than a shave biopsy (11102; national nonfacility Medicare payment, $106.42). 3 We sought to evaluate whether the decrease in reimbursement for shave biopsies and concurrent increase in reimbursement for punch biopsies led to a shift from shave to punch biopsy utilization.
Methods
We examined shave and punch biopsies submitted for pathologic examination at Brigham and Women’s Hospital, Massachusetts General Hospital, and Massachusetts General Physician’s Organization (all in Boston, Massachusetts), and Penn Medicine, University of Pennsylvania Health System (Philadelphia, Pennsylvania), in May 2018 vs May 2019 (four months after new codes were implemented). This study was approved by Partners HealthCare (Boston, Massachusetts) and the University of Pennsylvania institutional review boards.
We included shave and punch biopsies of skin performed by physician dermatologists and mid-level providers (ie, physician assistants, nurse practitioners) at dermatology practices. All biopsies performed by a technique other than shave or punch, unspecified biopsy type, consultation cases, nonskin biopsies (eg, mucosa), and biopsies performed at nondermatology practices were excluded. We also excluded biopsies by providers who were not present during both study periods to account for provider mix.
Statistical Analysis
To evaluate for changes in the ratio of shave to punch biopsy utilization over time, we performed χ2 tests. Because care practices may differ between private and academic settings as well as between physicians and mid-level providers, we performed subgroup analyses by practice setting and provider type.4
Results
We identified 11,785 biopsies (12.11% of which were punch) submitted for pathologic examination in May 2018 compared to 11,291 biopsies (12.08% of which were punch) in May 2019 (Table). The overall use of punch biopsies relative to shave biopsies did not change between the years. There was a relative decrease in punch biopsy use among academic practices (−1.88%; P=.032) and a relative increase in punch biopsy use among private practices (+0.90%; P=.043). Provider type was not associated with differing utilization of biopsy type.
Comment
Overall, there was not a considerable shift in utilization behavior from shave to punch biopsies after the introduction of new coding changes. However, our study does demonstrate a small yet significant increase in punch biopsy utilization among private practices, and a decrease among academic practices. Although the change in biopsy utilization behavior is small in magnitude, it may have a substantial impact when extrapolated to behavior across the entire United States.
We were unable to assess additional factors, such as clinical diagnosis, body site, and cosmetic concerns, that may impact biopsy type selection in this study. Although we included multiple study sites to improve generalizability, our findings may not be representative of all biopsies performed in the dermatology setting. The baseline difference in relative punch biopsy use in academic vs private practices may reflect differences in patient populations and chief concerns, but assuming these features are stable over a 1-year time period, our findings should remain valid. Future studies should focus on qualitative evaluations of physician decision-making and evaluate whether similar trends persist over time.
Conclusion
Skin biopsy utilization trends among differing practice and provider types should continue to be monitored to assess for longitudinal trends in utilization within the context of updated billing codes and associated reimbursements.
In 2019, the 2 Current Procedural Terminology (CPT) codes for skin biopsies (11100 and 11101) were replaced with 6 new CPT codes that specify biopsy technique and associated procedural complexity. 1,2 Prior to the coding changes, all biopsies were reimbursed at the same payment level, but a punch biopsy (11104; national nonfacility Medicare payment, $133.29) is now reimbursed more than a shave biopsy (11102; national nonfacility Medicare payment, $106.42). 3 We sought to evaluate whether the decrease in reimbursement for shave biopsies and concurrent increase in reimbursement for punch biopsies led to a shift from shave to punch biopsy utilization.
Methods
We examined shave and punch biopsies submitted for pathologic examination at Brigham and Women’s Hospital, Massachusetts General Hospital, and Massachusetts General Physician’s Organization (all in Boston, Massachusetts), and Penn Medicine, University of Pennsylvania Health System (Philadelphia, Pennsylvania), in May 2018 vs May 2019 (four months after new codes were implemented). This study was approved by Partners HealthCare (Boston, Massachusetts) and the University of Pennsylvania institutional review boards.
We included shave and punch biopsies of skin performed by physician dermatologists and mid-level providers (ie, physician assistants, nurse practitioners) at dermatology practices. All biopsies performed by a technique other than shave or punch, unspecified biopsy type, consultation cases, nonskin biopsies (eg, mucosa), and biopsies performed at nondermatology practices were excluded. We also excluded biopsies by providers who were not present during both study periods to account for provider mix.
Statistical Analysis
To evaluate for changes in the ratio of shave to punch biopsy utilization over time, we performed χ2 tests. Because care practices may differ between private and academic settings as well as between physicians and mid-level providers, we performed subgroup analyses by practice setting and provider type.4
Results
We identified 11,785 biopsies (12.11% of which were punch) submitted for pathologic examination in May 2018 compared to 11,291 biopsies (12.08% of which were punch) in May 2019 (Table). The overall use of punch biopsies relative to shave biopsies did not change between the years. There was a relative decrease in punch biopsy use among academic practices (−1.88%; P=.032) and a relative increase in punch biopsy use among private practices (+0.90%; P=.043). Provider type was not associated with differing utilization of biopsy type.
Comment
Overall, there was not a considerable shift in utilization behavior from shave to punch biopsies after the introduction of new coding changes. However, our study does demonstrate a small yet significant increase in punch biopsy utilization among private practices, and a decrease among academic practices. Although the change in biopsy utilization behavior is small in magnitude, it may have a substantial impact when extrapolated to behavior across the entire United States.
We were unable to assess additional factors, such as clinical diagnosis, body site, and cosmetic concerns, that may impact biopsy type selection in this study. Although we included multiple study sites to improve generalizability, our findings may not be representative of all biopsies performed in the dermatology setting. The baseline difference in relative punch biopsy use in academic vs private practices may reflect differences in patient populations and chief concerns, but assuming these features are stable over a 1-year time period, our findings should remain valid. Future studies should focus on qualitative evaluations of physician decision-making and evaluate whether similar trends persist over time.
Conclusion
Skin biopsy utilization trends among differing practice and provider types should continue to be monitored to assess for longitudinal trends in utilization within the context of updated billing codes and associated reimbursements.
- Grider D. 2019 CPT® coding for skin biopsies. ICD10 monitor website. September 17, 2018. Updated January 7, 2019. Accessed February 17, 2021. https://www.icd10monitor.com/2019-cpt-coding-for-skin-biopsies 2.
- Tongdee E, Siegel DM, Markowitz O. New diagnostic procedure codes and reimbursement. Cutis. 2019;103:208-211.
- Search the physician fee schedule. Centers for Medicare & Medicaid Services website. Updated January 20, 2021. Accessed February 17, 2021. https://www.cms.gov/medicare/physician-fee-schedule/search
- Zhang M, Zippin J, Kaffenberger B. Trends and scope of dermatology procedures billed by advanced practice professionals from 2012 through 2015. JAMA Dermatol. 2018;154:1040-1044.
- Grider D. 2019 CPT® coding for skin biopsies. ICD10 monitor website. September 17, 2018. Updated January 7, 2019. Accessed February 17, 2021. https://www.icd10monitor.com/2019-cpt-coding-for-skin-biopsies 2.
- Tongdee E, Siegel DM, Markowitz O. New diagnostic procedure codes and reimbursement. Cutis. 2019;103:208-211.
- Search the physician fee schedule. Centers for Medicare & Medicaid Services website. Updated January 20, 2021. Accessed February 17, 2021. https://www.cms.gov/medicare/physician-fee-schedule/search
- Zhang M, Zippin J, Kaffenberger B. Trends and scope of dermatology procedures billed by advanced practice professionals from 2012 through 2015. JAMA Dermatol. 2018;154:1040-1044.
Practice Points
- Dermatologists should be aware that skin biopsy billing codes and reimbursements were changed in 2019 to reflect their level of complexity, which may impact how often each type of biopsy is used.
- Even small shifts in biopsy utilization behavior among dermatologists in the context of reimbursement changes can have a large impact on net reimbursements.
Upper Lip Anatomy, Mechanics of Local Flaps, and Considerations for Reconstruction
The upper lip poses challenges during reconstruction. Distortion of well-defined anatomic structures, including the vermilion border, oral commissures, Cupid’s bow, and philtrum, leads to noticeable deformities. Furthermore, maintenance of upper and lower lip function is essential for verbal communication, facial expression, and controlled opening of the oral cavity.
Similar to a prior review focused on the lower lip,1 we conducted a review of the literature using the PubMed database (1976-2017) and the following search terms: upper lip, lower lip, anatomy, comparison, cadaver, histology, local flap, and reconstruction. We reviewed studies that assessed anatomic and histologic characteristics of the upper and the lower lips, function of the upper lip, mechanics of local flaps, and upper lip reconstruction techniques including local flaps and regional flaps. Articles with an emphasis on free flaps were excluded.
The initial search resulted in 1326 articles. Of these, 1201 were excluded after abstracts were screened. Full-text review of the remaining 125 articles resulted in exclusion of 85 papers (9 foreign language, 4 duplicates, and 72 irrelevant). Among the 40 articles eligible for inclusion, 12 articles discussed anatomy and histology of the upper lip, 9 examined function of the upper lip, and 19 reviewed available techniques for reconstruction of the upper lip.
In this article, we review the anatomy and function of the upper lip as well as various repair techniques to provide the reconstructive surgeon with greater familiarity with the local flaps and an algorithmic approach for upper lip reconstruction.
Anatomic Characteristics of the Upper Lip
The muscular component of the upper lip primarily is comprised of the orbicularis oris (OO) muscle divided into 2 distinct concentric components: pars peripheralis and pars marginalis.2,3 It is discontinuous in some individuals.4 Although OO is the primary muscle of the lower lip, the upper lip is remarkably complex. Orbicularis oris and 3 additional muscles contribute to upper lip function: depressor septi nasi, the alar portion of the nasalis, and levator labii superioris alaeque nasi (LLSAN).5
The modiolus, a muscular structure located just lateral to the commissures, serves as a convergence point for facial muscle animation and lip function while distributing contraction forces between the lips and face.6 It is imperative to preserve its location in reconstruction to allow for good functional and aesthetic outcomes.
The upper lip is divided into 3 distinct aesthetic subunits: the philtrum and 1 lateral subunit on each side.7,8 Its unique surface features include the Cupid’s bow, vermilion tubercle, and philtral columns. The philtral columns are created by the dermal insertion on each side of the OO, which originates from the modiolus, decussates, and inserts into the skin of the contralateral philtral groove.2,9-11 The OO has additional insertions into the dermis lateral to the philtrum.5 During its course across the midline, it decreases its insertions, leading to the formation and thinness of the philtral dimple.9 The philtral shape primarily is due to the intermingling of LLSAN and the pars peripheralis in an axial plane. The LLSAN enters superolateral to the ipsilateral philtral ridge and courses along this ridge to contribute to the philtral shape.2 Formation of the philtrum’s contour arises from the opposing force of both muscles pulling the skin in opposite directions.2,5 The vermilion tubercle arises from the dermal insertion of the pars marginalis originating from the ipsilateral modiolus and follows the vermilion border.2 The Cupid’s bow is part of the white roll at the vermilion-cutaneous junction produced by the anterior projection of the pars peripheralis.10 The complex anatomy of this structure explains the intricacy of lip reconstructions in this area.
Function of the Upper Lip
Although the primary purpose of OO is sphincteric function, the upper lip’s key role is coverage of dentition and facial animation.12 The latter is achieved through the relationship of multiple muscles, including levator labii superioris, levator septi nasi, risorius, zygomaticus minor, zygomaticus major, levator anguli oris, and buccinator.7,13-17 Their smooth coordination results in various facial expressions. In comparison, the lower lip is critical for preservation of oral competence, prevention of drooling, eating, and speech due to the actions of OO and vertical support from the mentalis muscle.1,18-22
Reconstructive Methods for the Upper Lip
Multiple options are available for reconstruction of upper lip defects, with the aim to preserve facial animation and coverage of dentition. When animation muscles are involved, restoring function is the goal, which can be achieved by placing sutures to reapproximate the muscle edges in smaller defects or anchor the remaining muscle edge to preserve deep structures in larger defects, respecting the vector of contraction and attempting simulation of the muscle function. Additionally, restoration of the continuity of OO also is important for good aesthetic and functional outcomes.
Janis23 proposed the rule of thirds to approach upper and lower lip reconstruction. Using these rules, we briefly analyze the available flaps focusing on animation, OO restoration, preservation of the modiolus position, and sensation for each (eTable).
The perialar crescentic flap, an advancement flap, can be utilized for laterally located partial-thickness defects affecting up to one-third of the upper lip, especially those adjacent to the alar base, as well as full-thickness defects affecting up to two-thirds of the upper lip.7,24 The OO continuity and position of the modiolus often are preserved, sensation is maintained, and muscles of animation commonly are unaffected by this flap, especially in partial-thickness defects. In males, caution should be exercised where non–hair-bearing skin of the cheek is advanced to the upper lip region. Other potential complications include obliteration of the melolabial crease and pincushioning.7
Nasolabial (ie, melolabial) flaps are suggested for repair of defects up to one-third of the upper lip, especially when the vermilion is unaffected, or in lateral defects with or without commissure involvement.7,24-28 This flap is based on the facial artery and may be used as a direct transposition, V-Y advancement, or island flap with good aesthetic and functional outcomes (Figure 1).29,30 There is limited literature regarding the effects on animation. However, it may be beneficial in avoiding microstomia, as regional tissue is transferred from the cheek area, maintaining upper lip length. Additionally, the location of the modiolus often is unaffected, especially when the flap is harvested above the level of the muscle, providing superior facial animation function. Flap design is critical in areas lateral to the commissure and over the modiolus, as distortion of its position can occur.26 Similar to crescentic advancement, it is important to exercise caution in male patients, as non–hair-bearing tissue can be transferred to the upper lip. Reported adverse outcomes of the nasolabial flap include a thin flat upper lip, obliteration of the Cupid’s bow, and hypoesthesia that may improve over time.30
The Abbe flap is suitable for reconstruction of upper lip defects affecting up to two-thirds of the upper lip and lateral defects, provided the commissure or philtrum is unaffected.7,8 It is a 2-stage lip-switch flap based on the inferior labial artery, where tissue is harvested and transferred from the lower lip (Figure 2).23,31 It is particularly useful for philtral reconstruction, as incision lines at the flap edges can recreate the skin folds of the philtrum. Moreover, incision lines are better concealed under the nose, making it favorable for female patients. Surgeons should consider the difference in philtral width between sexes when designing this flap for optimal aesthetic outcome, as males have larger philtral width than females.21 The Abbe flap allows preservation of the Cupid’s bow, oral commissure, and modiolus position; however, it is an insensate flap and does not establish continuity of OO.23 For central defects, the function of animation muscles is not critically affected. In philtral reconstruction using an Abbe flap, a common adverse outcome is widening of the central segment because of tension and contraction forces applied by the adjacent OO. Restoration of the continuity of the muscle through dissection and advancement in small defects or anchoring of muscle edges on deeper surfaces may avoid direct pull on the flap. In larger central defects extending beyond the native philtrum, it is important to recreate the philtrum proportional to the remaining upper and lower lips. The recommended technique is a combination of a thin Abbe flap with bilateral perialar crescentic advancement flaps to maintain a proportional philtrum. Several variations have been described, including 3D planning with muscular suspension for natural raised philtral columns, avoiding a flat upper lip.5
The Yu flap, a sensate single-stage rotational advancement flap, can be used in a variety of ways for repair of upper lip defects, depending on the size and location.26 Lateral defects up to one-half of the upper lip should be repaired with a unilateral reverse Yu flap, central defects up to one-half of the upper lip can be reconstructed with bilateral reverse Yu flaps, and defects up to two-thirds of the upper lip can be repaired with bilateral Yu flaps. This flap restores OO continuity and thus preserves sphincter function, minimizes oral incompetence, and has a low risk of microstomia. The muscles of facial animation are preserved, yet the modiolus is not. Good aesthetic outcomes have been reported depending on the location of the Yu flap because scars can be placed in the nasolabial sulcus, commissures, or medially to recreate the philtrum.26
The Estlander flap is a single-stage flap utilizing donor tissue from the opposing lip for reconstruction of lateral defects up to two-thirds of the upper lip with commissure and philtrum involvement (Figure 3).8,23,32 It is an insensate flap that alters the position of the modiolus, distorting oral and facial animation.23 The superomedial position of the modiolus is better tolerated in the upper lip because it increases the relaxation tone of the lower lip and simulates the vector of contraction of major animation muscles, positively impacting the sphincteric function of the reconstructed lip. Sphincteric function action is not as impaired compared with the lower lip because the new position of the modiolus tightens the lower lip and prevents drooling.33 When designing the flap, one should consider that the inferior labial artery has been reported to remain with 10 mm of the superior border of the lower lip; therefore, pedicles of the Abbe and Estlander flaps should be at least 10 mm from the vermilion border to preserve vascular supply.34,35
The Karapandzic flap, a modified Gilles fan flap, can be employed for repair of central defects up to two-thirds of the upper lip.8,23,32,36-39 The bilateral advancement of full-thickness adjacent tissue edges preserves neurovascular structures allowing sensation and restores OO continuation.40 Prior studies have shown the average distance of the superior labial artery emergence from the facial artery and labial commissure is 12.1 mm; thus, at least 12.1 mm of tissue from the commissure should be preserved to prevent vascular compromise in Karapandzic flaps.34,35 The modiolus position is altered, and facial animation muscles are disrupted, consequently impairing facial animation, especially elevation of the lip.36 The philtrum is obliterated, producing unfavorable aesthetic outcomes. Finally, the upper lip is thinner and smaller in volume than the lower lip, increasing the risk for microstomia compared with the lower lip with a similar reconstructive technique.36
Defects larger than two-thirds of the upper lip require a Bernard Burrow flap, distant free flap, or combination of multiple regional and local flaps dependent on the characteristics of the defect.36,41 Distant free flaps are beyond the scope of this review. The Bernard Burrow flap consists of bilaterally opposing cheek advancement flaps. It is an insensate flap that does not restore OO continuity, producing minimal muscle function and poor animation. Microstomia is a common adverse outcome.36
Conclusion
Comprehensive understanding of labial anatomy and its intimate relationship to function and aesthetics of the upper lip are critical. Flap anatomy and mechanics are key factors for successful reconstruction. The purpose of this article is to utilize knowledge of histology, anatomy, and function of the upper lip to improve the outcomes of reconstruction. The Abbe flap often is utilized for reconstruction of the philtrum and central upper lip defects, though it is a less desirable option for lower lip reconstruction. The Karapandzic flap, while sensate and restorative of OO continuity, may have less optimal functional and cosmetic results compared with its use in the lower lip. Regarding lateral defects involving the commissure, the Estlander flap provides a reasonable option for the upper lip when compared with its use in lower lip defects, where outcomes are usually inferior.
- Boukovalas S, Boson AL, Hays JP, et al. A systematic review of lower lip anatomy, mechanics of local flaps, and special considerations for lower lip reconstruction. J Drugs Dermatol. 2017;16:1254-1261.
- Wu J, Yin N. Detailed anatomy of the nasolabial muscle in human fetuses as determined by micro-CT combined with iodine staining. Ann Plast Surg. 2016;76:111-116.
- Pepper JP, Baker SR. Local flaps: cheek and lip reconstruction. JAMA Facial Plast Surg. 2013;15:374-382.
- Rogers CR, Weinberg SM, Smith TD, et al. Anatomical basis for apparent subepithelial cleft lip: a histological and ultrasonographic survey of the orbicularis oris muscle. Cleft Palate Craniofac J. 2008;45:518-524.
- Yin N, Wu D, Wang Y, et al. Complete philtrum reconstruction on the partial-thickness cross-lip flap by nasolabial muscle tension line group reconstruction in the same stage of flap transfer. JAMA Facial Plast Surg. 2017;19:496-501.
- Al-Hoqail RA, Abdel Meguid EM. An anatomical and analytical study of the modiolus: enlightening its relevance to plastic surgery. Aesthetic Plast Surg. 2009;33:147-152.
- Galyon SW, Frodel JL. Lip and perioral defects. Otolaryngol Clin North Am. 2001;34:647-666.
- Massa AF, Otero-Rivas M, González-Sixto B, et al. Combined cutaneous rotation flap and myomucosal tongue flap for reconstruction of an upper lip defect. Actas Dermosifiliogr. 2014;105:869-871.
- Latham RA, Deaton TG. The structural basis of the philtrum and the contour of the vermilion border: a study of the musculature of the upper lip. J Anat. 1976;121:151-160.
- Garcia de Mitchell CA, Pessa JE, Schaverien MV, et al. The philtrum: anatomical observations from a new perspective. Plast Reconstr Surg. 2008;122:1756-1760.
- Bo C, Ningbei Y. Reconstruction of upper lip muscle system by anatomy, magnetic resonance imaging, and serial histological sections. J Craniofac Surg. 2014;25:48-54.
- Ishii LE, Byrne PJ. Lip reconstruction. Facial Plast Surg Clin North Am. 2009;17:445-453.
- Hur MS, Youn KH, Hu KS, et al. New anatomic considerations on the levator labii superioris related with the nasal ala. J Craniofac Surg. 2010;21:258-260.
- Song R, Ma H, Pan F. The “levator septi nasi muscle” and its clinical significance. Plast Reconstr Surg. 2002;109:1707-1712; discussion 1713.
- Choi DY, Hur MS, Youn KH, et al. Clinical anatomic considerations of the zygomaticus minor muscle based on the morphology and insertion pattern. Dermatol Surg. 2014;40:858-863.
- Youn KH, Park JT, Park DS, et al. Morphology of the zygomaticus minor and its relationship with the orbicularis oculi muscle. J Craniofac Surg. 2012;23:546-548.
- Vercruysse H, Van Nassauw L, San Miguel-Moragas J, et al. The effect of a Le Fort I incision on nose and upper lip dynamics: unraveling the mystery of the “Le Fort I lip.” J Craniomaxillofac Surg. 2016;44:1917-1921.
- Vinkka-Puhakka H, Kean MR, Heap SW. Ultrasonic investigation of the circumoral musculature. J Anat. 1989;166:121-133.
- Ferrario VF, Rosati R, Peretta R, et al. Labial morphology: a 3-dimensional anthropometric study. J Oral Maxillofac Surg. 2009;67:1832-1839.
- Ferrario VF, Sforza C, Schmitz JH, et al. Normal growth and development of the lips: a 3-dimensional study from 6 years to adulthood using a geometric model. J Anat. 2000;196:415-423.
- Sforza C, Grandi G, Binelli M, et al. Age- and sex-related changes in three-dimensional lip morphology. Forensic Sci Int. 2010;200:182.e181-187.
- Wilson DB. Embryonic development of the head and neck: part 3, the face. Head Neck Surg. 1979;2:145-153.
- Janis JE, ed. Essentials of Plastic Surgery. 2nd ed. Boca Raton, FL: Taylor & Francis Group; 2014.
- Burusapat C, Pitiseree A. Advanced squamous cell carcinoma involving both upper and lower lips and oral commissure with simultaneous reconstruction by local flap: a case report. J Med Case Rep. 2012;6:23.
- El-Marakby HH. The versatile naso-labial flaps in facial reconstruction. J Egypt Natl Canc Inst. 2005;17:245-250.
- Li ZN, Li RW, Tan XX, et al. Yu’s flap for lower lip and reverse Yu’s flap for upper lip reconstruction: 20 years experience. Br J Oral Maxillofac Surg. 2013;51:767-772.
- Wollina U. Reconstructive surgery in advanced perioral non-melanoma skin cancer. Results in elderly patients. J Dermatol Case Rep. 2014;8:103-107.
- Younger RA. The versatile melolabial flap. Otolaryngol Head Neck Surg. 1992;107:721-726.
- Włodarkiewicz A, Wojszwiłło-Geppert E, Placek W, et al. Upper lip reconstruction with local island flap after neoplasm excision. Dermatol Surg. 1997;23:1075-1079.
- Cook JL. The reconstruction of two large full-thickness wounds of the upper lip with different operative techniques: when possible, a local flap repair is preferable to reconstruction with free tissue transfer. Dermatol Surg. 2013;39:281-289.
- Kriet JD, Cupp CL, Sherris DA, et al. The extended Abbé flap. Laryngoscope. 1995;105:988-992.
- Khan AA, Kulkarni JV. Karapandzic flap. Indian J Dent. 2014;5:107-109.
- Raschke GF, Rieger UM, Bader RD, et al. Lip reconstruction: an anthropometric and functional analysis of surgical outcomes. Int J Oral Maxillofac Surg. 2012;41:744-750.
- Maǧden O, Edizer M, Atabey A, et al. Cadaveric study of the arterial anatomy of the upper lip. Plast Reconstr Surg. 2004;114:355-359.
- Al-Hoqail RA, Meguid EM. Anatomic dissection of the arterial supply of the lips: an anatomical and analytical approach. J Craniofac Surg. 2008;19:785-794.
- Kim JC, Hadlock T, Varvares MA, et al. Hair-bearing temporoparietal fascial flap reconstruction of upper lip and scalp defects. Arch Facial Plast Surg. 2001;3:170-177.
- Teemul TA, Telfer A, Singh RP, et al. The versatility of the Karapandzic flap: a review of 65 cases with patient-reported outcomes. J Craniomaxillofac Surg. 2017;45:325-329.
- Matteini C, Mazzone N, Rendine G, et al. Lip reconstruction with local m-shaped composite flap. J Craniofac Surg. 2010;21:225-228.
- Williams EF, Setzen G, Mulvaney MJ. Modified Bernard-Burow cheek advancement and cross-lip flap for total lip reconstruction. Arch Otolaryngol Head Neck Surg. 1996;122:1253-1258.
- Jaquet Y, Pasche P, Brossard E, et al. Meyer’s surgical procedure for the treatment of lip carcinoma. Eur Arch Otorhinolaryngol. 2005;262:11-16.
- Dang M, Greenbaum SS. Modified Burow’s wedge flap for upper lateral lip defects. Dermatol Surg. 2000;26:497-498.
The upper lip poses challenges during reconstruction. Distortion of well-defined anatomic structures, including the vermilion border, oral commissures, Cupid’s bow, and philtrum, leads to noticeable deformities. Furthermore, maintenance of upper and lower lip function is essential for verbal communication, facial expression, and controlled opening of the oral cavity.
Similar to a prior review focused on the lower lip,1 we conducted a review of the literature using the PubMed database (1976-2017) and the following search terms: upper lip, lower lip, anatomy, comparison, cadaver, histology, local flap, and reconstruction. We reviewed studies that assessed anatomic and histologic characteristics of the upper and the lower lips, function of the upper lip, mechanics of local flaps, and upper lip reconstruction techniques including local flaps and regional flaps. Articles with an emphasis on free flaps were excluded.
The initial search resulted in 1326 articles. Of these, 1201 were excluded after abstracts were screened. Full-text review of the remaining 125 articles resulted in exclusion of 85 papers (9 foreign language, 4 duplicates, and 72 irrelevant). Among the 40 articles eligible for inclusion, 12 articles discussed anatomy and histology of the upper lip, 9 examined function of the upper lip, and 19 reviewed available techniques for reconstruction of the upper lip.
In this article, we review the anatomy and function of the upper lip as well as various repair techniques to provide the reconstructive surgeon with greater familiarity with the local flaps and an algorithmic approach for upper lip reconstruction.
Anatomic Characteristics of the Upper Lip
The muscular component of the upper lip primarily is comprised of the orbicularis oris (OO) muscle divided into 2 distinct concentric components: pars peripheralis and pars marginalis.2,3 It is discontinuous in some individuals.4 Although OO is the primary muscle of the lower lip, the upper lip is remarkably complex. Orbicularis oris and 3 additional muscles contribute to upper lip function: depressor septi nasi, the alar portion of the nasalis, and levator labii superioris alaeque nasi (LLSAN).5
The modiolus, a muscular structure located just lateral to the commissures, serves as a convergence point for facial muscle animation and lip function while distributing contraction forces between the lips and face.6 It is imperative to preserve its location in reconstruction to allow for good functional and aesthetic outcomes.
The upper lip is divided into 3 distinct aesthetic subunits: the philtrum and 1 lateral subunit on each side.7,8 Its unique surface features include the Cupid’s bow, vermilion tubercle, and philtral columns. The philtral columns are created by the dermal insertion on each side of the OO, which originates from the modiolus, decussates, and inserts into the skin of the contralateral philtral groove.2,9-11 The OO has additional insertions into the dermis lateral to the philtrum.5 During its course across the midline, it decreases its insertions, leading to the formation and thinness of the philtral dimple.9 The philtral shape primarily is due to the intermingling of LLSAN and the pars peripheralis in an axial plane. The LLSAN enters superolateral to the ipsilateral philtral ridge and courses along this ridge to contribute to the philtral shape.2 Formation of the philtrum’s contour arises from the opposing force of both muscles pulling the skin in opposite directions.2,5 The vermilion tubercle arises from the dermal insertion of the pars marginalis originating from the ipsilateral modiolus and follows the vermilion border.2 The Cupid’s bow is part of the white roll at the vermilion-cutaneous junction produced by the anterior projection of the pars peripheralis.10 The complex anatomy of this structure explains the intricacy of lip reconstructions in this area.
Function of the Upper Lip
Although the primary purpose of OO is sphincteric function, the upper lip’s key role is coverage of dentition and facial animation.12 The latter is achieved through the relationship of multiple muscles, including levator labii superioris, levator septi nasi, risorius, zygomaticus minor, zygomaticus major, levator anguli oris, and buccinator.7,13-17 Their smooth coordination results in various facial expressions. In comparison, the lower lip is critical for preservation of oral competence, prevention of drooling, eating, and speech due to the actions of OO and vertical support from the mentalis muscle.1,18-22
Reconstructive Methods for the Upper Lip
Multiple options are available for reconstruction of upper lip defects, with the aim to preserve facial animation and coverage of dentition. When animation muscles are involved, restoring function is the goal, which can be achieved by placing sutures to reapproximate the muscle edges in smaller defects or anchor the remaining muscle edge to preserve deep structures in larger defects, respecting the vector of contraction and attempting simulation of the muscle function. Additionally, restoration of the continuity of OO also is important for good aesthetic and functional outcomes.
Janis23 proposed the rule of thirds to approach upper and lower lip reconstruction. Using these rules, we briefly analyze the available flaps focusing on animation, OO restoration, preservation of the modiolus position, and sensation for each (eTable).
The perialar crescentic flap, an advancement flap, can be utilized for laterally located partial-thickness defects affecting up to one-third of the upper lip, especially those adjacent to the alar base, as well as full-thickness defects affecting up to two-thirds of the upper lip.7,24 The OO continuity and position of the modiolus often are preserved, sensation is maintained, and muscles of animation commonly are unaffected by this flap, especially in partial-thickness defects. In males, caution should be exercised where non–hair-bearing skin of the cheek is advanced to the upper lip region. Other potential complications include obliteration of the melolabial crease and pincushioning.7
Nasolabial (ie, melolabial) flaps are suggested for repair of defects up to one-third of the upper lip, especially when the vermilion is unaffected, or in lateral defects with or without commissure involvement.7,24-28 This flap is based on the facial artery and may be used as a direct transposition, V-Y advancement, or island flap with good aesthetic and functional outcomes (Figure 1).29,30 There is limited literature regarding the effects on animation. However, it may be beneficial in avoiding microstomia, as regional tissue is transferred from the cheek area, maintaining upper lip length. Additionally, the location of the modiolus often is unaffected, especially when the flap is harvested above the level of the muscle, providing superior facial animation function. Flap design is critical in areas lateral to the commissure and over the modiolus, as distortion of its position can occur.26 Similar to crescentic advancement, it is important to exercise caution in male patients, as non–hair-bearing tissue can be transferred to the upper lip. Reported adverse outcomes of the nasolabial flap include a thin flat upper lip, obliteration of the Cupid’s bow, and hypoesthesia that may improve over time.30
The Abbe flap is suitable for reconstruction of upper lip defects affecting up to two-thirds of the upper lip and lateral defects, provided the commissure or philtrum is unaffected.7,8 It is a 2-stage lip-switch flap based on the inferior labial artery, where tissue is harvested and transferred from the lower lip (Figure 2).23,31 It is particularly useful for philtral reconstruction, as incision lines at the flap edges can recreate the skin folds of the philtrum. Moreover, incision lines are better concealed under the nose, making it favorable for female patients. Surgeons should consider the difference in philtral width between sexes when designing this flap for optimal aesthetic outcome, as males have larger philtral width than females.21 The Abbe flap allows preservation of the Cupid’s bow, oral commissure, and modiolus position; however, it is an insensate flap and does not establish continuity of OO.23 For central defects, the function of animation muscles is not critically affected. In philtral reconstruction using an Abbe flap, a common adverse outcome is widening of the central segment because of tension and contraction forces applied by the adjacent OO. Restoration of the continuity of the muscle through dissection and advancement in small defects or anchoring of muscle edges on deeper surfaces may avoid direct pull on the flap. In larger central defects extending beyond the native philtrum, it is important to recreate the philtrum proportional to the remaining upper and lower lips. The recommended technique is a combination of a thin Abbe flap with bilateral perialar crescentic advancement flaps to maintain a proportional philtrum. Several variations have been described, including 3D planning with muscular suspension for natural raised philtral columns, avoiding a flat upper lip.5
The Yu flap, a sensate single-stage rotational advancement flap, can be used in a variety of ways for repair of upper lip defects, depending on the size and location.26 Lateral defects up to one-half of the upper lip should be repaired with a unilateral reverse Yu flap, central defects up to one-half of the upper lip can be reconstructed with bilateral reverse Yu flaps, and defects up to two-thirds of the upper lip can be repaired with bilateral Yu flaps. This flap restores OO continuity and thus preserves sphincter function, minimizes oral incompetence, and has a low risk of microstomia. The muscles of facial animation are preserved, yet the modiolus is not. Good aesthetic outcomes have been reported depending on the location of the Yu flap because scars can be placed in the nasolabial sulcus, commissures, or medially to recreate the philtrum.26
The Estlander flap is a single-stage flap utilizing donor tissue from the opposing lip for reconstruction of lateral defects up to two-thirds of the upper lip with commissure and philtrum involvement (Figure 3).8,23,32 It is an insensate flap that alters the position of the modiolus, distorting oral and facial animation.23 The superomedial position of the modiolus is better tolerated in the upper lip because it increases the relaxation tone of the lower lip and simulates the vector of contraction of major animation muscles, positively impacting the sphincteric function of the reconstructed lip. Sphincteric function action is not as impaired compared with the lower lip because the new position of the modiolus tightens the lower lip and prevents drooling.33 When designing the flap, one should consider that the inferior labial artery has been reported to remain with 10 mm of the superior border of the lower lip; therefore, pedicles of the Abbe and Estlander flaps should be at least 10 mm from the vermilion border to preserve vascular supply.34,35
The Karapandzic flap, a modified Gilles fan flap, can be employed for repair of central defects up to two-thirds of the upper lip.8,23,32,36-39 The bilateral advancement of full-thickness adjacent tissue edges preserves neurovascular structures allowing sensation and restores OO continuation.40 Prior studies have shown the average distance of the superior labial artery emergence from the facial artery and labial commissure is 12.1 mm; thus, at least 12.1 mm of tissue from the commissure should be preserved to prevent vascular compromise in Karapandzic flaps.34,35 The modiolus position is altered, and facial animation muscles are disrupted, consequently impairing facial animation, especially elevation of the lip.36 The philtrum is obliterated, producing unfavorable aesthetic outcomes. Finally, the upper lip is thinner and smaller in volume than the lower lip, increasing the risk for microstomia compared with the lower lip with a similar reconstructive technique.36
Defects larger than two-thirds of the upper lip require a Bernard Burrow flap, distant free flap, or combination of multiple regional and local flaps dependent on the characteristics of the defect.36,41 Distant free flaps are beyond the scope of this review. The Bernard Burrow flap consists of bilaterally opposing cheek advancement flaps. It is an insensate flap that does not restore OO continuity, producing minimal muscle function and poor animation. Microstomia is a common adverse outcome.36
Conclusion
Comprehensive understanding of labial anatomy and its intimate relationship to function and aesthetics of the upper lip are critical. Flap anatomy and mechanics are key factors for successful reconstruction. The purpose of this article is to utilize knowledge of histology, anatomy, and function of the upper lip to improve the outcomes of reconstruction. The Abbe flap often is utilized for reconstruction of the philtrum and central upper lip defects, though it is a less desirable option for lower lip reconstruction. The Karapandzic flap, while sensate and restorative of OO continuity, may have less optimal functional and cosmetic results compared with its use in the lower lip. Regarding lateral defects involving the commissure, the Estlander flap provides a reasonable option for the upper lip when compared with its use in lower lip defects, where outcomes are usually inferior.
The upper lip poses challenges during reconstruction. Distortion of well-defined anatomic structures, including the vermilion border, oral commissures, Cupid’s bow, and philtrum, leads to noticeable deformities. Furthermore, maintenance of upper and lower lip function is essential for verbal communication, facial expression, and controlled opening of the oral cavity.
Similar to a prior review focused on the lower lip,1 we conducted a review of the literature using the PubMed database (1976-2017) and the following search terms: upper lip, lower lip, anatomy, comparison, cadaver, histology, local flap, and reconstruction. We reviewed studies that assessed anatomic and histologic characteristics of the upper and the lower lips, function of the upper lip, mechanics of local flaps, and upper lip reconstruction techniques including local flaps and regional flaps. Articles with an emphasis on free flaps were excluded.
The initial search resulted in 1326 articles. Of these, 1201 were excluded after abstracts were screened. Full-text review of the remaining 125 articles resulted in exclusion of 85 papers (9 foreign language, 4 duplicates, and 72 irrelevant). Among the 40 articles eligible for inclusion, 12 articles discussed anatomy and histology of the upper lip, 9 examined function of the upper lip, and 19 reviewed available techniques for reconstruction of the upper lip.
In this article, we review the anatomy and function of the upper lip as well as various repair techniques to provide the reconstructive surgeon with greater familiarity with the local flaps and an algorithmic approach for upper lip reconstruction.
Anatomic Characteristics of the Upper Lip
The muscular component of the upper lip primarily is comprised of the orbicularis oris (OO) muscle divided into 2 distinct concentric components: pars peripheralis and pars marginalis.2,3 It is discontinuous in some individuals.4 Although OO is the primary muscle of the lower lip, the upper lip is remarkably complex. Orbicularis oris and 3 additional muscles contribute to upper lip function: depressor septi nasi, the alar portion of the nasalis, and levator labii superioris alaeque nasi (LLSAN).5
The modiolus, a muscular structure located just lateral to the commissures, serves as a convergence point for facial muscle animation and lip function while distributing contraction forces between the lips and face.6 It is imperative to preserve its location in reconstruction to allow for good functional and aesthetic outcomes.
The upper lip is divided into 3 distinct aesthetic subunits: the philtrum and 1 lateral subunit on each side.7,8 Its unique surface features include the Cupid’s bow, vermilion tubercle, and philtral columns. The philtral columns are created by the dermal insertion on each side of the OO, which originates from the modiolus, decussates, and inserts into the skin of the contralateral philtral groove.2,9-11 The OO has additional insertions into the dermis lateral to the philtrum.5 During its course across the midline, it decreases its insertions, leading to the formation and thinness of the philtral dimple.9 The philtral shape primarily is due to the intermingling of LLSAN and the pars peripheralis in an axial plane. The LLSAN enters superolateral to the ipsilateral philtral ridge and courses along this ridge to contribute to the philtral shape.2 Formation of the philtrum’s contour arises from the opposing force of both muscles pulling the skin in opposite directions.2,5 The vermilion tubercle arises from the dermal insertion of the pars marginalis originating from the ipsilateral modiolus and follows the vermilion border.2 The Cupid’s bow is part of the white roll at the vermilion-cutaneous junction produced by the anterior projection of the pars peripheralis.10 The complex anatomy of this structure explains the intricacy of lip reconstructions in this area.
Function of the Upper Lip
Although the primary purpose of OO is sphincteric function, the upper lip’s key role is coverage of dentition and facial animation.12 The latter is achieved through the relationship of multiple muscles, including levator labii superioris, levator septi nasi, risorius, zygomaticus minor, zygomaticus major, levator anguli oris, and buccinator.7,13-17 Their smooth coordination results in various facial expressions. In comparison, the lower lip is critical for preservation of oral competence, prevention of drooling, eating, and speech due to the actions of OO and vertical support from the mentalis muscle.1,18-22
Reconstructive Methods for the Upper Lip
Multiple options are available for reconstruction of upper lip defects, with the aim to preserve facial animation and coverage of dentition. When animation muscles are involved, restoring function is the goal, which can be achieved by placing sutures to reapproximate the muscle edges in smaller defects or anchor the remaining muscle edge to preserve deep structures in larger defects, respecting the vector of contraction and attempting simulation of the muscle function. Additionally, restoration of the continuity of OO also is important for good aesthetic and functional outcomes.
Janis23 proposed the rule of thirds to approach upper and lower lip reconstruction. Using these rules, we briefly analyze the available flaps focusing on animation, OO restoration, preservation of the modiolus position, and sensation for each (eTable).
The perialar crescentic flap, an advancement flap, can be utilized for laterally located partial-thickness defects affecting up to one-third of the upper lip, especially those adjacent to the alar base, as well as full-thickness defects affecting up to two-thirds of the upper lip.7,24 The OO continuity and position of the modiolus often are preserved, sensation is maintained, and muscles of animation commonly are unaffected by this flap, especially in partial-thickness defects. In males, caution should be exercised where non–hair-bearing skin of the cheek is advanced to the upper lip region. Other potential complications include obliteration of the melolabial crease and pincushioning.7
Nasolabial (ie, melolabial) flaps are suggested for repair of defects up to one-third of the upper lip, especially when the vermilion is unaffected, or in lateral defects with or without commissure involvement.7,24-28 This flap is based on the facial artery and may be used as a direct transposition, V-Y advancement, or island flap with good aesthetic and functional outcomes (Figure 1).29,30 There is limited literature regarding the effects on animation. However, it may be beneficial in avoiding microstomia, as regional tissue is transferred from the cheek area, maintaining upper lip length. Additionally, the location of the modiolus often is unaffected, especially when the flap is harvested above the level of the muscle, providing superior facial animation function. Flap design is critical in areas lateral to the commissure and over the modiolus, as distortion of its position can occur.26 Similar to crescentic advancement, it is important to exercise caution in male patients, as non–hair-bearing tissue can be transferred to the upper lip. Reported adverse outcomes of the nasolabial flap include a thin flat upper lip, obliteration of the Cupid’s bow, and hypoesthesia that may improve over time.30
The Abbe flap is suitable for reconstruction of upper lip defects affecting up to two-thirds of the upper lip and lateral defects, provided the commissure or philtrum is unaffected.7,8 It is a 2-stage lip-switch flap based on the inferior labial artery, where tissue is harvested and transferred from the lower lip (Figure 2).23,31 It is particularly useful for philtral reconstruction, as incision lines at the flap edges can recreate the skin folds of the philtrum. Moreover, incision lines are better concealed under the nose, making it favorable for female patients. Surgeons should consider the difference in philtral width between sexes when designing this flap for optimal aesthetic outcome, as males have larger philtral width than females.21 The Abbe flap allows preservation of the Cupid’s bow, oral commissure, and modiolus position; however, it is an insensate flap and does not establish continuity of OO.23 For central defects, the function of animation muscles is not critically affected. In philtral reconstruction using an Abbe flap, a common adverse outcome is widening of the central segment because of tension and contraction forces applied by the adjacent OO. Restoration of the continuity of the muscle through dissection and advancement in small defects or anchoring of muscle edges on deeper surfaces may avoid direct pull on the flap. In larger central defects extending beyond the native philtrum, it is important to recreate the philtrum proportional to the remaining upper and lower lips. The recommended technique is a combination of a thin Abbe flap with bilateral perialar crescentic advancement flaps to maintain a proportional philtrum. Several variations have been described, including 3D planning with muscular suspension for natural raised philtral columns, avoiding a flat upper lip.5
The Yu flap, a sensate single-stage rotational advancement flap, can be used in a variety of ways for repair of upper lip defects, depending on the size and location.26 Lateral defects up to one-half of the upper lip should be repaired with a unilateral reverse Yu flap, central defects up to one-half of the upper lip can be reconstructed with bilateral reverse Yu flaps, and defects up to two-thirds of the upper lip can be repaired with bilateral Yu flaps. This flap restores OO continuity and thus preserves sphincter function, minimizes oral incompetence, and has a low risk of microstomia. The muscles of facial animation are preserved, yet the modiolus is not. Good aesthetic outcomes have been reported depending on the location of the Yu flap because scars can be placed in the nasolabial sulcus, commissures, or medially to recreate the philtrum.26
The Estlander flap is a single-stage flap utilizing donor tissue from the opposing lip for reconstruction of lateral defects up to two-thirds of the upper lip with commissure and philtrum involvement (Figure 3).8,23,32 It is an insensate flap that alters the position of the modiolus, distorting oral and facial animation.23 The superomedial position of the modiolus is better tolerated in the upper lip because it increases the relaxation tone of the lower lip and simulates the vector of contraction of major animation muscles, positively impacting the sphincteric function of the reconstructed lip. Sphincteric function action is not as impaired compared with the lower lip because the new position of the modiolus tightens the lower lip and prevents drooling.33 When designing the flap, one should consider that the inferior labial artery has been reported to remain with 10 mm of the superior border of the lower lip; therefore, pedicles of the Abbe and Estlander flaps should be at least 10 mm from the vermilion border to preserve vascular supply.34,35
The Karapandzic flap, a modified Gilles fan flap, can be employed for repair of central defects up to two-thirds of the upper lip.8,23,32,36-39 The bilateral advancement of full-thickness adjacent tissue edges preserves neurovascular structures allowing sensation and restores OO continuation.40 Prior studies have shown the average distance of the superior labial artery emergence from the facial artery and labial commissure is 12.1 mm; thus, at least 12.1 mm of tissue from the commissure should be preserved to prevent vascular compromise in Karapandzic flaps.34,35 The modiolus position is altered, and facial animation muscles are disrupted, consequently impairing facial animation, especially elevation of the lip.36 The philtrum is obliterated, producing unfavorable aesthetic outcomes. Finally, the upper lip is thinner and smaller in volume than the lower lip, increasing the risk for microstomia compared with the lower lip with a similar reconstructive technique.36
Defects larger than two-thirds of the upper lip require a Bernard Burrow flap, distant free flap, or combination of multiple regional and local flaps dependent on the characteristics of the defect.36,41 Distant free flaps are beyond the scope of this review. The Bernard Burrow flap consists of bilaterally opposing cheek advancement flaps. It is an insensate flap that does not restore OO continuity, producing minimal muscle function and poor animation. Microstomia is a common adverse outcome.36
Conclusion
Comprehensive understanding of labial anatomy and its intimate relationship to function and aesthetics of the upper lip are critical. Flap anatomy and mechanics are key factors for successful reconstruction. The purpose of this article is to utilize knowledge of histology, anatomy, and function of the upper lip to improve the outcomes of reconstruction. The Abbe flap often is utilized for reconstruction of the philtrum and central upper lip defects, though it is a less desirable option for lower lip reconstruction. The Karapandzic flap, while sensate and restorative of OO continuity, may have less optimal functional and cosmetic results compared with its use in the lower lip. Regarding lateral defects involving the commissure, the Estlander flap provides a reasonable option for the upper lip when compared with its use in lower lip defects, where outcomes are usually inferior.
- Boukovalas S, Boson AL, Hays JP, et al. A systematic review of lower lip anatomy, mechanics of local flaps, and special considerations for lower lip reconstruction. J Drugs Dermatol. 2017;16:1254-1261.
- Wu J, Yin N. Detailed anatomy of the nasolabial muscle in human fetuses as determined by micro-CT combined with iodine staining. Ann Plast Surg. 2016;76:111-116.
- Pepper JP, Baker SR. Local flaps: cheek and lip reconstruction. JAMA Facial Plast Surg. 2013;15:374-382.
- Rogers CR, Weinberg SM, Smith TD, et al. Anatomical basis for apparent subepithelial cleft lip: a histological and ultrasonographic survey of the orbicularis oris muscle. Cleft Palate Craniofac J. 2008;45:518-524.
- Yin N, Wu D, Wang Y, et al. Complete philtrum reconstruction on the partial-thickness cross-lip flap by nasolabial muscle tension line group reconstruction in the same stage of flap transfer. JAMA Facial Plast Surg. 2017;19:496-501.
- Al-Hoqail RA, Abdel Meguid EM. An anatomical and analytical study of the modiolus: enlightening its relevance to plastic surgery. Aesthetic Plast Surg. 2009;33:147-152.
- Galyon SW, Frodel JL. Lip and perioral defects. Otolaryngol Clin North Am. 2001;34:647-666.
- Massa AF, Otero-Rivas M, González-Sixto B, et al. Combined cutaneous rotation flap and myomucosal tongue flap for reconstruction of an upper lip defect. Actas Dermosifiliogr. 2014;105:869-871.
- Latham RA, Deaton TG. The structural basis of the philtrum and the contour of the vermilion border: a study of the musculature of the upper lip. J Anat. 1976;121:151-160.
- Garcia de Mitchell CA, Pessa JE, Schaverien MV, et al. The philtrum: anatomical observations from a new perspective. Plast Reconstr Surg. 2008;122:1756-1760.
- Bo C, Ningbei Y. Reconstruction of upper lip muscle system by anatomy, magnetic resonance imaging, and serial histological sections. J Craniofac Surg. 2014;25:48-54.
- Ishii LE, Byrne PJ. Lip reconstruction. Facial Plast Surg Clin North Am. 2009;17:445-453.
- Hur MS, Youn KH, Hu KS, et al. New anatomic considerations on the levator labii superioris related with the nasal ala. J Craniofac Surg. 2010;21:258-260.
- Song R, Ma H, Pan F. The “levator septi nasi muscle” and its clinical significance. Plast Reconstr Surg. 2002;109:1707-1712; discussion 1713.
- Choi DY, Hur MS, Youn KH, et al. Clinical anatomic considerations of the zygomaticus minor muscle based on the morphology and insertion pattern. Dermatol Surg. 2014;40:858-863.
- Youn KH, Park JT, Park DS, et al. Morphology of the zygomaticus minor and its relationship with the orbicularis oculi muscle. J Craniofac Surg. 2012;23:546-548.
- Vercruysse H, Van Nassauw L, San Miguel-Moragas J, et al. The effect of a Le Fort I incision on nose and upper lip dynamics: unraveling the mystery of the “Le Fort I lip.” J Craniomaxillofac Surg. 2016;44:1917-1921.
- Vinkka-Puhakka H, Kean MR, Heap SW. Ultrasonic investigation of the circumoral musculature. J Anat. 1989;166:121-133.
- Ferrario VF, Rosati R, Peretta R, et al. Labial morphology: a 3-dimensional anthropometric study. J Oral Maxillofac Surg. 2009;67:1832-1839.
- Ferrario VF, Sforza C, Schmitz JH, et al. Normal growth and development of the lips: a 3-dimensional study from 6 years to adulthood using a geometric model. J Anat. 2000;196:415-423.
- Sforza C, Grandi G, Binelli M, et al. Age- and sex-related changes in three-dimensional lip morphology. Forensic Sci Int. 2010;200:182.e181-187.
- Wilson DB. Embryonic development of the head and neck: part 3, the face. Head Neck Surg. 1979;2:145-153.
- Janis JE, ed. Essentials of Plastic Surgery. 2nd ed. Boca Raton, FL: Taylor & Francis Group; 2014.
- Burusapat C, Pitiseree A. Advanced squamous cell carcinoma involving both upper and lower lips and oral commissure with simultaneous reconstruction by local flap: a case report. J Med Case Rep. 2012;6:23.
- El-Marakby HH. The versatile naso-labial flaps in facial reconstruction. J Egypt Natl Canc Inst. 2005;17:245-250.
- Li ZN, Li RW, Tan XX, et al. Yu’s flap for lower lip and reverse Yu’s flap for upper lip reconstruction: 20 years experience. Br J Oral Maxillofac Surg. 2013;51:767-772.
- Wollina U. Reconstructive surgery in advanced perioral non-melanoma skin cancer. Results in elderly patients. J Dermatol Case Rep. 2014;8:103-107.
- Younger RA. The versatile melolabial flap. Otolaryngol Head Neck Surg. 1992;107:721-726.
- Włodarkiewicz A, Wojszwiłło-Geppert E, Placek W, et al. Upper lip reconstruction with local island flap after neoplasm excision. Dermatol Surg. 1997;23:1075-1079.
- Cook JL. The reconstruction of two large full-thickness wounds of the upper lip with different operative techniques: when possible, a local flap repair is preferable to reconstruction with free tissue transfer. Dermatol Surg. 2013;39:281-289.
- Kriet JD, Cupp CL, Sherris DA, et al. The extended Abbé flap. Laryngoscope. 1995;105:988-992.
- Khan AA, Kulkarni JV. Karapandzic flap. Indian J Dent. 2014;5:107-109.
- Raschke GF, Rieger UM, Bader RD, et al. Lip reconstruction: an anthropometric and functional analysis of surgical outcomes. Int J Oral Maxillofac Surg. 2012;41:744-750.
- Maǧden O, Edizer M, Atabey A, et al. Cadaveric study of the arterial anatomy of the upper lip. Plast Reconstr Surg. 2004;114:355-359.
- Al-Hoqail RA, Meguid EM. Anatomic dissection of the arterial supply of the lips: an anatomical and analytical approach. J Craniofac Surg. 2008;19:785-794.
- Kim JC, Hadlock T, Varvares MA, et al. Hair-bearing temporoparietal fascial flap reconstruction of upper lip and scalp defects. Arch Facial Plast Surg. 2001;3:170-177.
- Teemul TA, Telfer A, Singh RP, et al. The versatility of the Karapandzic flap: a review of 65 cases with patient-reported outcomes. J Craniomaxillofac Surg. 2017;45:325-329.
- Matteini C, Mazzone N, Rendine G, et al. Lip reconstruction with local m-shaped composite flap. J Craniofac Surg. 2010;21:225-228.
- Williams EF, Setzen G, Mulvaney MJ. Modified Bernard-Burow cheek advancement and cross-lip flap for total lip reconstruction. Arch Otolaryngol Head Neck Surg. 1996;122:1253-1258.
- Jaquet Y, Pasche P, Brossard E, et al. Meyer’s surgical procedure for the treatment of lip carcinoma. Eur Arch Otorhinolaryngol. 2005;262:11-16.
- Dang M, Greenbaum SS. Modified Burow’s wedge flap for upper lateral lip defects. Dermatol Surg. 2000;26:497-498.
- Boukovalas S, Boson AL, Hays JP, et al. A systematic review of lower lip anatomy, mechanics of local flaps, and special considerations for lower lip reconstruction. J Drugs Dermatol. 2017;16:1254-1261.
- Wu J, Yin N. Detailed anatomy of the nasolabial muscle in human fetuses as determined by micro-CT combined with iodine staining. Ann Plast Surg. 2016;76:111-116.
- Pepper JP, Baker SR. Local flaps: cheek and lip reconstruction. JAMA Facial Plast Surg. 2013;15:374-382.
- Rogers CR, Weinberg SM, Smith TD, et al. Anatomical basis for apparent subepithelial cleft lip: a histological and ultrasonographic survey of the orbicularis oris muscle. Cleft Palate Craniofac J. 2008;45:518-524.
- Yin N, Wu D, Wang Y, et al. Complete philtrum reconstruction on the partial-thickness cross-lip flap by nasolabial muscle tension line group reconstruction in the same stage of flap transfer. JAMA Facial Plast Surg. 2017;19:496-501.
- Al-Hoqail RA, Abdel Meguid EM. An anatomical and analytical study of the modiolus: enlightening its relevance to plastic surgery. Aesthetic Plast Surg. 2009;33:147-152.
- Galyon SW, Frodel JL. Lip and perioral defects. Otolaryngol Clin North Am. 2001;34:647-666.
- Massa AF, Otero-Rivas M, González-Sixto B, et al. Combined cutaneous rotation flap and myomucosal tongue flap for reconstruction of an upper lip defect. Actas Dermosifiliogr. 2014;105:869-871.
- Latham RA, Deaton TG. The structural basis of the philtrum and the contour of the vermilion border: a study of the musculature of the upper lip. J Anat. 1976;121:151-160.
- Garcia de Mitchell CA, Pessa JE, Schaverien MV, et al. The philtrum: anatomical observations from a new perspective. Plast Reconstr Surg. 2008;122:1756-1760.
- Bo C, Ningbei Y. Reconstruction of upper lip muscle system by anatomy, magnetic resonance imaging, and serial histological sections. J Craniofac Surg. 2014;25:48-54.
- Ishii LE, Byrne PJ. Lip reconstruction. Facial Plast Surg Clin North Am. 2009;17:445-453.
- Hur MS, Youn KH, Hu KS, et al. New anatomic considerations on the levator labii superioris related with the nasal ala. J Craniofac Surg. 2010;21:258-260.
- Song R, Ma H, Pan F. The “levator septi nasi muscle” and its clinical significance. Plast Reconstr Surg. 2002;109:1707-1712; discussion 1713.
- Choi DY, Hur MS, Youn KH, et al. Clinical anatomic considerations of the zygomaticus minor muscle based on the morphology and insertion pattern. Dermatol Surg. 2014;40:858-863.
- Youn KH, Park JT, Park DS, et al. Morphology of the zygomaticus minor and its relationship with the orbicularis oculi muscle. J Craniofac Surg. 2012;23:546-548.
- Vercruysse H, Van Nassauw L, San Miguel-Moragas J, et al. The effect of a Le Fort I incision on nose and upper lip dynamics: unraveling the mystery of the “Le Fort I lip.” J Craniomaxillofac Surg. 2016;44:1917-1921.
- Vinkka-Puhakka H, Kean MR, Heap SW. Ultrasonic investigation of the circumoral musculature. J Anat. 1989;166:121-133.
- Ferrario VF, Rosati R, Peretta R, et al. Labial morphology: a 3-dimensional anthropometric study. J Oral Maxillofac Surg. 2009;67:1832-1839.
- Ferrario VF, Sforza C, Schmitz JH, et al. Normal growth and development of the lips: a 3-dimensional study from 6 years to adulthood using a geometric model. J Anat. 2000;196:415-423.
- Sforza C, Grandi G, Binelli M, et al. Age- and sex-related changes in three-dimensional lip morphology. Forensic Sci Int. 2010;200:182.e181-187.
- Wilson DB. Embryonic development of the head and neck: part 3, the face. Head Neck Surg. 1979;2:145-153.
- Janis JE, ed. Essentials of Plastic Surgery. 2nd ed. Boca Raton, FL: Taylor & Francis Group; 2014.
- Burusapat C, Pitiseree A. Advanced squamous cell carcinoma involving both upper and lower lips and oral commissure with simultaneous reconstruction by local flap: a case report. J Med Case Rep. 2012;6:23.
- El-Marakby HH. The versatile naso-labial flaps in facial reconstruction. J Egypt Natl Canc Inst. 2005;17:245-250.
- Li ZN, Li RW, Tan XX, et al. Yu’s flap for lower lip and reverse Yu’s flap for upper lip reconstruction: 20 years experience. Br J Oral Maxillofac Surg. 2013;51:767-772.
- Wollina U. Reconstructive surgery in advanced perioral non-melanoma skin cancer. Results in elderly patients. J Dermatol Case Rep. 2014;8:103-107.
- Younger RA. The versatile melolabial flap. Otolaryngol Head Neck Surg. 1992;107:721-726.
- Włodarkiewicz A, Wojszwiłło-Geppert E, Placek W, et al. Upper lip reconstruction with local island flap after neoplasm excision. Dermatol Surg. 1997;23:1075-1079.
- Cook JL. The reconstruction of two large full-thickness wounds of the upper lip with different operative techniques: when possible, a local flap repair is preferable to reconstruction with free tissue transfer. Dermatol Surg. 2013;39:281-289.
- Kriet JD, Cupp CL, Sherris DA, et al. The extended Abbé flap. Laryngoscope. 1995;105:988-992.
- Khan AA, Kulkarni JV. Karapandzic flap. Indian J Dent. 2014;5:107-109.
- Raschke GF, Rieger UM, Bader RD, et al. Lip reconstruction: an anthropometric and functional analysis of surgical outcomes. Int J Oral Maxillofac Surg. 2012;41:744-750.
- Maǧden O, Edizer M, Atabey A, et al. Cadaveric study of the arterial anatomy of the upper lip. Plast Reconstr Surg. 2004;114:355-359.
- Al-Hoqail RA, Meguid EM. Anatomic dissection of the arterial supply of the lips: an anatomical and analytical approach. J Craniofac Surg. 2008;19:785-794.
- Kim JC, Hadlock T, Varvares MA, et al. Hair-bearing temporoparietal fascial flap reconstruction of upper lip and scalp defects. Arch Facial Plast Surg. 2001;3:170-177.
- Teemul TA, Telfer A, Singh RP, et al. The versatility of the Karapandzic flap: a review of 65 cases with patient-reported outcomes. J Craniomaxillofac Surg. 2017;45:325-329.
- Matteini C, Mazzone N, Rendine G, et al. Lip reconstruction with local m-shaped composite flap. J Craniofac Surg. 2010;21:225-228.
- Williams EF, Setzen G, Mulvaney MJ. Modified Bernard-Burow cheek advancement and cross-lip flap for total lip reconstruction. Arch Otolaryngol Head Neck Surg. 1996;122:1253-1258.
- Jaquet Y, Pasche P, Brossard E, et al. Meyer’s surgical procedure for the treatment of lip carcinoma. Eur Arch Otorhinolaryngol. 2005;262:11-16.
- Dang M, Greenbaum SS. Modified Burow’s wedge flap for upper lateral lip defects. Dermatol Surg. 2000;26:497-498.
Vibrio vulnificus: Review of Mild to Life-threatening Skin Infections
Vibrio vulnificus is a member of the Vibrio genus. Most Vibrio species are nonpathogenic in humans; however, V vulnificus is one of the pathogenic strains.1 In Latin, the term vulnificus means “wounding,” and V vulnificus can cause life-threatening infections in patients. The mortality rate of V vulnificus infections is approximately 33% in the United States.2Vibrio vulnificus is a gram-negative bacterium that was first isolated by the Centers for Disease Control and Prevention in 1964 and was given its current name in 1979.3-6 It has been found in numerous organisms, including oysters, crabs, clams, shrimp, mussels, mullets, and sea bass.4 The vast majority of infections in the United States are due to oyster exposure and consumption.2,7Vibrio vulnificus is responsible for more than 95% of seafood-related deaths in the United States and has the highest mortality rate of all food-borne illness in the United States.2,5 It also has the highest per-case economic impact of all food-related diseases in the United States.1
What distinguishes a pathogenic vs nonpathogenic Vibrio isolate remains unknown; Vibrio species rapidly undergo horizontal gene transfer, making DNA isolation difficult.1 Some characteristics of V vulnificus that may confer virulence are the capsular polysaccharide, lipopolysaccharide, binding proteins, and tissue-degrading enzymes.1,5 First, encapsulated strains are more virulent and invasive than unencapsulated strains.1 The mucopolysaccharide capsule protects the bacterium from the immune system, allowing it to evade immune surveillance, cause more severe infection, and invade into the subcutaneous tissue.3,5 Second, production of sialic acid–like molecules alter the lipopolysaccharide, allowing for motility and biofilm formation.1 This allows the bacterium to survive in marine waters and within the bloodstream, the latter leading to sepsis in humans. Third, production of N-acetylglucosamine–binding protein A allows for adhesion to chitin. Shellfish consume chitin, and chitin accumulates in shellfish. N-acetylglucosamine–binding protein A also binds mucin; this may be how V vulnificus binds to mucin in the gastrointestinal tract in humans, causing gastroenteritis.1 Binding to the human mucosae also may allow the bacteria to gain access to the blood supply, leading to septicemia.4 Finally, tissue-degrading enzymes such as proteases are responsible for necrotizing wound infections associated with V vulnificus, as the enzymes allow for invasion into the skin and subcutaneous tissues. Proteases also increase vascular permeability and lead to edema.3 Hence, these virulence factors may provide V vulnificus the pathogenicity to cause infection in humans.
Three biotypes of V vulnificus have been discovered. Biotype 1 is the most common and is found worldwide in brackish water.8 It can cause the entire spectrum of illnesses, and it has a case fatality rate of 50% in humans. Biotype 1 is presumably responsible for all infections in the United States. Biotype 2 is found in the Far East and Western Europe; it inhabits a unique niche—saltwater used for eel farming. It typically causes infection in eels, but rarely it can cause wound infections in humans. Biotype 3 is found in freshwater fish farming in Israel, and it is a hybrid of biotypes 1 and 2.It can cause severe soft tissue infections in humans, sometimes requiring amputation.8
Epidemiology
Vibrio vulnificus is a motile, gram-negative, halophilic, aquatic bacterium.1,4,5,8,9 It is part of the normal estuarine microbiome and typically is found in warm coastal waters.1,5,10 The ideal conditions for growth and survival of V vulnificus are water temperatures at 18 °C (64.4 °F) and water salinities between 15 to 25 parts per thousand.2,8,9 These conditions are found in tropical and subtropical regions.2Vibrio vulnificus is found all over the world, including Denmark, Italy, Japan, Australia, Brazil, and the United States,2 where most infections come from oyster exposure and consumption in the Gulf of Mexico.2,8,11 The incidence of infection in the United States is highest between April and October.8,11
Some populations are at a higher risk of infection. Risk factors include male sex, liver cirrhosis, hemochromatosis, end-stage renal disease, immunosuppression, and diabetes mellitus.1,8,11 Healthy patients with no risk factors account for less than 5% of US V vulnificus infections.8
Male Predilection
Men are 6 times more likely to be affected by V vulnificus than women.Some hypotheses for this discrepancy are that estrogen is protective againstV vulnificus and that women may be less likely to engage in risky water activities and seafood handling.5 Additionally, older males (aged >60 years) are most often affected,1,8 likely due to the association between increasing age with number of comorbidities, such as diabetes mellitus, heart disease, and chronic disease.8
Iron Levels
Iron appears to play an important role in V vulnificus infection. Iron is essential for bacterial growth, and the ability to obtain iron from a host increases the organism’s pathogenicity.3Vibrio vulnificus rapidly grows when transferrin saturation exceeds 70%.8 Additionally, iron overload decreases the inoculum needed to cause sepsis in animal studies, which could play a role in human pathogenesis.4 Iron levels are elevated in patients with hemochromatosis due to increased iron absorption, cirrhosis and chronic liver disease due to impaired iron metabolism, and end-stage renal disease, especially in patients receiving parenteral iron.8
Immunosuppression
Patients who are immunocompromised and those with chronic liver disease are at an increased risk of infection because of neutrophils having decreased phagocytic activity.4
Diabetes Mellitus
Patients with diabetes mellitus may have peripheral neuropathy and may be unaware of pre-existing wounds that serve as entry points for V vulnificus.12
Etiology
Vibrio vulnificus infects humans via seafood consumption and handling as well as exposure to contaminated water.2,5 With respect to seafood consumption, raw shellfish are the primary type of seafood that harbor high levels of V vulnificus.5 Oysters are the most common etiology, but consumption of crabs, clams, and shrimp also can lead to infection.5,7Vibrio vulnificus contamination does not change the appearance, taste, or odor of shellfish, making it hard to detect.8 An inoculate of 1 million bacteria typically is necessary for infection after consumption.5 Contaminated seawater is another primary cause of V vulnificus infection. When open wounds are exposed to seawater harboring the bacteria, wound infections can arise.7 Infections can be acquired when swimming, fishing, or participating in water sports. Wound infections also occur while handling contaminated seafood, such as oyster shucking.5 There is a short incubation period for V vulnificus infections; the onset of symptoms and clinical outcome typically occur within 24 hours.5
Clinical Presentation
Vibrio vulnificus infections can have numerous clinical presentations, including gastroenteritis, wound infections, necrotizing fasciitis, and sepsis.1,8 There also is a spectrum of clinical outcomes; for instance, gastroenteritis typically is self-limited, whereas necrotizing fasciitis or sepsis can be fatal.2
Gastroenteritis
Vibrio vulnificus gastroenteritis is due to ingestion of contaminated shellfish.2,9 Symptoms typically are mild to moderate and include nausea, vomiting, diarrhea, fever, chills, abdominal pain, and cramping.2,4,8 Cases likely are underreported in the United States because gastroenteritis is self-limited, and many patients do not seek treatment.2,11
Wound Infections
Wound infections with V vulnificus have a cutaneous port of entry. Exposure to contaminated seawater or seafood can inoculate an open wound, leading to infection.7,8 Wound infections usually stem from 1 of 2 routes: (1) a pre-existing open wound gets infected while the patient is swimming in contaminated water, or (2) a traumatic injury occurs while the patient is handling contaminated shellfish, knives, or fishhooks. Many shellfish, such as oysters, have sharp points on their shells that can lacerate the skin.8 A wound on the hand can be contaminated by V vulnificus while handling contaminated seafood (eg, oyster shucking).13 Minor abrasions should not be dismissed; in fact, a small puncture or skin break often acts as the port of entry.9,11 Wound infections tend to arise within 7 days of exposure, though they can manifest up to 12 days after exposure.8 Wound infections can present as cellulitis, bullae, or ecchymoses.7 Lesions are exquisitely tender, and the skin is erythematous with marked surrounding soft tissue edema.3,4,8 Cellulitis typically arises first, with hemorrhagic bullae rapidly following.14 Lesions are limited to the affected extremity or area of inoculation.8 Systemic symptoms are rare, but fever and chills may accompany the infection.8,14 Unfortunately, lesions can become necrotic and progress rapidly to necrotizing fasciitis if left untreated.4,7,11 In these cases, secondary sepsis can occur.8
Necrotizing Fasciitis
Wound infections caused by V vulnificus can progress to necrotizing skin and soft tissue infections, such as necrotizing fasciitis and gangrene.5 Necrotizing fasciitis accounts for approximately one-third of V vulnificus infections.9 It usually stems from an open wound that is inoculated by contact with contaminated seafood or seawater.2,9 The wound infection begins as cellulitis with extreme tenderness, erythematous skin, and marked soft tissue edema, then rapidly progresses, becoming necrotic. These necrotic lesions present as black and purple eschars as the skin, blood supply, and subcutaneous tissues are infiltrated by the bacteria and destroyed. Lesions may have blistering or exudation. Many patients have accompanying systemic symptoms, including fever, chills, abdominal pain, diarrhea, hypotension, and sepsis.11,14 However, some patients may not present with systemic symptoms, so it is important to maintain a high index of suspicion even in the absence of these symptoms. The infection typically is limited to the affected extremity; necrotizing infections can lead to amputation and even death, depending on the extent of destruction and spread of the bacteria.11,13 The infection may spread beyond the inoculated extremity if the bacteria gains access to the bloodstream.8,9 In these cases, fulminant purpura or secondary septicemia can occur.8,15 Fatalityrates in the United States for necrotizing V vulnificus infections approach 30%.2 Necrotizing fasciitis accounts for approximately 8% of deaths associated with the pathogen in the United States.9
Interestingly, one reported case of necrotizing fasciitis associated with V vulnificus infection was triggered by acupuncture.16 The patient worked in a fish hatchery, where he was exposed to V vulnificus, and subsequent acupuncture led to the inoculation of bacteria into his bloodstream. This case raises the important point that we typically sequence the pathogenesis of V vulnificus infection as a patient having an open wound that is subsequently exposed to contaminated water; however, it also can follow the reverse sequence. Thus, proper cleansing of the skin after swimming in brackish water or handling shellfish is important to prevent V vulnificus infection.16 Additionally, dermatologists should be sure to cleanse patients’ skin thoroughly before performing procedures that could cause breaks in the skin.
Septicemia
Primary septicemia is the most common presentation of V vulnificus infection.2,8 Septicemia accounts for approximately 58% of V vulnificus infections in the United States.9 Infection typically occurs after ingestion of contaminated oysters, with subsequent absorption into the bloodstream through the ileum or cecum.2,8,9 Patients with chronic liver disease are 80 times more likely to develop primary sepsis than healthy individuals.8 Patients typically present with sudden-onset fever and chills, vomiting, diarrhea, and pain in the abdomen and/or extremities within hours to days of ingestion.4,8,9 The median time from ingestion to symptom onset is 18 hours.4,16 However, symptoms can be delayed up to 14 days.2 Progression is rapid; secondary lesions such as bullae, ecchymoses, cellulitis, purpura, macular or maculopapular eruptions, pustules, vasculitis, urticaria, and erythema multiforme–like lesions appear on the extremities within 24 hours of symptom onset. 2,3,4,8,17 Hemorrhagic bullae are the most common cutaneous manifestation of sepsis.4 Lesions are extremely tender to palpation.3 Cutaneous lesions can progress to necrotic ulcers, necrotizing fasciitis, gangrene, necrotizing vasculitis, or myonecrosis.4,8 Evidence of petechiae may indicate progression to disseminated intravascular coagulation (DIC). Elevated D-dimer and fibrin split products also may indicate DIC, and elevated creatine kinase may signify rhabdomyolysis.3 Unfortunately, septicemia has the worst outcomes of all V vulnificus presentations, with morality rates greater than 50% in the United States.1,2,4Vibrio vulnificus septicemia has a similar case-fatality rate to pathogens such as anthrax, Ebola virus disease, and the bubonic plague.5 Septicemia accounts for approximately 80% of the deaths associated with V vulnificus in the United States.8,9
Septicemia due to V vulnificus progresses to septic shock in two-thirds of cases.8 Septic shock presents with hypotension, mental status changes, and thrombocytopenia.2,8,17 Patients can become tachycardic, tachypneic, and hypoxic. Intubation may be required for resuscitation. In cases of septic shock secondary to V vulnificus infection, mortality rates reach 92%.3 Hypotension with a systolic blood pressure less than 90 mm Hg is a poor prognostic factor; patients presenting with hypotension secondary to V vulnificus infection have a fatality rate approaching 75% within 12 hours.2
Atypical Presentations
Rare atypical presentations of V vulnificus infection that have been reported in the literature include meningitis, corneal ulcers, epiglottitis, tonsillitis, spontaneous bacterial peritonitis, pneumonia, endometritis, septic arthritis, osteomyelitis, rhabdomyolysis endophthalmitis, and keratitis.2,4,6,13,18,19
Diagnosis
When diagnosing V vulnificus, providers need to obtain a thorough patient history, including any history of consumption or handling of raw seafood and recent water activities. Providers practicing in tropical climates or in warm summer months should keep V vulnificus in mind, as it is the ideal climate for the pathogen.9 Vital signs can range from unremarkable to fever, hypotension, tachycardia, and/or hypoxia. Skin examination may show exquisitely tender, erythematous skin with marked soft tissue edema, hemorrhagic bullae, ecchymoses, and/or necrosis. As physical examination findings can be nonspecific, wound cultures, blood cultures, and skin biopsies should be taken.
A wound culture and blood culture should be taken immediately if V vulnificus is suspected.8,11 A wound culture using discharge or fluid from necrotic or bullous lesions should be analyzed via gram stain.8,9 Gram stains of V vulnificus show short, slim, curved gram-negative rods under light microscopy.9,20 Special stains also can be done on cultures; V vulnificus is an oxidase-positive, lactose-positive, lysine-positive, salicin-positive, and arginine-negative organism. This knowledge can help differentiate V vulnificus from other gram-negative rods.13 Blood cultures will be positive in approximately 97% of patients with primary septicemia and 30% of patients with septicemia secondary to V vulnificus wound infections.3,9
Histologically, perilesional skin biopsies show epidermal necrosis with dermal and subcutaneous inflammation.12,17 There typically is an inflammatory infiltrate with neutrophilic abscesses and extensive tissue destruction in the subcutaneous tissue extending into the deep dermis.12,17 The superficial dermis is edematous but can lack the inflammatory infiltrate found in the subcutaneous tissue.17 Subepidermal bullae can form with numerous organisms within the fluid of the bullae. There also may be evidence of leukocytoclastic vasculitis with accompanying vessel wall necrosis. Fibrin clot formation and extravasated red blood cells may be visualized with few inflammatory cells but numerous organisms around the involved vessels.17
Management
Early diagnosis and treatment are vital.5,17 Cultures should be taken before aggressive treatment is started.3 Treatment is multifaceted; it requires antibiotics and wound care, except in cases of self-limited gastroenteritis.2,11 Aggressive debridement, fasciotomy, amputation, and supportive measures also may be necessary depending on the patient’s presentation.2,3,8,9 Establishing 2 peripheral intravenous lines is important in case rapid resuscitation becomes necessary.
Antibiotics
Primary cellulitis wound infections should be treated with doxycycline or a quinolone. If untreated, the wound can rapidly progress to necrotizing fasciitis.11 For necrotizing fasciitis and septicemia, broader-spectrum antibiotics are needed. For adults, ceftazidime plus doxycycline is the mainstay of antibiotic treatment for V vulnificus.2,9,11 For children, trimethoprim-sulfamethoxazole plus an aminoglycoside is preferred (Table).2,11
Antibiotic treatment has become more difficult as resistance arises. Antibiotic resistance likely is due to extensive antibiotic use in health care along with the agriculture and aquaculture industries using prophylactic and therapeutic antibiotics that wash into or are directly added to marine waters, where V vulnificus resides. Thus, antibiotic treatment should be tailored to the resistance profile of V vulnificus in various regions; for example, ceftazidime has an intermediate resistance profile in the United States, so cefotaxime and ceftriaxone may be better options.2
Wound Care
Wound infections must be extensively irrigated.9,21 For mild wound infections, proper wound care and oral antibiotics are appropriate (Table).21 Mild wounds should be irrigated thoroughly and followed by wound coverage to prevent progression, secondary infection, and necrosis. The dressing of choice will depend on the presenting lesion and provider preference; nonadherent, occlusive, or wet-to-dry dressings often are the best choices.22 Nonadherent dressings, such as petrolatum-covered gauze, do not pull off the newly formed epithelium when removed, making them beneficial to the skin’s healing process. Another option is occlusive dressings, which maintain a moist environment to hasten healing. They also enhance the autodigestion of necrotic tissue, which can be beneficial for necrotizing V vulnificus infections. Wet-to-dry dressings also may be used; these typically are comprised of gauze soaked with water, an astringent, and an antimicrobial or antiseptic solution. These dressings help to treat acute inflammation and also remove any exudate from the wound.22
Soft tissue and necrotizing infections require debridement.2,8 Early debridement decreases mortality rates.2,8,9 Necrotizing fasciitis often requires serial debridement to clear all the dead tissue and reduce the bacterial burden.8,9 Debridement prevents contiguous spread and metastatic seeding of the bacteria; it is important to prevent spread to the blood vessels, as vasculitis can necrose vessels, preventing antibiotics from reaching the dead tissue.17 Providers also should monitor for compartment syndrome, which should be treated with fasciotomy to decrease mortality.9,23 Many physicians leave V vulnificus–infected wounds open in order to heal by secondary intention.9 Hyperbaric oxygen therapy may be helpful as an adjunct to aggressive antimicrobial treatment for wound healing.8
Supportive Measures
Supportive care for dehydration, sepsis, DIC, and septic shock may be necessary, depending on the patient’s course. Treatment for severe V vulnificus infection includes intravenous fluids, crystalloids, oxygen, and/or intubation. Furthermore, if DIC develops, fresh frozen plasma, cryoprecipitate, a packed red blood cell transfusion, and/or anticoagulation may be required for resuscitation.3
Timing
Time to treatment and fatality rate are directly proportional in V vulnificus infection; the greater the delay in treatment, the higher the fatality rate.2 A 24-hour delay in antibiotic treatment is associated with a 33% case-fatality rate, and a 72-hour delay is associated with a 100% case-fatality rate.9 Even with early, appropriate treatment, mortality rates remain high.4
Prevention
Prevention of V vulnificus infections is an important consideration, especially for patients with chronic liver disease, immunosuppression, and hemochromatosis. Public education about the risks of eating raw shellfish is important.4 Oysters need to be treated properly to prevent growth and survival of V vulnificus.2 The most reliable method for destroying the bacteria is cooking shellfish.8,13 Only 15% of high-risk patients in the United States are aware of the risks associated with raw oyster consumption.3 High-risk patients should avoid eating raw oysters and shellfish and should cook seafood thoroughly before consumption.2,8 They also should wear protective clothing (ie, gloves) and eye protection when handling seafood and protective footwear (ie, wading shoes) while in seawater.2,8,13 It also is important to avoid contact with brackish water if one has any open wounds and to cleanse properly after exposure to brackish water or shellfish.2,8,16 Because severe V vulnificus infections can lead to death, prevention should be strongly encouraged by providers.2
Conclusion
Vibrio vulnificus infection typically occurs due to consumption of contaminated seafood or exposure to contaminated seawater. It most frequently affects older male patients with chronic liver disease, immunosuppression, hemochromatosis, or diabetes mellitus. Vibrio vulnificus can cause a vast spectrum of diseases, including gastroenteritis, wound infections, necrotizing fasciitis, and sepsis. Septicemia is the most common presentation of V vulnificus infection and accounts for the most fatalities from the bacteria. Septicemia often presents with fever, chills, vomiting, diarrhea, and hemorrhagic bullae. Vibrio vulnificus also commonly causes necrotizing fasciitis, which initially presents as cellulitis and rapidly progresses to hemorrhagic bullae or necrosis with accompanying systemic symptoms. Prompt diagnosis and treatment are vital to prevent mortality.
Interestingly, regions impacted by V vulnificus are expanding because of global warming.5,7Vibrio vulnificus thrives in warm waters, and increasing water temperatures are enhancing V vulnificus growth and survival.1,9 As global warming continues, the incidence of V vulnificus infections may rise. In fact, the number of infections increased by 78% between 1996 and 2006 in the United States.5 This rise likely was due to a combination of factors, including an aging population with more comorbidities, improvements in diagnosis, and climate change. Thus, as the number of V vulnificus infections rises, so too must providers’ suspicion for the pathogen.
- Phillips KE, Satchell KJF. Vibrio vulnificus: from oyster colonist to human pathogen [published online January 5, 2017]. PLOS Pathog. doi:10.1371/journal.ppat.1006053
- Heng SP, Letchumanan V, Deng CY, et al. Vibrio vulnificus: an environmental and clinical burden. Front Microbiol. 2017;8:997.
- Kumamoto KS, Vukich DJ. Clinical infections of Vibrio vulnificus: a case report and review of the literature. J Emerg Med. 1998;16:61-66.
- Borenstein M, Kerdel F. Infections with Vibrio vulnificus. Dermatol Clin. 2003;21:245-248.
- Baker-Austin C, Oliver JD. Vibrio vulnificus: new insights into a deadly opportunistic pathogen. Environ Microbiol. 2018;20:423-430.
- Kim SJ, Kim BC, Kim DC, et al. A fatal case of Vibrio vulnificus meningoencephalitis. Clin Microbiol Infect. 2003;9:568-571.
- Jones MK, Oliver JD. Vibrio vulnificus: disease and pathogenesis. Infect Immun. 2009;77:1723-1733.
- Horseman MA, Surani S. A comprehensive review of Vibrio vulnificus infection: an important cause of severe sepsis and skin and soft-tissue infection. Int J Infect Dis. 2011;15:E157-E166.
- Diaz JH. Skin and soft tissue infections following marine injuries and exposures in travelers. J Travel Med. 2014;21:207-213.
- Kikawa K, Yamasaki K, Sukiura T, et al. A successfully treated case of Vibrio vulnificus septicemia with shock. Jpn J Med. 1990;29:313-319.
- Perkins AP, Trimmier M. Recreational waterborne illnesses: recognition, treatment, and prevention. Am Fam Physician. 2017;95:554-560.
- Patel VJ, Gardner E, Burton CS. Vibrio vulnificus septicemia and leg ulcer. J Am Acad Dermatol. 2002;46(5 suppl):S144-S145.
- Ulusarac O, Carter E. Varied clinical presentations of Vibrio vulnificus infections: a report of four unusual cases and review of the literature. South Med J. 2004;97:613-618.
- Bross MH, Soch K, Morales R, et al. Vibrio vulnificus infection: diagnosis and treatment. Am Fam Physician. 2007;76:539-544.
- Hori M, Nakayama A, Kitagawa D, et al. A case of Vibrio vulnificus infection complicated with fulminant purpura: gene and biotype analysis of the pathogen [published online May 19, 2017]. JMM Case Rep. doi:10.1099/jmmcr.0.005096
- Kotton Y, Soboh S, Bisharat N. Vibrio vulnificus necrotizing fasciitis associated with acupuncture. Infect Dis Rep. 2015;7:5901.
- Hoffman TJ, Nelson B, Darouiche R, et al. Vibrio vulnificus septicemia. Arch Intern Med. 1988;148:1825-1827.
- Alsaad AA, Sotello D, Kruse BT, et al. Vibrio vulnificus tonsillitis after swimming in the Gulf of Mexico [published online June 28, 2017]. BMJ Case Rep. doi:10.1136/bcr-2017-221161
- Tison DL, Kelly MT. Vibrio vulnificus endometritis. J Clin Microbiol. 1984;20:185-186.
- Beatty NL, Marquez J, Mohajer MA. Skin manifestations of primary Vibrio vulnificus septicemia. Am J Trop Med Hyg. 2017;97:1-2.
- Foote A, Henderson R, Lindberg A, et al. The Australian mid-west coastal marine wound infections study. Aust Fam Physician. 2017;46:923-927.
- Marks JG Jr, Miller JJ. Lookingbill and Marks’ Principles of Dermatology. 6th ed. Elsevier; 2019.
- Kim CS, Bae EH, Ma SK, et al. Severe septicemia, necrotizing fasciitis, and peritonitis due to Vibrio vulnificus in a patient undergoing continuous ambulatory peritoneal dialysis: a case report. BMC Infect Dis. 2015;15:422.
Vibrio vulnificus is a member of the Vibrio genus. Most Vibrio species are nonpathogenic in humans; however, V vulnificus is one of the pathogenic strains.1 In Latin, the term vulnificus means “wounding,” and V vulnificus can cause life-threatening infections in patients. The mortality rate of V vulnificus infections is approximately 33% in the United States.2Vibrio vulnificus is a gram-negative bacterium that was first isolated by the Centers for Disease Control and Prevention in 1964 and was given its current name in 1979.3-6 It has been found in numerous organisms, including oysters, crabs, clams, shrimp, mussels, mullets, and sea bass.4 The vast majority of infections in the United States are due to oyster exposure and consumption.2,7Vibrio vulnificus is responsible for more than 95% of seafood-related deaths in the United States and has the highest mortality rate of all food-borne illness in the United States.2,5 It also has the highest per-case economic impact of all food-related diseases in the United States.1
What distinguishes a pathogenic vs nonpathogenic Vibrio isolate remains unknown; Vibrio species rapidly undergo horizontal gene transfer, making DNA isolation difficult.1 Some characteristics of V vulnificus that may confer virulence are the capsular polysaccharide, lipopolysaccharide, binding proteins, and tissue-degrading enzymes.1,5 First, encapsulated strains are more virulent and invasive than unencapsulated strains.1 The mucopolysaccharide capsule protects the bacterium from the immune system, allowing it to evade immune surveillance, cause more severe infection, and invade into the subcutaneous tissue.3,5 Second, production of sialic acid–like molecules alter the lipopolysaccharide, allowing for motility and biofilm formation.1 This allows the bacterium to survive in marine waters and within the bloodstream, the latter leading to sepsis in humans. Third, production of N-acetylglucosamine–binding protein A allows for adhesion to chitin. Shellfish consume chitin, and chitin accumulates in shellfish. N-acetylglucosamine–binding protein A also binds mucin; this may be how V vulnificus binds to mucin in the gastrointestinal tract in humans, causing gastroenteritis.1 Binding to the human mucosae also may allow the bacteria to gain access to the blood supply, leading to septicemia.4 Finally, tissue-degrading enzymes such as proteases are responsible for necrotizing wound infections associated with V vulnificus, as the enzymes allow for invasion into the skin and subcutaneous tissues. Proteases also increase vascular permeability and lead to edema.3 Hence, these virulence factors may provide V vulnificus the pathogenicity to cause infection in humans.
Three biotypes of V vulnificus have been discovered. Biotype 1 is the most common and is found worldwide in brackish water.8 It can cause the entire spectrum of illnesses, and it has a case fatality rate of 50% in humans. Biotype 1 is presumably responsible for all infections in the United States. Biotype 2 is found in the Far East and Western Europe; it inhabits a unique niche—saltwater used for eel farming. It typically causes infection in eels, but rarely it can cause wound infections in humans. Biotype 3 is found in freshwater fish farming in Israel, and it is a hybrid of biotypes 1 and 2.It can cause severe soft tissue infections in humans, sometimes requiring amputation.8
Epidemiology
Vibrio vulnificus is a motile, gram-negative, halophilic, aquatic bacterium.1,4,5,8,9 It is part of the normal estuarine microbiome and typically is found in warm coastal waters.1,5,10 The ideal conditions for growth and survival of V vulnificus are water temperatures at 18 °C (64.4 °F) and water salinities between 15 to 25 parts per thousand.2,8,9 These conditions are found in tropical and subtropical regions.2Vibrio vulnificus is found all over the world, including Denmark, Italy, Japan, Australia, Brazil, and the United States,2 where most infections come from oyster exposure and consumption in the Gulf of Mexico.2,8,11 The incidence of infection in the United States is highest between April and October.8,11
Some populations are at a higher risk of infection. Risk factors include male sex, liver cirrhosis, hemochromatosis, end-stage renal disease, immunosuppression, and diabetes mellitus.1,8,11 Healthy patients with no risk factors account for less than 5% of US V vulnificus infections.8
Male Predilection
Men are 6 times more likely to be affected by V vulnificus than women.Some hypotheses for this discrepancy are that estrogen is protective againstV vulnificus and that women may be less likely to engage in risky water activities and seafood handling.5 Additionally, older males (aged >60 years) are most often affected,1,8 likely due to the association between increasing age with number of comorbidities, such as diabetes mellitus, heart disease, and chronic disease.8
Iron Levels
Iron appears to play an important role in V vulnificus infection. Iron is essential for bacterial growth, and the ability to obtain iron from a host increases the organism’s pathogenicity.3Vibrio vulnificus rapidly grows when transferrin saturation exceeds 70%.8 Additionally, iron overload decreases the inoculum needed to cause sepsis in animal studies, which could play a role in human pathogenesis.4 Iron levels are elevated in patients with hemochromatosis due to increased iron absorption, cirrhosis and chronic liver disease due to impaired iron metabolism, and end-stage renal disease, especially in patients receiving parenteral iron.8
Immunosuppression
Patients who are immunocompromised and those with chronic liver disease are at an increased risk of infection because of neutrophils having decreased phagocytic activity.4
Diabetes Mellitus
Patients with diabetes mellitus may have peripheral neuropathy and may be unaware of pre-existing wounds that serve as entry points for V vulnificus.12
Etiology
Vibrio vulnificus infects humans via seafood consumption and handling as well as exposure to contaminated water.2,5 With respect to seafood consumption, raw shellfish are the primary type of seafood that harbor high levels of V vulnificus.5 Oysters are the most common etiology, but consumption of crabs, clams, and shrimp also can lead to infection.5,7Vibrio vulnificus contamination does not change the appearance, taste, or odor of shellfish, making it hard to detect.8 An inoculate of 1 million bacteria typically is necessary for infection after consumption.5 Contaminated seawater is another primary cause of V vulnificus infection. When open wounds are exposed to seawater harboring the bacteria, wound infections can arise.7 Infections can be acquired when swimming, fishing, or participating in water sports. Wound infections also occur while handling contaminated seafood, such as oyster shucking.5 There is a short incubation period for V vulnificus infections; the onset of symptoms and clinical outcome typically occur within 24 hours.5
Clinical Presentation
Vibrio vulnificus infections can have numerous clinical presentations, including gastroenteritis, wound infections, necrotizing fasciitis, and sepsis.1,8 There also is a spectrum of clinical outcomes; for instance, gastroenteritis typically is self-limited, whereas necrotizing fasciitis or sepsis can be fatal.2
Gastroenteritis
Vibrio vulnificus gastroenteritis is due to ingestion of contaminated shellfish.2,9 Symptoms typically are mild to moderate and include nausea, vomiting, diarrhea, fever, chills, abdominal pain, and cramping.2,4,8 Cases likely are underreported in the United States because gastroenteritis is self-limited, and many patients do not seek treatment.2,11
Wound Infections
Wound infections with V vulnificus have a cutaneous port of entry. Exposure to contaminated seawater or seafood can inoculate an open wound, leading to infection.7,8 Wound infections usually stem from 1 of 2 routes: (1) a pre-existing open wound gets infected while the patient is swimming in contaminated water, or (2) a traumatic injury occurs while the patient is handling contaminated shellfish, knives, or fishhooks. Many shellfish, such as oysters, have sharp points on their shells that can lacerate the skin.8 A wound on the hand can be contaminated by V vulnificus while handling contaminated seafood (eg, oyster shucking).13 Minor abrasions should not be dismissed; in fact, a small puncture or skin break often acts as the port of entry.9,11 Wound infections tend to arise within 7 days of exposure, though they can manifest up to 12 days after exposure.8 Wound infections can present as cellulitis, bullae, or ecchymoses.7 Lesions are exquisitely tender, and the skin is erythematous with marked surrounding soft tissue edema.3,4,8 Cellulitis typically arises first, with hemorrhagic bullae rapidly following.14 Lesions are limited to the affected extremity or area of inoculation.8 Systemic symptoms are rare, but fever and chills may accompany the infection.8,14 Unfortunately, lesions can become necrotic and progress rapidly to necrotizing fasciitis if left untreated.4,7,11 In these cases, secondary sepsis can occur.8
Necrotizing Fasciitis
Wound infections caused by V vulnificus can progress to necrotizing skin and soft tissue infections, such as necrotizing fasciitis and gangrene.5 Necrotizing fasciitis accounts for approximately one-third of V vulnificus infections.9 It usually stems from an open wound that is inoculated by contact with contaminated seafood or seawater.2,9 The wound infection begins as cellulitis with extreme tenderness, erythematous skin, and marked soft tissue edema, then rapidly progresses, becoming necrotic. These necrotic lesions present as black and purple eschars as the skin, blood supply, and subcutaneous tissues are infiltrated by the bacteria and destroyed. Lesions may have blistering or exudation. Many patients have accompanying systemic symptoms, including fever, chills, abdominal pain, diarrhea, hypotension, and sepsis.11,14 However, some patients may not present with systemic symptoms, so it is important to maintain a high index of suspicion even in the absence of these symptoms. The infection typically is limited to the affected extremity; necrotizing infections can lead to amputation and even death, depending on the extent of destruction and spread of the bacteria.11,13 The infection may spread beyond the inoculated extremity if the bacteria gains access to the bloodstream.8,9 In these cases, fulminant purpura or secondary septicemia can occur.8,15 Fatalityrates in the United States for necrotizing V vulnificus infections approach 30%.2 Necrotizing fasciitis accounts for approximately 8% of deaths associated with the pathogen in the United States.9
Interestingly, one reported case of necrotizing fasciitis associated with V vulnificus infection was triggered by acupuncture.16 The patient worked in a fish hatchery, where he was exposed to V vulnificus, and subsequent acupuncture led to the inoculation of bacteria into his bloodstream. This case raises the important point that we typically sequence the pathogenesis of V vulnificus infection as a patient having an open wound that is subsequently exposed to contaminated water; however, it also can follow the reverse sequence. Thus, proper cleansing of the skin after swimming in brackish water or handling shellfish is important to prevent V vulnificus infection.16 Additionally, dermatologists should be sure to cleanse patients’ skin thoroughly before performing procedures that could cause breaks in the skin.
Septicemia
Primary septicemia is the most common presentation of V vulnificus infection.2,8 Septicemia accounts for approximately 58% of V vulnificus infections in the United States.9 Infection typically occurs after ingestion of contaminated oysters, with subsequent absorption into the bloodstream through the ileum or cecum.2,8,9 Patients with chronic liver disease are 80 times more likely to develop primary sepsis than healthy individuals.8 Patients typically present with sudden-onset fever and chills, vomiting, diarrhea, and pain in the abdomen and/or extremities within hours to days of ingestion.4,8,9 The median time from ingestion to symptom onset is 18 hours.4,16 However, symptoms can be delayed up to 14 days.2 Progression is rapid; secondary lesions such as bullae, ecchymoses, cellulitis, purpura, macular or maculopapular eruptions, pustules, vasculitis, urticaria, and erythema multiforme–like lesions appear on the extremities within 24 hours of symptom onset. 2,3,4,8,17 Hemorrhagic bullae are the most common cutaneous manifestation of sepsis.4 Lesions are extremely tender to palpation.3 Cutaneous lesions can progress to necrotic ulcers, necrotizing fasciitis, gangrene, necrotizing vasculitis, or myonecrosis.4,8 Evidence of petechiae may indicate progression to disseminated intravascular coagulation (DIC). Elevated D-dimer and fibrin split products also may indicate DIC, and elevated creatine kinase may signify rhabdomyolysis.3 Unfortunately, septicemia has the worst outcomes of all V vulnificus presentations, with morality rates greater than 50% in the United States.1,2,4Vibrio vulnificus septicemia has a similar case-fatality rate to pathogens such as anthrax, Ebola virus disease, and the bubonic plague.5 Septicemia accounts for approximately 80% of the deaths associated with V vulnificus in the United States.8,9
Septicemia due to V vulnificus progresses to septic shock in two-thirds of cases.8 Septic shock presents with hypotension, mental status changes, and thrombocytopenia.2,8,17 Patients can become tachycardic, tachypneic, and hypoxic. Intubation may be required for resuscitation. In cases of septic shock secondary to V vulnificus infection, mortality rates reach 92%.3 Hypotension with a systolic blood pressure less than 90 mm Hg is a poor prognostic factor; patients presenting with hypotension secondary to V vulnificus infection have a fatality rate approaching 75% within 12 hours.2
Atypical Presentations
Rare atypical presentations of V vulnificus infection that have been reported in the literature include meningitis, corneal ulcers, epiglottitis, tonsillitis, spontaneous bacterial peritonitis, pneumonia, endometritis, septic arthritis, osteomyelitis, rhabdomyolysis endophthalmitis, and keratitis.2,4,6,13,18,19
Diagnosis
When diagnosing V vulnificus, providers need to obtain a thorough patient history, including any history of consumption or handling of raw seafood and recent water activities. Providers practicing in tropical climates or in warm summer months should keep V vulnificus in mind, as it is the ideal climate for the pathogen.9 Vital signs can range from unremarkable to fever, hypotension, tachycardia, and/or hypoxia. Skin examination may show exquisitely tender, erythematous skin with marked soft tissue edema, hemorrhagic bullae, ecchymoses, and/or necrosis. As physical examination findings can be nonspecific, wound cultures, blood cultures, and skin biopsies should be taken.
A wound culture and blood culture should be taken immediately if V vulnificus is suspected.8,11 A wound culture using discharge or fluid from necrotic or bullous lesions should be analyzed via gram stain.8,9 Gram stains of V vulnificus show short, slim, curved gram-negative rods under light microscopy.9,20 Special stains also can be done on cultures; V vulnificus is an oxidase-positive, lactose-positive, lysine-positive, salicin-positive, and arginine-negative organism. This knowledge can help differentiate V vulnificus from other gram-negative rods.13 Blood cultures will be positive in approximately 97% of patients with primary septicemia and 30% of patients with septicemia secondary to V vulnificus wound infections.3,9
Histologically, perilesional skin biopsies show epidermal necrosis with dermal and subcutaneous inflammation.12,17 There typically is an inflammatory infiltrate with neutrophilic abscesses and extensive tissue destruction in the subcutaneous tissue extending into the deep dermis.12,17 The superficial dermis is edematous but can lack the inflammatory infiltrate found in the subcutaneous tissue.17 Subepidermal bullae can form with numerous organisms within the fluid of the bullae. There also may be evidence of leukocytoclastic vasculitis with accompanying vessel wall necrosis. Fibrin clot formation and extravasated red blood cells may be visualized with few inflammatory cells but numerous organisms around the involved vessels.17
Management
Early diagnosis and treatment are vital.5,17 Cultures should be taken before aggressive treatment is started.3 Treatment is multifaceted; it requires antibiotics and wound care, except in cases of self-limited gastroenteritis.2,11 Aggressive debridement, fasciotomy, amputation, and supportive measures also may be necessary depending on the patient’s presentation.2,3,8,9 Establishing 2 peripheral intravenous lines is important in case rapid resuscitation becomes necessary.
Antibiotics
Primary cellulitis wound infections should be treated with doxycycline or a quinolone. If untreated, the wound can rapidly progress to necrotizing fasciitis.11 For necrotizing fasciitis and septicemia, broader-spectrum antibiotics are needed. For adults, ceftazidime plus doxycycline is the mainstay of antibiotic treatment for V vulnificus.2,9,11 For children, trimethoprim-sulfamethoxazole plus an aminoglycoside is preferred (Table).2,11
Antibiotic treatment has become more difficult as resistance arises. Antibiotic resistance likely is due to extensive antibiotic use in health care along with the agriculture and aquaculture industries using prophylactic and therapeutic antibiotics that wash into or are directly added to marine waters, where V vulnificus resides. Thus, antibiotic treatment should be tailored to the resistance profile of V vulnificus in various regions; for example, ceftazidime has an intermediate resistance profile in the United States, so cefotaxime and ceftriaxone may be better options.2
Wound Care
Wound infections must be extensively irrigated.9,21 For mild wound infections, proper wound care and oral antibiotics are appropriate (Table).21 Mild wounds should be irrigated thoroughly and followed by wound coverage to prevent progression, secondary infection, and necrosis. The dressing of choice will depend on the presenting lesion and provider preference; nonadherent, occlusive, or wet-to-dry dressings often are the best choices.22 Nonadherent dressings, such as petrolatum-covered gauze, do not pull off the newly formed epithelium when removed, making them beneficial to the skin’s healing process. Another option is occlusive dressings, which maintain a moist environment to hasten healing. They also enhance the autodigestion of necrotic tissue, which can be beneficial for necrotizing V vulnificus infections. Wet-to-dry dressings also may be used; these typically are comprised of gauze soaked with water, an astringent, and an antimicrobial or antiseptic solution. These dressings help to treat acute inflammation and also remove any exudate from the wound.22
Soft tissue and necrotizing infections require debridement.2,8 Early debridement decreases mortality rates.2,8,9 Necrotizing fasciitis often requires serial debridement to clear all the dead tissue and reduce the bacterial burden.8,9 Debridement prevents contiguous spread and metastatic seeding of the bacteria; it is important to prevent spread to the blood vessels, as vasculitis can necrose vessels, preventing antibiotics from reaching the dead tissue.17 Providers also should monitor for compartment syndrome, which should be treated with fasciotomy to decrease mortality.9,23 Many physicians leave V vulnificus–infected wounds open in order to heal by secondary intention.9 Hyperbaric oxygen therapy may be helpful as an adjunct to aggressive antimicrobial treatment for wound healing.8
Supportive Measures
Supportive care for dehydration, sepsis, DIC, and septic shock may be necessary, depending on the patient’s course. Treatment for severe V vulnificus infection includes intravenous fluids, crystalloids, oxygen, and/or intubation. Furthermore, if DIC develops, fresh frozen plasma, cryoprecipitate, a packed red blood cell transfusion, and/or anticoagulation may be required for resuscitation.3
Timing
Time to treatment and fatality rate are directly proportional in V vulnificus infection; the greater the delay in treatment, the higher the fatality rate.2 A 24-hour delay in antibiotic treatment is associated with a 33% case-fatality rate, and a 72-hour delay is associated with a 100% case-fatality rate.9 Even with early, appropriate treatment, mortality rates remain high.4
Prevention
Prevention of V vulnificus infections is an important consideration, especially for patients with chronic liver disease, immunosuppression, and hemochromatosis. Public education about the risks of eating raw shellfish is important.4 Oysters need to be treated properly to prevent growth and survival of V vulnificus.2 The most reliable method for destroying the bacteria is cooking shellfish.8,13 Only 15% of high-risk patients in the United States are aware of the risks associated with raw oyster consumption.3 High-risk patients should avoid eating raw oysters and shellfish and should cook seafood thoroughly before consumption.2,8 They also should wear protective clothing (ie, gloves) and eye protection when handling seafood and protective footwear (ie, wading shoes) while in seawater.2,8,13 It also is important to avoid contact with brackish water if one has any open wounds and to cleanse properly after exposure to brackish water or shellfish.2,8,16 Because severe V vulnificus infections can lead to death, prevention should be strongly encouraged by providers.2
Conclusion
Vibrio vulnificus infection typically occurs due to consumption of contaminated seafood or exposure to contaminated seawater. It most frequently affects older male patients with chronic liver disease, immunosuppression, hemochromatosis, or diabetes mellitus. Vibrio vulnificus can cause a vast spectrum of diseases, including gastroenteritis, wound infections, necrotizing fasciitis, and sepsis. Septicemia is the most common presentation of V vulnificus infection and accounts for the most fatalities from the bacteria. Septicemia often presents with fever, chills, vomiting, diarrhea, and hemorrhagic bullae. Vibrio vulnificus also commonly causes necrotizing fasciitis, which initially presents as cellulitis and rapidly progresses to hemorrhagic bullae or necrosis with accompanying systemic symptoms. Prompt diagnosis and treatment are vital to prevent mortality.
Interestingly, regions impacted by V vulnificus are expanding because of global warming.5,7Vibrio vulnificus thrives in warm waters, and increasing water temperatures are enhancing V vulnificus growth and survival.1,9 As global warming continues, the incidence of V vulnificus infections may rise. In fact, the number of infections increased by 78% between 1996 and 2006 in the United States.5 This rise likely was due to a combination of factors, including an aging population with more comorbidities, improvements in diagnosis, and climate change. Thus, as the number of V vulnificus infections rises, so too must providers’ suspicion for the pathogen.
Vibrio vulnificus is a member of the Vibrio genus. Most Vibrio species are nonpathogenic in humans; however, V vulnificus is one of the pathogenic strains.1 In Latin, the term vulnificus means “wounding,” and V vulnificus can cause life-threatening infections in patients. The mortality rate of V vulnificus infections is approximately 33% in the United States.2Vibrio vulnificus is a gram-negative bacterium that was first isolated by the Centers for Disease Control and Prevention in 1964 and was given its current name in 1979.3-6 It has been found in numerous organisms, including oysters, crabs, clams, shrimp, mussels, mullets, and sea bass.4 The vast majority of infections in the United States are due to oyster exposure and consumption.2,7Vibrio vulnificus is responsible for more than 95% of seafood-related deaths in the United States and has the highest mortality rate of all food-borne illness in the United States.2,5 It also has the highest per-case economic impact of all food-related diseases in the United States.1
What distinguishes a pathogenic vs nonpathogenic Vibrio isolate remains unknown; Vibrio species rapidly undergo horizontal gene transfer, making DNA isolation difficult.1 Some characteristics of V vulnificus that may confer virulence are the capsular polysaccharide, lipopolysaccharide, binding proteins, and tissue-degrading enzymes.1,5 First, encapsulated strains are more virulent and invasive than unencapsulated strains.1 The mucopolysaccharide capsule protects the bacterium from the immune system, allowing it to evade immune surveillance, cause more severe infection, and invade into the subcutaneous tissue.3,5 Second, production of sialic acid–like molecules alter the lipopolysaccharide, allowing for motility and biofilm formation.1 This allows the bacterium to survive in marine waters and within the bloodstream, the latter leading to sepsis in humans. Third, production of N-acetylglucosamine–binding protein A allows for adhesion to chitin. Shellfish consume chitin, and chitin accumulates in shellfish. N-acetylglucosamine–binding protein A also binds mucin; this may be how V vulnificus binds to mucin in the gastrointestinal tract in humans, causing gastroenteritis.1 Binding to the human mucosae also may allow the bacteria to gain access to the blood supply, leading to septicemia.4 Finally, tissue-degrading enzymes such as proteases are responsible for necrotizing wound infections associated with V vulnificus, as the enzymes allow for invasion into the skin and subcutaneous tissues. Proteases also increase vascular permeability and lead to edema.3 Hence, these virulence factors may provide V vulnificus the pathogenicity to cause infection in humans.
Three biotypes of V vulnificus have been discovered. Biotype 1 is the most common and is found worldwide in brackish water.8 It can cause the entire spectrum of illnesses, and it has a case fatality rate of 50% in humans. Biotype 1 is presumably responsible for all infections in the United States. Biotype 2 is found in the Far East and Western Europe; it inhabits a unique niche—saltwater used for eel farming. It typically causes infection in eels, but rarely it can cause wound infections in humans. Biotype 3 is found in freshwater fish farming in Israel, and it is a hybrid of biotypes 1 and 2.It can cause severe soft tissue infections in humans, sometimes requiring amputation.8
Epidemiology
Vibrio vulnificus is a motile, gram-negative, halophilic, aquatic bacterium.1,4,5,8,9 It is part of the normal estuarine microbiome and typically is found in warm coastal waters.1,5,10 The ideal conditions for growth and survival of V vulnificus are water temperatures at 18 °C (64.4 °F) and water salinities between 15 to 25 parts per thousand.2,8,9 These conditions are found in tropical and subtropical regions.2Vibrio vulnificus is found all over the world, including Denmark, Italy, Japan, Australia, Brazil, and the United States,2 where most infections come from oyster exposure and consumption in the Gulf of Mexico.2,8,11 The incidence of infection in the United States is highest between April and October.8,11
Some populations are at a higher risk of infection. Risk factors include male sex, liver cirrhosis, hemochromatosis, end-stage renal disease, immunosuppression, and diabetes mellitus.1,8,11 Healthy patients with no risk factors account for less than 5% of US V vulnificus infections.8
Male Predilection
Men are 6 times more likely to be affected by V vulnificus than women.Some hypotheses for this discrepancy are that estrogen is protective againstV vulnificus and that women may be less likely to engage in risky water activities and seafood handling.5 Additionally, older males (aged >60 years) are most often affected,1,8 likely due to the association between increasing age with number of comorbidities, such as diabetes mellitus, heart disease, and chronic disease.8
Iron Levels
Iron appears to play an important role in V vulnificus infection. Iron is essential for bacterial growth, and the ability to obtain iron from a host increases the organism’s pathogenicity.3Vibrio vulnificus rapidly grows when transferrin saturation exceeds 70%.8 Additionally, iron overload decreases the inoculum needed to cause sepsis in animal studies, which could play a role in human pathogenesis.4 Iron levels are elevated in patients with hemochromatosis due to increased iron absorption, cirrhosis and chronic liver disease due to impaired iron metabolism, and end-stage renal disease, especially in patients receiving parenteral iron.8
Immunosuppression
Patients who are immunocompromised and those with chronic liver disease are at an increased risk of infection because of neutrophils having decreased phagocytic activity.4
Diabetes Mellitus
Patients with diabetes mellitus may have peripheral neuropathy and may be unaware of pre-existing wounds that serve as entry points for V vulnificus.12
Etiology
Vibrio vulnificus infects humans via seafood consumption and handling as well as exposure to contaminated water.2,5 With respect to seafood consumption, raw shellfish are the primary type of seafood that harbor high levels of V vulnificus.5 Oysters are the most common etiology, but consumption of crabs, clams, and shrimp also can lead to infection.5,7Vibrio vulnificus contamination does not change the appearance, taste, or odor of shellfish, making it hard to detect.8 An inoculate of 1 million bacteria typically is necessary for infection after consumption.5 Contaminated seawater is another primary cause of V vulnificus infection. When open wounds are exposed to seawater harboring the bacteria, wound infections can arise.7 Infections can be acquired when swimming, fishing, or participating in water sports. Wound infections also occur while handling contaminated seafood, such as oyster shucking.5 There is a short incubation period for V vulnificus infections; the onset of symptoms and clinical outcome typically occur within 24 hours.5
Clinical Presentation
Vibrio vulnificus infections can have numerous clinical presentations, including gastroenteritis, wound infections, necrotizing fasciitis, and sepsis.1,8 There also is a spectrum of clinical outcomes; for instance, gastroenteritis typically is self-limited, whereas necrotizing fasciitis or sepsis can be fatal.2
Gastroenteritis
Vibrio vulnificus gastroenteritis is due to ingestion of contaminated shellfish.2,9 Symptoms typically are mild to moderate and include nausea, vomiting, diarrhea, fever, chills, abdominal pain, and cramping.2,4,8 Cases likely are underreported in the United States because gastroenteritis is self-limited, and many patients do not seek treatment.2,11
Wound Infections
Wound infections with V vulnificus have a cutaneous port of entry. Exposure to contaminated seawater or seafood can inoculate an open wound, leading to infection.7,8 Wound infections usually stem from 1 of 2 routes: (1) a pre-existing open wound gets infected while the patient is swimming in contaminated water, or (2) a traumatic injury occurs while the patient is handling contaminated shellfish, knives, or fishhooks. Many shellfish, such as oysters, have sharp points on their shells that can lacerate the skin.8 A wound on the hand can be contaminated by V vulnificus while handling contaminated seafood (eg, oyster shucking).13 Minor abrasions should not be dismissed; in fact, a small puncture or skin break often acts as the port of entry.9,11 Wound infections tend to arise within 7 days of exposure, though they can manifest up to 12 days after exposure.8 Wound infections can present as cellulitis, bullae, or ecchymoses.7 Lesions are exquisitely tender, and the skin is erythematous with marked surrounding soft tissue edema.3,4,8 Cellulitis typically arises first, with hemorrhagic bullae rapidly following.14 Lesions are limited to the affected extremity or area of inoculation.8 Systemic symptoms are rare, but fever and chills may accompany the infection.8,14 Unfortunately, lesions can become necrotic and progress rapidly to necrotizing fasciitis if left untreated.4,7,11 In these cases, secondary sepsis can occur.8
Necrotizing Fasciitis
Wound infections caused by V vulnificus can progress to necrotizing skin and soft tissue infections, such as necrotizing fasciitis and gangrene.5 Necrotizing fasciitis accounts for approximately one-third of V vulnificus infections.9 It usually stems from an open wound that is inoculated by contact with contaminated seafood or seawater.2,9 The wound infection begins as cellulitis with extreme tenderness, erythematous skin, and marked soft tissue edema, then rapidly progresses, becoming necrotic. These necrotic lesions present as black and purple eschars as the skin, blood supply, and subcutaneous tissues are infiltrated by the bacteria and destroyed. Lesions may have blistering or exudation. Many patients have accompanying systemic symptoms, including fever, chills, abdominal pain, diarrhea, hypotension, and sepsis.11,14 However, some patients may not present with systemic symptoms, so it is important to maintain a high index of suspicion even in the absence of these symptoms. The infection typically is limited to the affected extremity; necrotizing infections can lead to amputation and even death, depending on the extent of destruction and spread of the bacteria.11,13 The infection may spread beyond the inoculated extremity if the bacteria gains access to the bloodstream.8,9 In these cases, fulminant purpura or secondary septicemia can occur.8,15 Fatalityrates in the United States for necrotizing V vulnificus infections approach 30%.2 Necrotizing fasciitis accounts for approximately 8% of deaths associated with the pathogen in the United States.9
Interestingly, one reported case of necrotizing fasciitis associated with V vulnificus infection was triggered by acupuncture.16 The patient worked in a fish hatchery, where he was exposed to V vulnificus, and subsequent acupuncture led to the inoculation of bacteria into his bloodstream. This case raises the important point that we typically sequence the pathogenesis of V vulnificus infection as a patient having an open wound that is subsequently exposed to contaminated water; however, it also can follow the reverse sequence. Thus, proper cleansing of the skin after swimming in brackish water or handling shellfish is important to prevent V vulnificus infection.16 Additionally, dermatologists should be sure to cleanse patients’ skin thoroughly before performing procedures that could cause breaks in the skin.
Septicemia
Primary septicemia is the most common presentation of V vulnificus infection.2,8 Septicemia accounts for approximately 58% of V vulnificus infections in the United States.9 Infection typically occurs after ingestion of contaminated oysters, with subsequent absorption into the bloodstream through the ileum or cecum.2,8,9 Patients with chronic liver disease are 80 times more likely to develop primary sepsis than healthy individuals.8 Patients typically present with sudden-onset fever and chills, vomiting, diarrhea, and pain in the abdomen and/or extremities within hours to days of ingestion.4,8,9 The median time from ingestion to symptom onset is 18 hours.4,16 However, symptoms can be delayed up to 14 days.2 Progression is rapid; secondary lesions such as bullae, ecchymoses, cellulitis, purpura, macular or maculopapular eruptions, pustules, vasculitis, urticaria, and erythema multiforme–like lesions appear on the extremities within 24 hours of symptom onset. 2,3,4,8,17 Hemorrhagic bullae are the most common cutaneous manifestation of sepsis.4 Lesions are extremely tender to palpation.3 Cutaneous lesions can progress to necrotic ulcers, necrotizing fasciitis, gangrene, necrotizing vasculitis, or myonecrosis.4,8 Evidence of petechiae may indicate progression to disseminated intravascular coagulation (DIC). Elevated D-dimer and fibrin split products also may indicate DIC, and elevated creatine kinase may signify rhabdomyolysis.3 Unfortunately, septicemia has the worst outcomes of all V vulnificus presentations, with morality rates greater than 50% in the United States.1,2,4Vibrio vulnificus septicemia has a similar case-fatality rate to pathogens such as anthrax, Ebola virus disease, and the bubonic plague.5 Septicemia accounts for approximately 80% of the deaths associated with V vulnificus in the United States.8,9
Septicemia due to V vulnificus progresses to septic shock in two-thirds of cases.8 Septic shock presents with hypotension, mental status changes, and thrombocytopenia.2,8,17 Patients can become tachycardic, tachypneic, and hypoxic. Intubation may be required for resuscitation. In cases of septic shock secondary to V vulnificus infection, mortality rates reach 92%.3 Hypotension with a systolic blood pressure less than 90 mm Hg is a poor prognostic factor; patients presenting with hypotension secondary to V vulnificus infection have a fatality rate approaching 75% within 12 hours.2
Atypical Presentations
Rare atypical presentations of V vulnificus infection that have been reported in the literature include meningitis, corneal ulcers, epiglottitis, tonsillitis, spontaneous bacterial peritonitis, pneumonia, endometritis, septic arthritis, osteomyelitis, rhabdomyolysis endophthalmitis, and keratitis.2,4,6,13,18,19
Diagnosis
When diagnosing V vulnificus, providers need to obtain a thorough patient history, including any history of consumption or handling of raw seafood and recent water activities. Providers practicing in tropical climates or in warm summer months should keep V vulnificus in mind, as it is the ideal climate for the pathogen.9 Vital signs can range from unremarkable to fever, hypotension, tachycardia, and/or hypoxia. Skin examination may show exquisitely tender, erythematous skin with marked soft tissue edema, hemorrhagic bullae, ecchymoses, and/or necrosis. As physical examination findings can be nonspecific, wound cultures, blood cultures, and skin biopsies should be taken.
A wound culture and blood culture should be taken immediately if V vulnificus is suspected.8,11 A wound culture using discharge or fluid from necrotic or bullous lesions should be analyzed via gram stain.8,9 Gram stains of V vulnificus show short, slim, curved gram-negative rods under light microscopy.9,20 Special stains also can be done on cultures; V vulnificus is an oxidase-positive, lactose-positive, lysine-positive, salicin-positive, and arginine-negative organism. This knowledge can help differentiate V vulnificus from other gram-negative rods.13 Blood cultures will be positive in approximately 97% of patients with primary septicemia and 30% of patients with septicemia secondary to V vulnificus wound infections.3,9
Histologically, perilesional skin biopsies show epidermal necrosis with dermal and subcutaneous inflammation.12,17 There typically is an inflammatory infiltrate with neutrophilic abscesses and extensive tissue destruction in the subcutaneous tissue extending into the deep dermis.12,17 The superficial dermis is edematous but can lack the inflammatory infiltrate found in the subcutaneous tissue.17 Subepidermal bullae can form with numerous organisms within the fluid of the bullae. There also may be evidence of leukocytoclastic vasculitis with accompanying vessel wall necrosis. Fibrin clot formation and extravasated red blood cells may be visualized with few inflammatory cells but numerous organisms around the involved vessels.17
Management
Early diagnosis and treatment are vital.5,17 Cultures should be taken before aggressive treatment is started.3 Treatment is multifaceted; it requires antibiotics and wound care, except in cases of self-limited gastroenteritis.2,11 Aggressive debridement, fasciotomy, amputation, and supportive measures also may be necessary depending on the patient’s presentation.2,3,8,9 Establishing 2 peripheral intravenous lines is important in case rapid resuscitation becomes necessary.
Antibiotics
Primary cellulitis wound infections should be treated with doxycycline or a quinolone. If untreated, the wound can rapidly progress to necrotizing fasciitis.11 For necrotizing fasciitis and septicemia, broader-spectrum antibiotics are needed. For adults, ceftazidime plus doxycycline is the mainstay of antibiotic treatment for V vulnificus.2,9,11 For children, trimethoprim-sulfamethoxazole plus an aminoglycoside is preferred (Table).2,11
Antibiotic treatment has become more difficult as resistance arises. Antibiotic resistance likely is due to extensive antibiotic use in health care along with the agriculture and aquaculture industries using prophylactic and therapeutic antibiotics that wash into or are directly added to marine waters, where V vulnificus resides. Thus, antibiotic treatment should be tailored to the resistance profile of V vulnificus in various regions; for example, ceftazidime has an intermediate resistance profile in the United States, so cefotaxime and ceftriaxone may be better options.2
Wound Care
Wound infections must be extensively irrigated.9,21 For mild wound infections, proper wound care and oral antibiotics are appropriate (Table).21 Mild wounds should be irrigated thoroughly and followed by wound coverage to prevent progression, secondary infection, and necrosis. The dressing of choice will depend on the presenting lesion and provider preference; nonadherent, occlusive, or wet-to-dry dressings often are the best choices.22 Nonadherent dressings, such as petrolatum-covered gauze, do not pull off the newly formed epithelium when removed, making them beneficial to the skin’s healing process. Another option is occlusive dressings, which maintain a moist environment to hasten healing. They also enhance the autodigestion of necrotic tissue, which can be beneficial for necrotizing V vulnificus infections. Wet-to-dry dressings also may be used; these typically are comprised of gauze soaked with water, an astringent, and an antimicrobial or antiseptic solution. These dressings help to treat acute inflammation and also remove any exudate from the wound.22
Soft tissue and necrotizing infections require debridement.2,8 Early debridement decreases mortality rates.2,8,9 Necrotizing fasciitis often requires serial debridement to clear all the dead tissue and reduce the bacterial burden.8,9 Debridement prevents contiguous spread and metastatic seeding of the bacteria; it is important to prevent spread to the blood vessels, as vasculitis can necrose vessels, preventing antibiotics from reaching the dead tissue.17 Providers also should monitor for compartment syndrome, which should be treated with fasciotomy to decrease mortality.9,23 Many physicians leave V vulnificus–infected wounds open in order to heal by secondary intention.9 Hyperbaric oxygen therapy may be helpful as an adjunct to aggressive antimicrobial treatment for wound healing.8
Supportive Measures
Supportive care for dehydration, sepsis, DIC, and septic shock may be necessary, depending on the patient’s course. Treatment for severe V vulnificus infection includes intravenous fluids, crystalloids, oxygen, and/or intubation. Furthermore, if DIC develops, fresh frozen plasma, cryoprecipitate, a packed red blood cell transfusion, and/or anticoagulation may be required for resuscitation.3
Timing
Time to treatment and fatality rate are directly proportional in V vulnificus infection; the greater the delay in treatment, the higher the fatality rate.2 A 24-hour delay in antibiotic treatment is associated with a 33% case-fatality rate, and a 72-hour delay is associated with a 100% case-fatality rate.9 Even with early, appropriate treatment, mortality rates remain high.4
Prevention
Prevention of V vulnificus infections is an important consideration, especially for patients with chronic liver disease, immunosuppression, and hemochromatosis. Public education about the risks of eating raw shellfish is important.4 Oysters need to be treated properly to prevent growth and survival of V vulnificus.2 The most reliable method for destroying the bacteria is cooking shellfish.8,13 Only 15% of high-risk patients in the United States are aware of the risks associated with raw oyster consumption.3 High-risk patients should avoid eating raw oysters and shellfish and should cook seafood thoroughly before consumption.2,8 They also should wear protective clothing (ie, gloves) and eye protection when handling seafood and protective footwear (ie, wading shoes) while in seawater.2,8,13 It also is important to avoid contact with brackish water if one has any open wounds and to cleanse properly after exposure to brackish water or shellfish.2,8,16 Because severe V vulnificus infections can lead to death, prevention should be strongly encouraged by providers.2
Conclusion
Vibrio vulnificus infection typically occurs due to consumption of contaminated seafood or exposure to contaminated seawater. It most frequently affects older male patients with chronic liver disease, immunosuppression, hemochromatosis, or diabetes mellitus. Vibrio vulnificus can cause a vast spectrum of diseases, including gastroenteritis, wound infections, necrotizing fasciitis, and sepsis. Septicemia is the most common presentation of V vulnificus infection and accounts for the most fatalities from the bacteria. Septicemia often presents with fever, chills, vomiting, diarrhea, and hemorrhagic bullae. Vibrio vulnificus also commonly causes necrotizing fasciitis, which initially presents as cellulitis and rapidly progresses to hemorrhagic bullae or necrosis with accompanying systemic symptoms. Prompt diagnosis and treatment are vital to prevent mortality.
Interestingly, regions impacted by V vulnificus are expanding because of global warming.5,7Vibrio vulnificus thrives in warm waters, and increasing water temperatures are enhancing V vulnificus growth and survival.1,9 As global warming continues, the incidence of V vulnificus infections may rise. In fact, the number of infections increased by 78% between 1996 and 2006 in the United States.5 This rise likely was due to a combination of factors, including an aging population with more comorbidities, improvements in diagnosis, and climate change. Thus, as the number of V vulnificus infections rises, so too must providers’ suspicion for the pathogen.
- Phillips KE, Satchell KJF. Vibrio vulnificus: from oyster colonist to human pathogen [published online January 5, 2017]. PLOS Pathog. doi:10.1371/journal.ppat.1006053
- Heng SP, Letchumanan V, Deng CY, et al. Vibrio vulnificus: an environmental and clinical burden. Front Microbiol. 2017;8:997.
- Kumamoto KS, Vukich DJ. Clinical infections of Vibrio vulnificus: a case report and review of the literature. J Emerg Med. 1998;16:61-66.
- Borenstein M, Kerdel F. Infections with Vibrio vulnificus. Dermatol Clin. 2003;21:245-248.
- Baker-Austin C, Oliver JD. Vibrio vulnificus: new insights into a deadly opportunistic pathogen. Environ Microbiol. 2018;20:423-430.
- Kim SJ, Kim BC, Kim DC, et al. A fatal case of Vibrio vulnificus meningoencephalitis. Clin Microbiol Infect. 2003;9:568-571.
- Jones MK, Oliver JD. Vibrio vulnificus: disease and pathogenesis. Infect Immun. 2009;77:1723-1733.
- Horseman MA, Surani S. A comprehensive review of Vibrio vulnificus infection: an important cause of severe sepsis and skin and soft-tissue infection. Int J Infect Dis. 2011;15:E157-E166.
- Diaz JH. Skin and soft tissue infections following marine injuries and exposures in travelers. J Travel Med. 2014;21:207-213.
- Kikawa K, Yamasaki K, Sukiura T, et al. A successfully treated case of Vibrio vulnificus septicemia with shock. Jpn J Med. 1990;29:313-319.
- Perkins AP, Trimmier M. Recreational waterborne illnesses: recognition, treatment, and prevention. Am Fam Physician. 2017;95:554-560.
- Patel VJ, Gardner E, Burton CS. Vibrio vulnificus septicemia and leg ulcer. J Am Acad Dermatol. 2002;46(5 suppl):S144-S145.
- Ulusarac O, Carter E. Varied clinical presentations of Vibrio vulnificus infections: a report of four unusual cases and review of the literature. South Med J. 2004;97:613-618.
- Bross MH, Soch K, Morales R, et al. Vibrio vulnificus infection: diagnosis and treatment. Am Fam Physician. 2007;76:539-544.
- Hori M, Nakayama A, Kitagawa D, et al. A case of Vibrio vulnificus infection complicated with fulminant purpura: gene and biotype analysis of the pathogen [published online May 19, 2017]. JMM Case Rep. doi:10.1099/jmmcr.0.005096
- Kotton Y, Soboh S, Bisharat N. Vibrio vulnificus necrotizing fasciitis associated with acupuncture. Infect Dis Rep. 2015;7:5901.
- Hoffman TJ, Nelson B, Darouiche R, et al. Vibrio vulnificus septicemia. Arch Intern Med. 1988;148:1825-1827.
- Alsaad AA, Sotello D, Kruse BT, et al. Vibrio vulnificus tonsillitis after swimming in the Gulf of Mexico [published online June 28, 2017]. BMJ Case Rep. doi:10.1136/bcr-2017-221161
- Tison DL, Kelly MT. Vibrio vulnificus endometritis. J Clin Microbiol. 1984;20:185-186.
- Beatty NL, Marquez J, Mohajer MA. Skin manifestations of primary Vibrio vulnificus septicemia. Am J Trop Med Hyg. 2017;97:1-2.
- Foote A, Henderson R, Lindberg A, et al. The Australian mid-west coastal marine wound infections study. Aust Fam Physician. 2017;46:923-927.
- Marks JG Jr, Miller JJ. Lookingbill and Marks’ Principles of Dermatology. 6th ed. Elsevier; 2019.
- Kim CS, Bae EH, Ma SK, et al. Severe septicemia, necrotizing fasciitis, and peritonitis due to Vibrio vulnificus in a patient undergoing continuous ambulatory peritoneal dialysis: a case report. BMC Infect Dis. 2015;15:422.
- Phillips KE, Satchell KJF. Vibrio vulnificus: from oyster colonist to human pathogen [published online January 5, 2017]. PLOS Pathog. doi:10.1371/journal.ppat.1006053
- Heng SP, Letchumanan V, Deng CY, et al. Vibrio vulnificus: an environmental and clinical burden. Front Microbiol. 2017;8:997.
- Kumamoto KS, Vukich DJ. Clinical infections of Vibrio vulnificus: a case report and review of the literature. J Emerg Med. 1998;16:61-66.
- Borenstein M, Kerdel F. Infections with Vibrio vulnificus. Dermatol Clin. 2003;21:245-248.
- Baker-Austin C, Oliver JD. Vibrio vulnificus: new insights into a deadly opportunistic pathogen. Environ Microbiol. 2018;20:423-430.
- Kim SJ, Kim BC, Kim DC, et al. A fatal case of Vibrio vulnificus meningoencephalitis. Clin Microbiol Infect. 2003;9:568-571.
- Jones MK, Oliver JD. Vibrio vulnificus: disease and pathogenesis. Infect Immun. 2009;77:1723-1733.
- Horseman MA, Surani S. A comprehensive review of Vibrio vulnificus infection: an important cause of severe sepsis and skin and soft-tissue infection. Int J Infect Dis. 2011;15:E157-E166.
- Diaz JH. Skin and soft tissue infections following marine injuries and exposures in travelers. J Travel Med. 2014;21:207-213.
- Kikawa K, Yamasaki K, Sukiura T, et al. A successfully treated case of Vibrio vulnificus septicemia with shock. Jpn J Med. 1990;29:313-319.
- Perkins AP, Trimmier M. Recreational waterborne illnesses: recognition, treatment, and prevention. Am Fam Physician. 2017;95:554-560.
- Patel VJ, Gardner E, Burton CS. Vibrio vulnificus septicemia and leg ulcer. J Am Acad Dermatol. 2002;46(5 suppl):S144-S145.
- Ulusarac O, Carter E. Varied clinical presentations of Vibrio vulnificus infections: a report of four unusual cases and review of the literature. South Med J. 2004;97:613-618.
- Bross MH, Soch K, Morales R, et al. Vibrio vulnificus infection: diagnosis and treatment. Am Fam Physician. 2007;76:539-544.
- Hori M, Nakayama A, Kitagawa D, et al. A case of Vibrio vulnificus infection complicated with fulminant purpura: gene and biotype analysis of the pathogen [published online May 19, 2017]. JMM Case Rep. doi:10.1099/jmmcr.0.005096
- Kotton Y, Soboh S, Bisharat N. Vibrio vulnificus necrotizing fasciitis associated with acupuncture. Infect Dis Rep. 2015;7:5901.
- Hoffman TJ, Nelson B, Darouiche R, et al. Vibrio vulnificus septicemia. Arch Intern Med. 1988;148:1825-1827.
- Alsaad AA, Sotello D, Kruse BT, et al. Vibrio vulnificus tonsillitis after swimming in the Gulf of Mexico [published online June 28, 2017]. BMJ Case Rep. doi:10.1136/bcr-2017-221161
- Tison DL, Kelly MT. Vibrio vulnificus endometritis. J Clin Microbiol. 1984;20:185-186.
- Beatty NL, Marquez J, Mohajer MA. Skin manifestations of primary Vibrio vulnificus septicemia. Am J Trop Med Hyg. 2017;97:1-2.
- Foote A, Henderson R, Lindberg A, et al. The Australian mid-west coastal marine wound infections study. Aust Fam Physician. 2017;46:923-927.
- Marks JG Jr, Miller JJ. Lookingbill and Marks’ Principles of Dermatology. 6th ed. Elsevier; 2019.
- Kim CS, Bae EH, Ma SK, et al. Severe septicemia, necrotizing fasciitis, and peritonitis due to Vibrio vulnificus in a patient undergoing continuous ambulatory peritoneal dialysis: a case report. BMC Infect Dis. 2015;15:422.
Practice Points
- Vibrio vulnificus infection should be high on the differential for patients who present with chronic liver disease and immunosuppression; a history of raw seafood consumption or exposure to brackish water; and bullae, cellulitis, necrotic lesions, or sepsis.
- Time to treatment is directly proportional to mortality rates in V vulnificus infections, and prompt treatment with antibiotics, wound care, debridement, and supportive measures is necessary to decrease mortality rates.
- The incidence of V vulnificus infection is rising in the United States, likely due to a combination of factors, including an aging population with multiple comorbidities, improvements in diagnosis, and climate change.
Which behavioral health screening tool should you use—and when?
Many screening tools are available in the public domain to assess a variety of symptoms related to impaired mental health. These tools can be used to quickly evaluate for mood, suicidal ideation or behavior, anxiety, sleep, substance use, pain, trauma, memory, and cognition (TABLE). Individuals with poor mental health incur high health care costs. Those suffering from anxiety and posttraumatic stress have more outpatient and emergency department visits and hospitalizations than patients without these disorders,1,2 although use of mental health care services has been related to a decrease in the overutilization of health care services in general.3
Here we review several screening tools that can help you to identify symptoms of mental illnesses and thus, provide prompt early intervention, including referrals to psychological and psychiatric services.
Mood disorders
Most patients with mood disorders are treated in primary care settings.4 Quickly measuring patients’ mood symptoms can expedite treatment for those who need it. Many primary care clinics use the 9-item Patient Health Questionnaire (PHQ-9) to screen for depression.5 The US Preventive Services Task Force (USPSTF) has recommended screening for depression with adequate systems to ensure accurate diagnoses, effective treatment, and follow-up. Although the USPSTF did not specially endorse screening for bipolar disorder, it followed that recommendation with the qualifying statement, “positive screening results [for depression] should lead to additional assessment that considers severity of depression and comorbid psychological problems, alternate diagnoses, and medical conditions.”6 Thus, following a positive screen result for depression, consider using a screening tool for mood disorders to provide diagnostic clarification.
The Mood Disorder Questionnaire (MDQ) is a validated 15-item, self-administered questionnaire that takes only 5 minutes to use in screening adult patients for bipolar I disorder.7 The MDQ assesses specific behaviors related to bipolar disorder, symptom co-occurrence, and functional impairment. The MDQ has low sensitivity (58%) but good specificity (93%) in a primary care setting.8 However, the MDQ is not a diagnostic instrument. A positive screen result should prompt a more thorough clinical evaluation, if necessary, by a professional trained in psychiatric disorders.
We recommend completing the MDQ prior to prescribing antidepressants. You can also monitor a patient’s response to treatment with serial MDQ testing. The MDQ is useful, too, when a patient has unclear mood symptoms that may have features overlapping with bipolar disorder. Furthermore, we recommend screening for bipolar disorder with every patient who reports symptoms of depression, given that some pharmacologic treatments (predominately selective serotonin reuptake inhibitors) can induce mania in patients who actually have unrecognized bipolar disorder.9
Continue to: Suicide...
Suicide
Suicide is the 10th leading cause of death among the general population. All demographic groups are impacted by suicide; however, the most vulnerable are men ages 45 to 64 years.10 Given the imminent risk to individuals who experience suicidal ideation, properly assessing and targeting suicidal risk is paramount.
The Columbia Suicide Severity Rating Scale (C-SSRS) can be completed in an interview format or as a patient self-report. Versions of the C-SSRS are available for children, adolescents, and adults. It can be used in practice with any patient who may be at risk for suicide. Specifically, consider using the C-SSRS when a patient scores 1 or greater on the PHQ-9 or when risk is revealed with another brief screening tool that includes suicidal ideation.
The C-SSRS covers 10 categories related to suicidal ideation and behavior that the clinician explores with questions requiring only Yes/No responses. The C-SSRS demonstrates moderate-to-strong internal consistency and reliability, and it has shown a high degree of sensitivity (95%) and specificity (95%) for suicidal ideation.11
Anxiety and physiologic arousal
Generalized anxiety disorder (GAD) is one of the most common anxiety disorders, with an estimated prevalence of 2.8% to 8.5% among primary care patients.12 Brief, validated screening tools such as the Generalized Anxiety Disorder–7 item (GAD-7) scale can be effective in identifying anxiety and other related disorders in primary care settings.
The GAD-7 comprises 7 items inquiring about symptoms experienced in the past 2 weeks. Scores range from 0 to 21, with cutoffs of 5, 10, and 15 indicating mild, moderate, and severe anxiety, respectively. This questionnaire is appropriate for use with adults and has strong specificity, internal consistency, and test-retest reliability.12 Specificity and sensitivity of the GAD-7 are maximized at a cutoff score of 10 or greater, both exceeding 80%.12 The GAD-7 can be used when patients report symptoms of anxiety or when one needs to screen for anxiety with new patients or more clearly understand symptoms among patients who have complex mental health concerns.
The Screen for Child Anxiety Related Disorders (SCARED) is a 41-item self-report measure of anxiety for children ages 8 to 18. The SCARED questionnaire yields an overall anxiety score, as well as subscales for panic disorder or significant somatic symptoms, generalized anxiety disorder, separation anxiety, social anxiety disorder, and significant school avoidance.13 There is also a 5-item version of the SCARED, which can be useful for brief screening in fast-paced settings when no anxiety disorder is suspected, or for children who may have anxiety but exhibit reduced verbal capacity. The SCARED has been found to have moderate sensitivity (81.8%) and specificity (52%) for diagnosing anxiety disorders in a community sample, with an optimal cutoff point of 22 on the total scale.14
Sleep
Sleep concerns are common, with the prevalence of insomnia among adults in the United States estimated to be 19.2%.15 The importance of assessing these concerns cannot be overstated, and primary care providers are the ones patients consult most often.16 The gold standard in assessing sleep disorders is a structured clinical interview, polysomnography, sleep diary, and actigraphy (home-based monitoring of movement through a device, often worn on the wrist).17,18 However, this work-up is expensive, time-intensive, and impractical in integrated care settings; thus the need for a brief, self-report screening tool to guide further assessment and intervention.
The Insomnia Severity Index (ISI) assesses patients’ perceptions of their insomnia. The ISI was developed to aid both in the clinical evaluation of patients with insomnia and to measure treatment outcomes. Administration of the ISI takes approximately 5 minutes, and scoring takes less than 1 minute.
The ISI is composed of 7 items that measure the severity of sleep onset and sleep maintenance difficulties, satisfaction with current sleep, impact on daily functioning, impairment observable to others, and degree of distress caused by the sleep problems. Each item is scored on a 0 to 4 Likert-type scale, and the individual items are summed for a total score of 0 to 28, with higher scores suggesting more severe insomnia. Evidence-based guidelines recommend cognitive behavioral therapy for insomnia (CBT-I) as the first-line treatment for adults with primary insomnia.19
Several validation studies have found the ISI to be a reliable measure of perceived insomnia severity, and one that is sensitive to changes in patients’ perceptions of treatment outcomes.20,21 An additional validation study confirmed that in primary care settings, a cutoff score of 14 should be used to indicate the likely presence of clinical insomnia22 and to guide further assessment and intervention.
The percentage of insomniac patients correctly identified with the ISI was 82.2%, with moderate sensitivity (82.4%) and specificity (82.1%).22 A positive predictive value of 70% was found, meaning that an insomnia disorder is probable when the ISI total score is 14 or higher; conversely, the negative predictive value was 90.2%.
Continue to: Substance use and pain...
Substance use and pain
The evaluation of alcohol and drug use is an integral part of assessing risky health behaviors. The 10-item Alcohol Use Disorder Identification Test (AUDIT) is a self-report tool developed by the World Health Organization.23,24 Validated in medical settings, scores of 8 or higher suggest problematic drinking.25,26 The AUDIT has demonstrated high specificity (94%) and moderate sensitivity (81%) in primary care settings.27 The AUDIT-C (items 1, 2, and 3 of the AUDIT) has also demonstrated comparable sensitivity, although slightly lower specificity, than the full AUDIT, suggesting that this 3-question screen can also be used in primary care settings.27
Opioid medications, frequently prescribed for chronic pain, present serious risks for many patients. The Screener and Opioid Assessment for Patients with Pain–Revised (SOAPP-R) is a 24-item self-reporting scale that can be completed in approximately 10 minutes.28 A score of 18 or higher has identified 81% of patients at high risk for opioid misuse in a clinical setting, with moderate specificity (68%). Although other factors should be considered when assessing risk of opioid misuse, the SOAPP-R is a helpful and quick addition to an opioid risk assessment.
The CRAFFT Screening Tool for Adolescent Substance Use is administered by the clinician for youths ages 14 to 21. The first 3 questions ask about use of alcohol, marijuana, or other substances during the past 12 months. What follows are questions related to the young person’s specific experiences with substances in relation to Cars, Relaxation, being Alone, Forgetting, Family/Friends, and Trouble (CRAFFT). The CRAFFT has shown moderate sensitivity (76%) and good specificity (94%) for identifying any problem with substance use.29 These measures may be administered to clarify or confirm substance use patterns (ie, duration, frequency), or to determine the severity of problems related to substance use (ie, social or legal problems).
Trauma and PTSD
Approximately 7.7 million adults per year will experience posttraumatic stress disorder (PTSD) symptoms, although PTSD can affect individuals of any age.30 Given the impact that trauma can have, assess for PTSD in patients who have a history of trauma or who otherwise seem to be at risk. The Post-traumatic Stress Disorder Checklist (PCL-5) is a 20-item self-report questionnaire that screens for symptoms directly from the Diagnostic and Statistical Manual of Mental Disorders, 5th Edition (DSM-5) criteria for PTSD. One limitation is that the questionnaire is only validated for adults ages 18 years or older. Completion of the PCL-5 takes 5 to 10 minutes. The PCL-5 has strong internal consistency reliability (94%) and test-retest reliability (82%).31 With a cutoff score of 33 or higher, the sensitivity and specificity have been shown to be moderately high (74.5% and 70.6%, respectively).32
The Child and Adolescent Trauma Screen (CATS) is used to assess for potentially traumatic events and PTSD symptoms in children and adolescents. These symptoms are based on the DSM-5, and therefore the CATS can act as a useful diagnostic aid. The CATS is also available in Spanish, with both caregiver-report (for children ages 3-6 years or 7-17 years) and self-report (for ages 7-17 years) versions. Practical use of the PCL-5 and the CATS involves screening for PTSD symptoms, supporting a provisional diagnosis of PTSD, and monitoring PTSD symptom changes during and after treatment.
Memory and cognition
Cognitive screening is a first step in evaluating possible dementia and other neuropsychological disorders. The importance of brief cognitive screening in primary care cannot be understated, especially for an aging patient population. Although the Mini Mental Status Exam (MMSE) has been widely used among health care providers and researchers, we recommend the Montreal Cognitive Assessment (MoCA).
The MoCA is a simple, standalone cognitive screening tool validated for adults ages 55 to 85 years.33 The MoCA addresses many important cognitive domains, fits on one page, and can be administered by a trained provider in 10 minutes. Research also suggests that it has strong test-retest reliability and positive and negative predictive values for mild cognitive impairment and Alzheimer dementia, and it has been found to be more sensitive than the MMSE.34 We additionally recommend the MoCA as it measures several cognitive skills that are not addressed on the MMSE, including verbal fluency and abstraction.34 Scores below 25 are suggestive of cognitive impairment and should lead to a referral for neuropsychological testing.
The MoCA’s sensitivity for detecting cognitive impairment is high (94%), and specificity is low (42%).35 To ensure consistency and accuracy in administering the MoCA, certification is now required via an online training program through www.mocatest.org.
Adapting these screening tools to practice
These tools are not meant to be used at every appointment. Every practice is different, and each clinic or physician can tailor the use of these screening tools to the needs of the patient population, as concerns arise, or in collaboration with other providers. Additionally, these screening tools can be used in both integrated care and in private practice, to prompt a more thorough assessment or to aid in—and inform—treatment. Although some physicians choose to administer certain screening tools at each clinic visit, knowing about the availability of other tools can be useful in assessing various issues. The FIGURE can be used to aid in the clinical decision-making process.
- Robinson RL, Grabner M, Palli SR, et al. Covariates of depression and high utilizers of healthcare: impact on resource use and costs. J Psychosom Res. 2016,85:35-43.
- Fogarty CT, Sharma S, Chetty VK, et al. Mental health conditions are associated with increased health care utilization among urban family medicine patients. J Am Board Fam Med. 2008,21:398-407.
- Weissman JD, Russell D, Beasley J, et al. Relationships between adult emotional states and indicators of health care utilization: findings from the National Health Interview Survey 2006–2014. J Psychosom Res. 2016,91:75-81.
- Haddad M, Walters P. Mood disorders in primary care. Psychiatry. 2009,8:71-75.
- Mitchell AJ, Yadegarfar M, Gill J, et al. Case finding and screening clinical utility of the Patient Health Questionnaire (PHQ-9 and PHQ-2) for depression in primary care: a diagnostic metaanalysis of 40 studies. BJPsych Open. 2016,2:127-138.
- Siu AL and US Preventive Services Task Force. Screening for depression in adults. JAMA. 2016;315:380-387.
- Hirschfeld RM, Williams JB, Spitzer RL, et al. Development and validation of a screening instrument for bipolar spectrum disorder: the Mood Disorder Questionnaire. Am J Psychiatry. 2000;157:1873-1875.
- Hirschfeld RM, Cass AR, Holt DC, et al. Screening for bipolar disorder in patients treated for depression in a family medicine clinic. J Am Board Fam Med. 2005;18:233-239.
- Das AK, Olfson M, Gameroff MJ, et al. Screening for bipolar disorder in a primary care practice. JAMA. 2005;293:956-963.
- CDC. Suicide mortality in the United States, 1999-2017. www.cdc.gov/nchs/products/databriefs/db330.htm. Accessed October 23, 2020.
- Viguera AC, Milano N, Ralston L, et al. Comparison of electronic screening for suicidal risk with Patient Health Questionnaire Item 9 and the Columbia Suicide Severity Rating Scale in an outpatient psychiatric clinic. Psychosomatics. 2015;56:460-469.
- Spitzer RL, Kroenke K, Williams JBW, et al. A brief measure for assessing generalized anxiety disorder: the GAD-7. Arch Intern Med. 2006;166:1092-1097.
- Birmaher B, Khetarpal S, Brent D, et al. The Screen for Child Anxiety Related Emotional Disorders (SCARED): scale construction and psychometric characteristics. J Am Acad Chil Adolesc Psychiatry. 1997;36:545-553.
- DeSousa DA, Salum GA, Isolan LR, et al. Sensitivity and specificity of the Screen for Child Anxiety Related Emotional Disorders (SCARED): a community-based study. Child Psychiatry Hum Dev. 2013;44:391-399.
- Ford ES, Cunningham TJ, Giles WH, et al. Trends in insomnia and excessive daytime sleepiness among U.S. adults from 2002 to 2012. Sleep Med. 2015;16:372-378.
- Morin CM, LeBlanc M, Daley M, et al. Epidemiology of insomnia: prevalence, self-help treatments, consultations, and determinants of help-seeking behaviors. Sleep Med. 2006;7:123-130.
- Buysse DJ, Ancoli-Israel S, Edinger JD, et al. Recommendations for a standard research assessment of insomnia. Sleep. 2006;29:1155-1173.
- Martin JL, Hakim AD. Wrist actigraphy. Chest. 2011;139:1514-1527.
- Riemann D, Baglioni C, Bassetti C, et al. European guideline for the diagnosis and treatment of insomnia. J Sleep Res. 2017;26:675-700.
- Bastien CH, Vallières A, Morin CM. Validation of the Insomnia Severity Index as an outcome measure for insomnia research. Sleep Med. 2001;2:297-307.
- Wong ML, Lau KNT, Espie CA, et al. Psychometric properties of the Sleep Condition Indicator and Insomnia Severity Index in the evaluation of insomnia disorder. Sleep Med. 2017;33:76-81.
- Gagnon C, Bélanger L, Ivers H, et al. Validation of the Insomnia Severity Index in primary care. J Am Board Fam Med. 2013;26:701-710.
- Saunders JB, Aasland OG, Babor TF, et al. Development of the Alcohol Use Disorders Identification Test (AUDIT): WHO Collaborative Project on Early Detection of Persons with Harmful Alcohol Consumption. Addiction. 1993;88:791-804.
- Selin KH. Test-retest reliability of the Alcohol Use Disorder Identification Test in a general population sample. Alcohol Clin Exp Res. 2003;27:1428-1435.
- Bohn MJ, Babor TF, Kranzler HR. The Alcohol Use Disorders Identification Test (AUDIT): validation of a screening instrument for use in medical settings. J Stud Alcohol. 1995;56:423-432.
- Conigrave KM, Hall WD, Saunders JB. The AUDIT questionnaire: choosing a cut-off score. Addiction. 1995;90:1349-1356.
- Gomez A, Conde A, Santana JM, et al. Diagnostic usefulness of brief versions of Alcohol Use Identification Test (AUDIT) for detecting hazardous drinkers in primary care settings. J Stud Alcohol. 2005;66:305-308.
- Butler SF, Fernandez K, Benoit C, et al. Validation of the revised Screener and Opioid Assessment for Patients with Pain (SOAPPR). J Pain. 2008;9:360-372.
- Knight JR, Sherritt L, Shrier LA, et al. Validity of the CRAFFT substance abuse screening test among adolescent clinic patients. Arch Pediatr Adolesc Med. 2002;156:607-614.
- DHHS. Post-traumatic stress disorder (PTSD). https://archives.nih.gov/asites/report/09-09-2019/report.nih.gov/nihfactsheets/ViewFactSheetfdf8.html?csid=58&key=P#P. Accessed October 23,2020.
- Blevins CA, Weathers FW, Davis MT, et al. The Posttraumatic Stress Disorder Checklist for DSM-5 (PCL-5): development and initial psychometric evaluation. J Trauma Stress. 2015;28:489-498.
- Verhey R, Chilbanda D, Gibson L, et al. Validation of the Posttraumatic Stress Disorder Checklist- 5 (PCL-5) in a primary care population with high HIV prevalence in Zimbabwe. BMC Psychiatry. 2018;18:109.
- Nasreddine ZS, Phillips NA, Bédirian V, et al. The Montreal Cognitive Assessment, MoCA: a brief screening tool for mild cognitive impairment. J Am Geriatr Soc. 2005;53:695-699.
- Stewart S, O’Riley A, Edelstein B, et al. A preliminary comparison of three cognitive screening instruments in long term care: the MMSE, SLUMS, and MoCA. Clin Gerontol. 2012;35:57-75.
- Godefroy O, Fickl A, Roussel M, et al. Is the Montreal Cognitive Assessment superior to the Mini-Mental State Examination to detect poststroke cognitive impairment? A study with neuropsychological evaluation. Stroke. 2011;42:1712-1716.
Many screening tools are available in the public domain to assess a variety of symptoms related to impaired mental health. These tools can be used to quickly evaluate for mood, suicidal ideation or behavior, anxiety, sleep, substance use, pain, trauma, memory, and cognition (TABLE). Individuals with poor mental health incur high health care costs. Those suffering from anxiety and posttraumatic stress have more outpatient and emergency department visits and hospitalizations than patients without these disorders,1,2 although use of mental health care services has been related to a decrease in the overutilization of health care services in general.3
Here we review several screening tools that can help you to identify symptoms of mental illnesses and thus, provide prompt early intervention, including referrals to psychological and psychiatric services.
Mood disorders
Most patients with mood disorders are treated in primary care settings.4 Quickly measuring patients’ mood symptoms can expedite treatment for those who need it. Many primary care clinics use the 9-item Patient Health Questionnaire (PHQ-9) to screen for depression.5 The US Preventive Services Task Force (USPSTF) has recommended screening for depression with adequate systems to ensure accurate diagnoses, effective treatment, and follow-up. Although the USPSTF did not specially endorse screening for bipolar disorder, it followed that recommendation with the qualifying statement, “positive screening results [for depression] should lead to additional assessment that considers severity of depression and comorbid psychological problems, alternate diagnoses, and medical conditions.”6 Thus, following a positive screen result for depression, consider using a screening tool for mood disorders to provide diagnostic clarification.
The Mood Disorder Questionnaire (MDQ) is a validated 15-item, self-administered questionnaire that takes only 5 minutes to use in screening adult patients for bipolar I disorder.7 The MDQ assesses specific behaviors related to bipolar disorder, symptom co-occurrence, and functional impairment. The MDQ has low sensitivity (58%) but good specificity (93%) in a primary care setting.8 However, the MDQ is not a diagnostic instrument. A positive screen result should prompt a more thorough clinical evaluation, if necessary, by a professional trained in psychiatric disorders.
We recommend completing the MDQ prior to prescribing antidepressants. You can also monitor a patient’s response to treatment with serial MDQ testing. The MDQ is useful, too, when a patient has unclear mood symptoms that may have features overlapping with bipolar disorder. Furthermore, we recommend screening for bipolar disorder with every patient who reports symptoms of depression, given that some pharmacologic treatments (predominately selective serotonin reuptake inhibitors) can induce mania in patients who actually have unrecognized bipolar disorder.9
Continue to: Suicide...
Suicide
Suicide is the 10th leading cause of death among the general population. All demographic groups are impacted by suicide; however, the most vulnerable are men ages 45 to 64 years.10 Given the imminent risk to individuals who experience suicidal ideation, properly assessing and targeting suicidal risk is paramount.
The Columbia Suicide Severity Rating Scale (C-SSRS) can be completed in an interview format or as a patient self-report. Versions of the C-SSRS are available for children, adolescents, and adults. It can be used in practice with any patient who may be at risk for suicide. Specifically, consider using the C-SSRS when a patient scores 1 or greater on the PHQ-9 or when risk is revealed with another brief screening tool that includes suicidal ideation.
The C-SSRS covers 10 categories related to suicidal ideation and behavior that the clinician explores with questions requiring only Yes/No responses. The C-SSRS demonstrates moderate-to-strong internal consistency and reliability, and it has shown a high degree of sensitivity (95%) and specificity (95%) for suicidal ideation.11
Anxiety and physiologic arousal
Generalized anxiety disorder (GAD) is one of the most common anxiety disorders, with an estimated prevalence of 2.8% to 8.5% among primary care patients.12 Brief, validated screening tools such as the Generalized Anxiety Disorder–7 item (GAD-7) scale can be effective in identifying anxiety and other related disorders in primary care settings.
The GAD-7 comprises 7 items inquiring about symptoms experienced in the past 2 weeks. Scores range from 0 to 21, with cutoffs of 5, 10, and 15 indicating mild, moderate, and severe anxiety, respectively. This questionnaire is appropriate for use with adults and has strong specificity, internal consistency, and test-retest reliability.12 Specificity and sensitivity of the GAD-7 are maximized at a cutoff score of 10 or greater, both exceeding 80%.12 The GAD-7 can be used when patients report symptoms of anxiety or when one needs to screen for anxiety with new patients or more clearly understand symptoms among patients who have complex mental health concerns.
The Screen for Child Anxiety Related Disorders (SCARED) is a 41-item self-report measure of anxiety for children ages 8 to 18. The SCARED questionnaire yields an overall anxiety score, as well as subscales for panic disorder or significant somatic symptoms, generalized anxiety disorder, separation anxiety, social anxiety disorder, and significant school avoidance.13 There is also a 5-item version of the SCARED, which can be useful for brief screening in fast-paced settings when no anxiety disorder is suspected, or for children who may have anxiety but exhibit reduced verbal capacity. The SCARED has been found to have moderate sensitivity (81.8%) and specificity (52%) for diagnosing anxiety disorders in a community sample, with an optimal cutoff point of 22 on the total scale.14
Sleep
Sleep concerns are common, with the prevalence of insomnia among adults in the United States estimated to be 19.2%.15 The importance of assessing these concerns cannot be overstated, and primary care providers are the ones patients consult most often.16 The gold standard in assessing sleep disorders is a structured clinical interview, polysomnography, sleep diary, and actigraphy (home-based monitoring of movement through a device, often worn on the wrist).17,18 However, this work-up is expensive, time-intensive, and impractical in integrated care settings; thus the need for a brief, self-report screening tool to guide further assessment and intervention.
The Insomnia Severity Index (ISI) assesses patients’ perceptions of their insomnia. The ISI was developed to aid both in the clinical evaluation of patients with insomnia and to measure treatment outcomes. Administration of the ISI takes approximately 5 minutes, and scoring takes less than 1 minute.
The ISI is composed of 7 items that measure the severity of sleep onset and sleep maintenance difficulties, satisfaction with current sleep, impact on daily functioning, impairment observable to others, and degree of distress caused by the sleep problems. Each item is scored on a 0 to 4 Likert-type scale, and the individual items are summed for a total score of 0 to 28, with higher scores suggesting more severe insomnia. Evidence-based guidelines recommend cognitive behavioral therapy for insomnia (CBT-I) as the first-line treatment for adults with primary insomnia.19
Several validation studies have found the ISI to be a reliable measure of perceived insomnia severity, and one that is sensitive to changes in patients’ perceptions of treatment outcomes.20,21 An additional validation study confirmed that in primary care settings, a cutoff score of 14 should be used to indicate the likely presence of clinical insomnia22 and to guide further assessment and intervention.
The percentage of insomniac patients correctly identified with the ISI was 82.2%, with moderate sensitivity (82.4%) and specificity (82.1%).22 A positive predictive value of 70% was found, meaning that an insomnia disorder is probable when the ISI total score is 14 or higher; conversely, the negative predictive value was 90.2%.
Continue to: Substance use and pain...
Substance use and pain
The evaluation of alcohol and drug use is an integral part of assessing risky health behaviors. The 10-item Alcohol Use Disorder Identification Test (AUDIT) is a self-report tool developed by the World Health Organization.23,24 Validated in medical settings, scores of 8 or higher suggest problematic drinking.25,26 The AUDIT has demonstrated high specificity (94%) and moderate sensitivity (81%) in primary care settings.27 The AUDIT-C (items 1, 2, and 3 of the AUDIT) has also demonstrated comparable sensitivity, although slightly lower specificity, than the full AUDIT, suggesting that this 3-question screen can also be used in primary care settings.27
Opioid medications, frequently prescribed for chronic pain, present serious risks for many patients. The Screener and Opioid Assessment for Patients with Pain–Revised (SOAPP-R) is a 24-item self-reporting scale that can be completed in approximately 10 minutes.28 A score of 18 or higher has identified 81% of patients at high risk for opioid misuse in a clinical setting, with moderate specificity (68%). Although other factors should be considered when assessing risk of opioid misuse, the SOAPP-R is a helpful and quick addition to an opioid risk assessment.
The CRAFFT Screening Tool for Adolescent Substance Use is administered by the clinician for youths ages 14 to 21. The first 3 questions ask about use of alcohol, marijuana, or other substances during the past 12 months. What follows are questions related to the young person’s specific experiences with substances in relation to Cars, Relaxation, being Alone, Forgetting, Family/Friends, and Trouble (CRAFFT). The CRAFFT has shown moderate sensitivity (76%) and good specificity (94%) for identifying any problem with substance use.29 These measures may be administered to clarify or confirm substance use patterns (ie, duration, frequency), or to determine the severity of problems related to substance use (ie, social or legal problems).
Trauma and PTSD
Approximately 7.7 million adults per year will experience posttraumatic stress disorder (PTSD) symptoms, although PTSD can affect individuals of any age.30 Given the impact that trauma can have, assess for PTSD in patients who have a history of trauma or who otherwise seem to be at risk. The Post-traumatic Stress Disorder Checklist (PCL-5) is a 20-item self-report questionnaire that screens for symptoms directly from the Diagnostic and Statistical Manual of Mental Disorders, 5th Edition (DSM-5) criteria for PTSD. One limitation is that the questionnaire is only validated for adults ages 18 years or older. Completion of the PCL-5 takes 5 to 10 minutes. The PCL-5 has strong internal consistency reliability (94%) and test-retest reliability (82%).31 With a cutoff score of 33 or higher, the sensitivity and specificity have been shown to be moderately high (74.5% and 70.6%, respectively).32
The Child and Adolescent Trauma Screen (CATS) is used to assess for potentially traumatic events and PTSD symptoms in children and adolescents. These symptoms are based on the DSM-5, and therefore the CATS can act as a useful diagnostic aid. The CATS is also available in Spanish, with both caregiver-report (for children ages 3-6 years or 7-17 years) and self-report (for ages 7-17 years) versions. Practical use of the PCL-5 and the CATS involves screening for PTSD symptoms, supporting a provisional diagnosis of PTSD, and monitoring PTSD symptom changes during and after treatment.
Memory and cognition
Cognitive screening is a first step in evaluating possible dementia and other neuropsychological disorders. The importance of brief cognitive screening in primary care cannot be understated, especially for an aging patient population. Although the Mini Mental Status Exam (MMSE) has been widely used among health care providers and researchers, we recommend the Montreal Cognitive Assessment (MoCA).
The MoCA is a simple, standalone cognitive screening tool validated for adults ages 55 to 85 years.33 The MoCA addresses many important cognitive domains, fits on one page, and can be administered by a trained provider in 10 minutes. Research also suggests that it has strong test-retest reliability and positive and negative predictive values for mild cognitive impairment and Alzheimer dementia, and it has been found to be more sensitive than the MMSE.34 We additionally recommend the MoCA as it measures several cognitive skills that are not addressed on the MMSE, including verbal fluency and abstraction.34 Scores below 25 are suggestive of cognitive impairment and should lead to a referral for neuropsychological testing.
The MoCA’s sensitivity for detecting cognitive impairment is high (94%), and specificity is low (42%).35 To ensure consistency and accuracy in administering the MoCA, certification is now required via an online training program through www.mocatest.org.
Adapting these screening tools to practice
These tools are not meant to be used at every appointment. Every practice is different, and each clinic or physician can tailor the use of these screening tools to the needs of the patient population, as concerns arise, or in collaboration with other providers. Additionally, these screening tools can be used in both integrated care and in private practice, to prompt a more thorough assessment or to aid in—and inform—treatment. Although some physicians choose to administer certain screening tools at each clinic visit, knowing about the availability of other tools can be useful in assessing various issues. The FIGURE can be used to aid in the clinical decision-making process.
Many screening tools are available in the public domain to assess a variety of symptoms related to impaired mental health. These tools can be used to quickly evaluate for mood, suicidal ideation or behavior, anxiety, sleep, substance use, pain, trauma, memory, and cognition (TABLE). Individuals with poor mental health incur high health care costs. Those suffering from anxiety and posttraumatic stress have more outpatient and emergency department visits and hospitalizations than patients without these disorders,1,2 although use of mental health care services has been related to a decrease in the overutilization of health care services in general.3
Here we review several screening tools that can help you to identify symptoms of mental illnesses and thus, provide prompt early intervention, including referrals to psychological and psychiatric services.
Mood disorders
Most patients with mood disorders are treated in primary care settings.4 Quickly measuring patients’ mood symptoms can expedite treatment for those who need it. Many primary care clinics use the 9-item Patient Health Questionnaire (PHQ-9) to screen for depression.5 The US Preventive Services Task Force (USPSTF) has recommended screening for depression with adequate systems to ensure accurate diagnoses, effective treatment, and follow-up. Although the USPSTF did not specially endorse screening for bipolar disorder, it followed that recommendation with the qualifying statement, “positive screening results [for depression] should lead to additional assessment that considers severity of depression and comorbid psychological problems, alternate diagnoses, and medical conditions.”6 Thus, following a positive screen result for depression, consider using a screening tool for mood disorders to provide diagnostic clarification.
The Mood Disorder Questionnaire (MDQ) is a validated 15-item, self-administered questionnaire that takes only 5 minutes to use in screening adult patients for bipolar I disorder.7 The MDQ assesses specific behaviors related to bipolar disorder, symptom co-occurrence, and functional impairment. The MDQ has low sensitivity (58%) but good specificity (93%) in a primary care setting.8 However, the MDQ is not a diagnostic instrument. A positive screen result should prompt a more thorough clinical evaluation, if necessary, by a professional trained in psychiatric disorders.
We recommend completing the MDQ prior to prescribing antidepressants. You can also monitor a patient’s response to treatment with serial MDQ testing. The MDQ is useful, too, when a patient has unclear mood symptoms that may have features overlapping with bipolar disorder. Furthermore, we recommend screening for bipolar disorder with every patient who reports symptoms of depression, given that some pharmacologic treatments (predominately selective serotonin reuptake inhibitors) can induce mania in patients who actually have unrecognized bipolar disorder.9
Continue to: Suicide...
Suicide
Suicide is the 10th leading cause of death among the general population. All demographic groups are impacted by suicide; however, the most vulnerable are men ages 45 to 64 years.10 Given the imminent risk to individuals who experience suicidal ideation, properly assessing and targeting suicidal risk is paramount.
The Columbia Suicide Severity Rating Scale (C-SSRS) can be completed in an interview format or as a patient self-report. Versions of the C-SSRS are available for children, adolescents, and adults. It can be used in practice with any patient who may be at risk for suicide. Specifically, consider using the C-SSRS when a patient scores 1 or greater on the PHQ-9 or when risk is revealed with another brief screening tool that includes suicidal ideation.
The C-SSRS covers 10 categories related to suicidal ideation and behavior that the clinician explores with questions requiring only Yes/No responses. The C-SSRS demonstrates moderate-to-strong internal consistency and reliability, and it has shown a high degree of sensitivity (95%) and specificity (95%) for suicidal ideation.11
Anxiety and physiologic arousal
Generalized anxiety disorder (GAD) is one of the most common anxiety disorders, with an estimated prevalence of 2.8% to 8.5% among primary care patients.12 Brief, validated screening tools such as the Generalized Anxiety Disorder–7 item (GAD-7) scale can be effective in identifying anxiety and other related disorders in primary care settings.
The GAD-7 comprises 7 items inquiring about symptoms experienced in the past 2 weeks. Scores range from 0 to 21, with cutoffs of 5, 10, and 15 indicating mild, moderate, and severe anxiety, respectively. This questionnaire is appropriate for use with adults and has strong specificity, internal consistency, and test-retest reliability.12 Specificity and sensitivity of the GAD-7 are maximized at a cutoff score of 10 or greater, both exceeding 80%.12 The GAD-7 can be used when patients report symptoms of anxiety or when one needs to screen for anxiety with new patients or more clearly understand symptoms among patients who have complex mental health concerns.
The Screen for Child Anxiety Related Disorders (SCARED) is a 41-item self-report measure of anxiety for children ages 8 to 18. The SCARED questionnaire yields an overall anxiety score, as well as subscales for panic disorder or significant somatic symptoms, generalized anxiety disorder, separation anxiety, social anxiety disorder, and significant school avoidance.13 There is also a 5-item version of the SCARED, which can be useful for brief screening in fast-paced settings when no anxiety disorder is suspected, or for children who may have anxiety but exhibit reduced verbal capacity. The SCARED has been found to have moderate sensitivity (81.8%) and specificity (52%) for diagnosing anxiety disorders in a community sample, with an optimal cutoff point of 22 on the total scale.14
Sleep
Sleep concerns are common, with the prevalence of insomnia among adults in the United States estimated to be 19.2%.15 The importance of assessing these concerns cannot be overstated, and primary care providers are the ones patients consult most often.16 The gold standard in assessing sleep disorders is a structured clinical interview, polysomnography, sleep diary, and actigraphy (home-based monitoring of movement through a device, often worn on the wrist).17,18 However, this work-up is expensive, time-intensive, and impractical in integrated care settings; thus the need for a brief, self-report screening tool to guide further assessment and intervention.
The Insomnia Severity Index (ISI) assesses patients’ perceptions of their insomnia. The ISI was developed to aid both in the clinical evaluation of patients with insomnia and to measure treatment outcomes. Administration of the ISI takes approximately 5 minutes, and scoring takes less than 1 minute.
The ISI is composed of 7 items that measure the severity of sleep onset and sleep maintenance difficulties, satisfaction with current sleep, impact on daily functioning, impairment observable to others, and degree of distress caused by the sleep problems. Each item is scored on a 0 to 4 Likert-type scale, and the individual items are summed for a total score of 0 to 28, with higher scores suggesting more severe insomnia. Evidence-based guidelines recommend cognitive behavioral therapy for insomnia (CBT-I) as the first-line treatment for adults with primary insomnia.19
Several validation studies have found the ISI to be a reliable measure of perceived insomnia severity, and one that is sensitive to changes in patients’ perceptions of treatment outcomes.20,21 An additional validation study confirmed that in primary care settings, a cutoff score of 14 should be used to indicate the likely presence of clinical insomnia22 and to guide further assessment and intervention.
The percentage of insomniac patients correctly identified with the ISI was 82.2%, with moderate sensitivity (82.4%) and specificity (82.1%).22 A positive predictive value of 70% was found, meaning that an insomnia disorder is probable when the ISI total score is 14 or higher; conversely, the negative predictive value was 90.2%.
Continue to: Substance use and pain...
Substance use and pain
The evaluation of alcohol and drug use is an integral part of assessing risky health behaviors. The 10-item Alcohol Use Disorder Identification Test (AUDIT) is a self-report tool developed by the World Health Organization.23,24 Validated in medical settings, scores of 8 or higher suggest problematic drinking.25,26 The AUDIT has demonstrated high specificity (94%) and moderate sensitivity (81%) in primary care settings.27 The AUDIT-C (items 1, 2, and 3 of the AUDIT) has also demonstrated comparable sensitivity, although slightly lower specificity, than the full AUDIT, suggesting that this 3-question screen can also be used in primary care settings.27
Opioid medications, frequently prescribed for chronic pain, present serious risks for many patients. The Screener and Opioid Assessment for Patients with Pain–Revised (SOAPP-R) is a 24-item self-reporting scale that can be completed in approximately 10 minutes.28 A score of 18 or higher has identified 81% of patients at high risk for opioid misuse in a clinical setting, with moderate specificity (68%). Although other factors should be considered when assessing risk of opioid misuse, the SOAPP-R is a helpful and quick addition to an opioid risk assessment.
The CRAFFT Screening Tool for Adolescent Substance Use is administered by the clinician for youths ages 14 to 21. The first 3 questions ask about use of alcohol, marijuana, or other substances during the past 12 months. What follows are questions related to the young person’s specific experiences with substances in relation to Cars, Relaxation, being Alone, Forgetting, Family/Friends, and Trouble (CRAFFT). The CRAFFT has shown moderate sensitivity (76%) and good specificity (94%) for identifying any problem with substance use.29 These measures may be administered to clarify or confirm substance use patterns (ie, duration, frequency), or to determine the severity of problems related to substance use (ie, social or legal problems).
Trauma and PTSD
Approximately 7.7 million adults per year will experience posttraumatic stress disorder (PTSD) symptoms, although PTSD can affect individuals of any age.30 Given the impact that trauma can have, assess for PTSD in patients who have a history of trauma or who otherwise seem to be at risk. The Post-traumatic Stress Disorder Checklist (PCL-5) is a 20-item self-report questionnaire that screens for symptoms directly from the Diagnostic and Statistical Manual of Mental Disorders, 5th Edition (DSM-5) criteria for PTSD. One limitation is that the questionnaire is only validated for adults ages 18 years or older. Completion of the PCL-5 takes 5 to 10 minutes. The PCL-5 has strong internal consistency reliability (94%) and test-retest reliability (82%).31 With a cutoff score of 33 or higher, the sensitivity and specificity have been shown to be moderately high (74.5% and 70.6%, respectively).32
The Child and Adolescent Trauma Screen (CATS) is used to assess for potentially traumatic events and PTSD symptoms in children and adolescents. These symptoms are based on the DSM-5, and therefore the CATS can act as a useful diagnostic aid. The CATS is also available in Spanish, with both caregiver-report (for children ages 3-6 years or 7-17 years) and self-report (for ages 7-17 years) versions. Practical use of the PCL-5 and the CATS involves screening for PTSD symptoms, supporting a provisional diagnosis of PTSD, and monitoring PTSD symptom changes during and after treatment.
Memory and cognition
Cognitive screening is a first step in evaluating possible dementia and other neuropsychological disorders. The importance of brief cognitive screening in primary care cannot be understated, especially for an aging patient population. Although the Mini Mental Status Exam (MMSE) has been widely used among health care providers and researchers, we recommend the Montreal Cognitive Assessment (MoCA).
The MoCA is a simple, standalone cognitive screening tool validated for adults ages 55 to 85 years.33 The MoCA addresses many important cognitive domains, fits on one page, and can be administered by a trained provider in 10 minutes. Research also suggests that it has strong test-retest reliability and positive and negative predictive values for mild cognitive impairment and Alzheimer dementia, and it has been found to be more sensitive than the MMSE.34 We additionally recommend the MoCA as it measures several cognitive skills that are not addressed on the MMSE, including verbal fluency and abstraction.34 Scores below 25 are suggestive of cognitive impairment and should lead to a referral for neuropsychological testing.
The MoCA’s sensitivity for detecting cognitive impairment is high (94%), and specificity is low (42%).35 To ensure consistency and accuracy in administering the MoCA, certification is now required via an online training program through www.mocatest.org.
Adapting these screening tools to practice
These tools are not meant to be used at every appointment. Every practice is different, and each clinic or physician can tailor the use of these screening tools to the needs of the patient population, as concerns arise, or in collaboration with other providers. Additionally, these screening tools can be used in both integrated care and in private practice, to prompt a more thorough assessment or to aid in—and inform—treatment. Although some physicians choose to administer certain screening tools at each clinic visit, knowing about the availability of other tools can be useful in assessing various issues. The FIGURE can be used to aid in the clinical decision-making process.
- Robinson RL, Grabner M, Palli SR, et al. Covariates of depression and high utilizers of healthcare: impact on resource use and costs. J Psychosom Res. 2016,85:35-43.
- Fogarty CT, Sharma S, Chetty VK, et al. Mental health conditions are associated with increased health care utilization among urban family medicine patients. J Am Board Fam Med. 2008,21:398-407.
- Weissman JD, Russell D, Beasley J, et al. Relationships between adult emotional states and indicators of health care utilization: findings from the National Health Interview Survey 2006–2014. J Psychosom Res. 2016,91:75-81.
- Haddad M, Walters P. Mood disorders in primary care. Psychiatry. 2009,8:71-75.
- Mitchell AJ, Yadegarfar M, Gill J, et al. Case finding and screening clinical utility of the Patient Health Questionnaire (PHQ-9 and PHQ-2) for depression in primary care: a diagnostic metaanalysis of 40 studies. BJPsych Open. 2016,2:127-138.
- Siu AL and US Preventive Services Task Force. Screening for depression in adults. JAMA. 2016;315:380-387.
- Hirschfeld RM, Williams JB, Spitzer RL, et al. Development and validation of a screening instrument for bipolar spectrum disorder: the Mood Disorder Questionnaire. Am J Psychiatry. 2000;157:1873-1875.
- Hirschfeld RM, Cass AR, Holt DC, et al. Screening for bipolar disorder in patients treated for depression in a family medicine clinic. J Am Board Fam Med. 2005;18:233-239.
- Das AK, Olfson M, Gameroff MJ, et al. Screening for bipolar disorder in a primary care practice. JAMA. 2005;293:956-963.
- CDC. Suicide mortality in the United States, 1999-2017. www.cdc.gov/nchs/products/databriefs/db330.htm. Accessed October 23, 2020.
- Viguera AC, Milano N, Ralston L, et al. Comparison of electronic screening for suicidal risk with Patient Health Questionnaire Item 9 and the Columbia Suicide Severity Rating Scale in an outpatient psychiatric clinic. Psychosomatics. 2015;56:460-469.
- Spitzer RL, Kroenke K, Williams JBW, et al. A brief measure for assessing generalized anxiety disorder: the GAD-7. Arch Intern Med. 2006;166:1092-1097.
- Birmaher B, Khetarpal S, Brent D, et al. The Screen for Child Anxiety Related Emotional Disorders (SCARED): scale construction and psychometric characteristics. J Am Acad Chil Adolesc Psychiatry. 1997;36:545-553.
- DeSousa DA, Salum GA, Isolan LR, et al. Sensitivity and specificity of the Screen for Child Anxiety Related Emotional Disorders (SCARED): a community-based study. Child Psychiatry Hum Dev. 2013;44:391-399.
- Ford ES, Cunningham TJ, Giles WH, et al. Trends in insomnia and excessive daytime sleepiness among U.S. adults from 2002 to 2012. Sleep Med. 2015;16:372-378.
- Morin CM, LeBlanc M, Daley M, et al. Epidemiology of insomnia: prevalence, self-help treatments, consultations, and determinants of help-seeking behaviors. Sleep Med. 2006;7:123-130.
- Buysse DJ, Ancoli-Israel S, Edinger JD, et al. Recommendations for a standard research assessment of insomnia. Sleep. 2006;29:1155-1173.
- Martin JL, Hakim AD. Wrist actigraphy. Chest. 2011;139:1514-1527.
- Riemann D, Baglioni C, Bassetti C, et al. European guideline for the diagnosis and treatment of insomnia. J Sleep Res. 2017;26:675-700.
- Bastien CH, Vallières A, Morin CM. Validation of the Insomnia Severity Index as an outcome measure for insomnia research. Sleep Med. 2001;2:297-307.
- Wong ML, Lau KNT, Espie CA, et al. Psychometric properties of the Sleep Condition Indicator and Insomnia Severity Index in the evaluation of insomnia disorder. Sleep Med. 2017;33:76-81.
- Gagnon C, Bélanger L, Ivers H, et al. Validation of the Insomnia Severity Index in primary care. J Am Board Fam Med. 2013;26:701-710.
- Saunders JB, Aasland OG, Babor TF, et al. Development of the Alcohol Use Disorders Identification Test (AUDIT): WHO Collaborative Project on Early Detection of Persons with Harmful Alcohol Consumption. Addiction. 1993;88:791-804.
- Selin KH. Test-retest reliability of the Alcohol Use Disorder Identification Test in a general population sample. Alcohol Clin Exp Res. 2003;27:1428-1435.
- Bohn MJ, Babor TF, Kranzler HR. The Alcohol Use Disorders Identification Test (AUDIT): validation of a screening instrument for use in medical settings. J Stud Alcohol. 1995;56:423-432.
- Conigrave KM, Hall WD, Saunders JB. The AUDIT questionnaire: choosing a cut-off score. Addiction. 1995;90:1349-1356.
- Gomez A, Conde A, Santana JM, et al. Diagnostic usefulness of brief versions of Alcohol Use Identification Test (AUDIT) for detecting hazardous drinkers in primary care settings. J Stud Alcohol. 2005;66:305-308.
- Butler SF, Fernandez K, Benoit C, et al. Validation of the revised Screener and Opioid Assessment for Patients with Pain (SOAPPR). J Pain. 2008;9:360-372.
- Knight JR, Sherritt L, Shrier LA, et al. Validity of the CRAFFT substance abuse screening test among adolescent clinic patients. Arch Pediatr Adolesc Med. 2002;156:607-614.
- DHHS. Post-traumatic stress disorder (PTSD). https://archives.nih.gov/asites/report/09-09-2019/report.nih.gov/nihfactsheets/ViewFactSheetfdf8.html?csid=58&key=P#P. Accessed October 23,2020.
- Blevins CA, Weathers FW, Davis MT, et al. The Posttraumatic Stress Disorder Checklist for DSM-5 (PCL-5): development and initial psychometric evaluation. J Trauma Stress. 2015;28:489-498.
- Verhey R, Chilbanda D, Gibson L, et al. Validation of the Posttraumatic Stress Disorder Checklist- 5 (PCL-5) in a primary care population with high HIV prevalence in Zimbabwe. BMC Psychiatry. 2018;18:109.
- Nasreddine ZS, Phillips NA, Bédirian V, et al. The Montreal Cognitive Assessment, MoCA: a brief screening tool for mild cognitive impairment. J Am Geriatr Soc. 2005;53:695-699.
- Stewart S, O’Riley A, Edelstein B, et al. A preliminary comparison of three cognitive screening instruments in long term care: the MMSE, SLUMS, and MoCA. Clin Gerontol. 2012;35:57-75.
- Godefroy O, Fickl A, Roussel M, et al. Is the Montreal Cognitive Assessment superior to the Mini-Mental State Examination to detect poststroke cognitive impairment? A study with neuropsychological evaluation. Stroke. 2011;42:1712-1716.
- Robinson RL, Grabner M, Palli SR, et al. Covariates of depression and high utilizers of healthcare: impact on resource use and costs. J Psychosom Res. 2016,85:35-43.
- Fogarty CT, Sharma S, Chetty VK, et al. Mental health conditions are associated with increased health care utilization among urban family medicine patients. J Am Board Fam Med. 2008,21:398-407.
- Weissman JD, Russell D, Beasley J, et al. Relationships between adult emotional states and indicators of health care utilization: findings from the National Health Interview Survey 2006–2014. J Psychosom Res. 2016,91:75-81.
- Haddad M, Walters P. Mood disorders in primary care. Psychiatry. 2009,8:71-75.
- Mitchell AJ, Yadegarfar M, Gill J, et al. Case finding and screening clinical utility of the Patient Health Questionnaire (PHQ-9 and PHQ-2) for depression in primary care: a diagnostic metaanalysis of 40 studies. BJPsych Open. 2016,2:127-138.
- Siu AL and US Preventive Services Task Force. Screening for depression in adults. JAMA. 2016;315:380-387.
- Hirschfeld RM, Williams JB, Spitzer RL, et al. Development and validation of a screening instrument for bipolar spectrum disorder: the Mood Disorder Questionnaire. Am J Psychiatry. 2000;157:1873-1875.
- Hirschfeld RM, Cass AR, Holt DC, et al. Screening for bipolar disorder in patients treated for depression in a family medicine clinic. J Am Board Fam Med. 2005;18:233-239.
- Das AK, Olfson M, Gameroff MJ, et al. Screening for bipolar disorder in a primary care practice. JAMA. 2005;293:956-963.
- CDC. Suicide mortality in the United States, 1999-2017. www.cdc.gov/nchs/products/databriefs/db330.htm. Accessed October 23, 2020.
- Viguera AC, Milano N, Ralston L, et al. Comparison of electronic screening for suicidal risk with Patient Health Questionnaire Item 9 and the Columbia Suicide Severity Rating Scale in an outpatient psychiatric clinic. Psychosomatics. 2015;56:460-469.
- Spitzer RL, Kroenke K, Williams JBW, et al. A brief measure for assessing generalized anxiety disorder: the GAD-7. Arch Intern Med. 2006;166:1092-1097.
- Birmaher B, Khetarpal S, Brent D, et al. The Screen for Child Anxiety Related Emotional Disorders (SCARED): scale construction and psychometric characteristics. J Am Acad Chil Adolesc Psychiatry. 1997;36:545-553.
- DeSousa DA, Salum GA, Isolan LR, et al. Sensitivity and specificity of the Screen for Child Anxiety Related Emotional Disorders (SCARED): a community-based study. Child Psychiatry Hum Dev. 2013;44:391-399.
- Ford ES, Cunningham TJ, Giles WH, et al. Trends in insomnia and excessive daytime sleepiness among U.S. adults from 2002 to 2012. Sleep Med. 2015;16:372-378.
- Morin CM, LeBlanc M, Daley M, et al. Epidemiology of insomnia: prevalence, self-help treatments, consultations, and determinants of help-seeking behaviors. Sleep Med. 2006;7:123-130.
- Buysse DJ, Ancoli-Israel S, Edinger JD, et al. Recommendations for a standard research assessment of insomnia. Sleep. 2006;29:1155-1173.
- Martin JL, Hakim AD. Wrist actigraphy. Chest. 2011;139:1514-1527.
- Riemann D, Baglioni C, Bassetti C, et al. European guideline for the diagnosis and treatment of insomnia. J Sleep Res. 2017;26:675-700.
- Bastien CH, Vallières A, Morin CM. Validation of the Insomnia Severity Index as an outcome measure for insomnia research. Sleep Med. 2001;2:297-307.
- Wong ML, Lau KNT, Espie CA, et al. Psychometric properties of the Sleep Condition Indicator and Insomnia Severity Index in the evaluation of insomnia disorder. Sleep Med. 2017;33:76-81.
- Gagnon C, Bélanger L, Ivers H, et al. Validation of the Insomnia Severity Index in primary care. J Am Board Fam Med. 2013;26:701-710.
- Saunders JB, Aasland OG, Babor TF, et al. Development of the Alcohol Use Disorders Identification Test (AUDIT): WHO Collaborative Project on Early Detection of Persons with Harmful Alcohol Consumption. Addiction. 1993;88:791-804.
- Selin KH. Test-retest reliability of the Alcohol Use Disorder Identification Test in a general population sample. Alcohol Clin Exp Res. 2003;27:1428-1435.
- Bohn MJ, Babor TF, Kranzler HR. The Alcohol Use Disorders Identification Test (AUDIT): validation of a screening instrument for use in medical settings. J Stud Alcohol. 1995;56:423-432.
- Conigrave KM, Hall WD, Saunders JB. The AUDIT questionnaire: choosing a cut-off score. Addiction. 1995;90:1349-1356.
- Gomez A, Conde A, Santana JM, et al. Diagnostic usefulness of brief versions of Alcohol Use Identification Test (AUDIT) for detecting hazardous drinkers in primary care settings. J Stud Alcohol. 2005;66:305-308.
- Butler SF, Fernandez K, Benoit C, et al. Validation of the revised Screener and Opioid Assessment for Patients with Pain (SOAPPR). J Pain. 2008;9:360-372.
- Knight JR, Sherritt L, Shrier LA, et al. Validity of the CRAFFT substance abuse screening test among adolescent clinic patients. Arch Pediatr Adolesc Med. 2002;156:607-614.
- DHHS. Post-traumatic stress disorder (PTSD). https://archives.nih.gov/asites/report/09-09-2019/report.nih.gov/nihfactsheets/ViewFactSheetfdf8.html?csid=58&key=P#P. Accessed October 23,2020.
- Blevins CA, Weathers FW, Davis MT, et al. The Posttraumatic Stress Disorder Checklist for DSM-5 (PCL-5): development and initial psychometric evaluation. J Trauma Stress. 2015;28:489-498.
- Verhey R, Chilbanda D, Gibson L, et al. Validation of the Posttraumatic Stress Disorder Checklist- 5 (PCL-5) in a primary care population with high HIV prevalence in Zimbabwe. BMC Psychiatry. 2018;18:109.
- Nasreddine ZS, Phillips NA, Bédirian V, et al. The Montreal Cognitive Assessment, MoCA: a brief screening tool for mild cognitive impairment. J Am Geriatr Soc. 2005;53:695-699.
- Stewart S, O’Riley A, Edelstein B, et al. A preliminary comparison of three cognitive screening instruments in long term care: the MMSE, SLUMS, and MoCA. Clin Gerontol. 2012;35:57-75.
- Godefroy O, Fickl A, Roussel M, et al. Is the Montreal Cognitive Assessment superior to the Mini-Mental State Examination to detect poststroke cognitive impairment? A study with neuropsychological evaluation. Stroke. 2011;42:1712-1716.
A case of BV during pregnancy: Best management approach
CASE Pregnant woman with abnormal vaginal discharge
A 26-year-old woman (G2P1001) at 24 weeks of gestation requests evaluation for increased frothy, whitish-gray vaginal discharge with a fishy odor. She notes that her underclothes constantly feel damp. The vaginal pH is 4.5, and the amine test is positive.
- What is the most likely diagnosis?
- What obstetrical complications may be associated with this condition?
- How should her condition be treated?
Meet our perpetrator
Bacterial vaginosis (BV) is one of the most common conditions associated with vaginal discharge among women of reproductive age. It is characterized by a polymicrobial alteration of the vaginal microbiome, and most distinctly, a relative absence of vaginal lactobacilli. This review discusses the microbiology, epidemiology, specific obstetric and gynecologic complications, clinical manifestations, diagnosis, and treatment of BV.
The role of vaginal flora
Estrogen has a fundamental role in regulating the normal state of the vagina. In a woman’s reproductive years, estrogen increases glycogen in the vaginal epithelial cells, and the increased glycogen concentration promotes colonization by lactobacilli. The lack of estrogen in pre- and postmenopausal women inhibits the growth of the vaginal lactobacilli, leading to a high vaginal pH, which facilitates the growth of bacteria, particularly anaerobes, that can cause BV.
The vaginal microbiome is polymicrobial and has been classified into at least 5 community state types (CSTs). Four CSTs are dominated by lactobacilli. A fifth CST is characterized by the absence of lactobacilli and high concentrations of obligate or facultative anaerobes.1 The hydrogen peroxide–producing lactobacilli predominate in normal vaginal flora and make up 70% to 90% of the total microbiome. These hydrogen peroxide–producing lactobacilli are associated with reduced vaginal proinflammatory cytokines and a highly acidic vaginal pH. Both factors defend against sexually transmitted infections (STIs).2
BV is a polymicrobial disorder marked by the significant reduction in the number of vaginal lactobacilli (FIGURE 1). A recent study showed that BV is associated first with a decrease in Lactobacillus crispatus, followed by increase in Prevotella bivia, Gardnerella vaginalis, Atopobium vaginae, and Megasphaera type 1.3 The polymicrobial load is increased by a factor of up to 1,000, compared with normal vaginal flora.4 BV should be considered a biofilm infection caused by adherence of G vaginalis to the vaginal epithelium.5 This biofilm creates a favorable environment for the overgrowth of obligate anaerobic bacteria.
BMI factors into epidemiology
BV is the leading cause of vaginal discharge in reproductive-age women. In the United States, the National Health and Nutrition Examination Survey estimated a prevalence of 29% in the general population and 50% in Black women aged 14 to 49 years.6 In 2013, Kenyon and colleagues performed a systematic review to assess the worldwide epidemiology of BV, and the prevalence varied by country. Within the US population, rates were highest among non-Hispanic, Black women.7 Brookheart and colleagues demonstrated that, even after controlling for race, overweight and obese women had a higher frequency of BV compared with leaner women. In this investigation, the overall prevalence of BV was 28.1%. When categorized by body mass index (BMI), the prevalence was 21.3% in lean women, 30.4% in overweight women, and 34.5% in obese women (P<.001). The authors also found that Black women had a higher prevalence, independent of BMI, compared with White women.8
Complications may occur. BV is notable for having several serious sequelae in both pregnant and nonpregnant women. For obstetric patients, these sequelae include an increased risk of preterm birth; first trimester spontaneous abortion, particularly in the setting of in vitro fertilization; intra-amniotic infection; and endometritis.9,10 The risk of preterm birth increases by a factor of 2 in infected women; however, most women with BV do not deliver preterm.4 The risk of endometritis is increased 6-fold in women with BV.11 Nonpregnant women with BV are at increased risk for pelvic inflammatory disease, postoperative infections, and an increased susceptibility to STIs such as chlamydia, gonorrhea, herpes simplex virus, and HIV.12-15 The risk for vaginal-cuff cellulitis and abscess after hysterectomy is increased 6-fold in the setting of BV.16
Continue to: Clinical manifestations...
Clinical manifestations
BV is characterized by a milky, homogenous, and malodorous vaginal discharge accompanied by vulvovaginal discomfort and vulvar irritation. Vaginal inflammation typically is absent. The associated odor is fishy, and this odor is accentuated when potassium hydroxide (KOH) is added to the vaginal discharge (amine or “whiff” test) or after the patient has coitus. The distinctive odor is due to the release of organic acids and polyamines that are byproducts of anaerobic bacterial metabolism of putrescine and cadaverine. This release is enhanced by exposure of vaginal secretions to alkaline substances such as KOH or semen.
Diagnostic tests and criteria. The diagnosis of BV is made using Amsel criteria or Gram stain with Nugent scoring; bacterial culture is not recommended. Amsel criteria include:
- homogenous, thin, white-gray discharge
- >20% clue cells on saline microscopy (FIGURE 2)
- a pH >4.5 of vaginal fluid
- positive KOH whiff test.
For diagnosis, 3 of the 4 Amsel criteria must be present.17 Gram stain with Nugent score typically is used for research purposes. Nugent scoring assigns a value to different bacterial morphotypes on Gram stain of vaginal secretions. A score of 7 to 10 is consistent with BV.18
Oral and topical treatments
Treatment is recommended for symptomatic patients. Treatment may reduce the risk of transmission and acquisition of other STIs. The TABLE summarizes Centers for Disease Control and Prevention (CDC) guidelines for BV treatment,19 with options including both oral and topical regimens. Oral and topical metronidazole and oral and topical clindamycin are equally effective at eradicating the local source of infection20; however, only oral metronidazole and oral clindamycin are effective in preventing the systemic complications of BV. Oral metronidazole has more adverse effects than oral clindamycin—including nausea, vomiting, diarrhea, and a disulfiram-like reaction (characterized by flushing, dizziness, throbbing headache, chest and abdominal discomfort, and a distinct hangover effect in addition to nausea and vomiting). However, oral clindamycin can cause antibiotic-associated colitis and is more expensive than metronidazole.
Currently, there are no single-dose regimens for the treatment of BV readily available in the United States. Secnidazole, a 5-nitroimidazole with a longer half-life than metronidazole, (17 vs 8 hours) has been used as therapy in Europe and Asia but is not yet available commercially in the United States.21 Hiller and colleagues found that 1 g and 2 g secnidazole oral granules were superior to placebo in treating BV.22 A larger randomized trial comparing this regimen to standard treatment is necessary before this therapy is adopted as the standard of care.
Continue to: Managing recurrent disease...
Managing recurrent disease, a common problem. Bradshaw and colleagues noted that, although the initial treatment of BV is effective in approximately 80% of women, up to 50% have a recurrence within 12 months.23 Data are limited regarding optimal treatment for recurrent infections; however, most regimens consist of some form of suppressive therapy. One regimen includes one full applicator of metronidazole vaginal gel 0.75% twice weekly for 6 months.24 A second regimen consists of vaginal boric acid capsules 600 mg once daily at bedtime for 21 days. Upon completion of boric acid therapy, metronidazole vaginal gel 0.75% should be administered twice weekly for 6 months.25 A third option is oral metronidazole 2 g and fluconazole 250 mg once every month.26 Of note, boric acid can be fatal if consumed orally and is not recommended during pregnancy.
Most recently, a randomized trial evaluated the ability of L crispatus to prevent BV recurrence. After completion of standard treatment therapy with metronidazole, women were randomly assigned to receive vaginally administered L crispatus (152 patients) or placebo (76 patients) for 11 weeks. In the intention-to-treat population, recurrent BV occurred in 30% of patients in the L crispatus group and 45% of patients in the placebo group. The use of L crispatus significantly reduced recurrence of BV by one-third (P = .01; 95% confidence interval [CI], 0.44–0.87).27 These findings are encouraging; however, confirmatory studies are needed before adopting this as standard of care.
Should sexual partners be treated as well? BV has not traditionally been considered an STI, and the CDC does not currently recommend treatment of partners of women who have BV. However, in women who have sex with women, the rate of BV concordance is high, and in women who have sex with men, coitus can clearly influence disease activity. Therefore, in patients with refractory BV, we recommend treatment of the sexual partner(s) with metronidazole 500 mg orally twice daily for 7 days. For women having sex with men, we also recommend consistent use of condoms, at least until the patient’s infection is better controlled.28
CASE Resolved
The patient’s clinical findings are indicative of BV. This condition is associated with an increased risk of preterm delivery and intrapartum and postpartum infection. To reduce the risk of these systemic complications, she was treated with oral metronidazole 500 mg twice daily for 7 days. Within 1 week of completing treatment, she noted complete resolution of the malodorous discharge. ●
- Smith SB, Ravel J. The vaginal microbiota, host defence and reproductive physiology. J Physiol. 2017;595:451-463.
- Mitchell C, Fredricks D, Agnew K, et al. Hydrogen peroxide-producing lactobacilli are associated with lower levels of vaginal interleukin-1β, independent of bacterial vaginosis. Sex Transm Infect. 2015;42:358-363.
- Munzy CA, Blanchard E, Taylor CM, et al. Identification of key bacteria involved in the induction of incident bacterial vaginosis: a prospective study. J Infect. 2018;218:966-978.
- Paavonen J, Brunham RC. Bacterial vaginosis and desquamative inflammatory vaginitis. N Engl J Med. 2018; 379:2246-2254.
- Hardy L, Jespers V, Dahchour N, et al. Unravelling the bacterial vaginosis-associated biofilm: a multiplex Gardnerella vaginalis and Atopobium vaginae fluorescence in situ hybridization assay using peptide nucleic acid probes. PloS One. 2015;10:E0136658.
- Allswoth JE, Peipert JF. Prevalence of bacterial vaginosis: 2001-2004 national health and nutrition examination survey data. Obstet Gynecol. 2007;109:114-120.
- Kenyon C, Colebunders R, Crucitti T. The global epidemiology of bacterial vaginosis: a systematic review. Am J Obstet Gynecol. 2013;209:505-523.
- Brookheart RT, Lewis WG, Peipert JF, et al. Association between obesity and bacterial vaginosis as assessed by Nugent score. Am J Obstet Gynecol. 2019;220:476.e1-476.e11.
- Onderdonk AB, Delaney ML, Fichorova RN. The human microbiome during bacterial vaginosis. Clin Microbiol Rev. 2016;29:223-238.
- Brown RG, Marchesi JR, Lee YS, et al. Vaginal dysbiosis increases risk of preterm fetal membrane rupture, neonatal sepsis and is exacerbated by erythromycin. BMC Med. 2018;16:9.
- Watts DH, Eschenbach DA, Kenny GE. Early postpartum endometritis: the role of bacteria, genital mycoplasmas, and chlamydia trachomatis. Obstet Gynecol. 1989;73:52-60.
- Balkus JE, Richardson BA, Rabe LK, et al. Bacterial vaginosis and the risk of Trichomonas vaginalis acquisition among HIV1-negative women. Sex Transm Dis. 2014;41:123-128.
- Cherpes TL, Meyn LA, Krohn MA, et al. Association between acquisition of herpes simplex virus type 2 in women and bacterial vaginosis. Clin Infect Dis. 2003;37:319-325.
- Wiesenfeld HC, Hillier SL, Krohn MA, et al. Bacterial vaginosis is a strong predictor of Neisseria gonorrhoeae and Chlamydia trachomatis infection. Clin Infect Dis. 2003;36:663-668.
- Myer L, Denny L, Telerant R, et al. Bacterial vaginosis and susceptibility to HIV infection in South African women: a nested case-control study. J Infect. 2005;192:1372-1380.
- Soper DE, Bump RC, Hurt WG. Bacterial vaginosis and trichomoniasis vaginitis are risk factors for cuff cellulitis after abdominal hysterectomy. Am J Obstet Gynecol. 1990;163:1061-1121.
- Amsel R, Totten PA, Spiegel CA, et al. Nonspecific vaginitis. diagnostic criteria and microbial and epidemiologic associations. Am J Med. 1983;74:14-22.
- Nugent RP, Krohn MA, Hillier SL. Reliability of diagnosing bacterial vaginosis is improved by a standardized method of gram stain interpretation. J Clin Microbiol. 1991;29:297-301.
- Bacterial vaginosis. Centers for Disease Control and Prevention website. Updated June 4, 2015. Accessed December 9, 2020. https://www.cdc.gov/std/tg2015/bv.htm.
- Oduyebo OO, Anorlu RI, Ogunsola FT. The effects of antimicrobial therapy on bacterial vaginosis in non-pregnant women. Cochrane Database Syst Rev. 2009:CD006055.
- Videau D, Niel G, Siboulet A, et al. Secnidazole. a 5-nitroimidazole derivative with a long half-life. Br J Vener Dis. 1978;54:77-80.
- Hillier SL, Nyirjesy P, Waldbaum AS, et al. Secnidazole treatment of bacterial vaginosis: a randomized controlled trial. Obstet Gynecol. 2017;130:379-386.
- Bradshaw CS, Morton AN, Hocking J, et al. High recurrence rates of bacterial vaginosis over the course of 12 months after oral metronidazole therapy and factors associated with recurrence. J Infect. 2006;193:1478-1486.
- Sobel JD, Ferris D, Schwebke J, et al. Suppressive antibacterial therapy with 0.75% metronidazole vaginal gel to prevent recurrent bacterial vaginosis. Am J Obstet Gynecol. 2006;194:1283-1289.
- Reichman O, Akins R, Sobel JD. Boric acid addition to suppressive antimicrobial therapy for recurrent bacterial vaginosis. Sex Transm Dis. 2009;36:732-734.
- McClelland RS, Richardson BA, Hassan WM, et al. Improvement of vaginal health for Kenyan women at risk for acquisition of human immunodeficiency virus type 1: results of a randomized trial. J Infect. 2008;197:1361-1368.
- Cohen CR, Wierzbicki MR, French AL, et al. Randomized trial of lactin-v to prevent recurrence of bacterial vaginosis. N Engl J Med. 2020;382:906-915.
- Barbieri RL. Effective treatment of recurrent bacterial vaginosis. OBG Manag. 2017;29:7-12.
CASE Pregnant woman with abnormal vaginal discharge
A 26-year-old woman (G2P1001) at 24 weeks of gestation requests evaluation for increased frothy, whitish-gray vaginal discharge with a fishy odor. She notes that her underclothes constantly feel damp. The vaginal pH is 4.5, and the amine test is positive.
- What is the most likely diagnosis?
- What obstetrical complications may be associated with this condition?
- How should her condition be treated?
Meet our perpetrator
Bacterial vaginosis (BV) is one of the most common conditions associated with vaginal discharge among women of reproductive age. It is characterized by a polymicrobial alteration of the vaginal microbiome, and most distinctly, a relative absence of vaginal lactobacilli. This review discusses the microbiology, epidemiology, specific obstetric and gynecologic complications, clinical manifestations, diagnosis, and treatment of BV.
The role of vaginal flora
Estrogen has a fundamental role in regulating the normal state of the vagina. In a woman’s reproductive years, estrogen increases glycogen in the vaginal epithelial cells, and the increased glycogen concentration promotes colonization by lactobacilli. The lack of estrogen in pre- and postmenopausal women inhibits the growth of the vaginal lactobacilli, leading to a high vaginal pH, which facilitates the growth of bacteria, particularly anaerobes, that can cause BV.
The vaginal microbiome is polymicrobial and has been classified into at least 5 community state types (CSTs). Four CSTs are dominated by lactobacilli. A fifth CST is characterized by the absence of lactobacilli and high concentrations of obligate or facultative anaerobes.1 The hydrogen peroxide–producing lactobacilli predominate in normal vaginal flora and make up 70% to 90% of the total microbiome. These hydrogen peroxide–producing lactobacilli are associated with reduced vaginal proinflammatory cytokines and a highly acidic vaginal pH. Both factors defend against sexually transmitted infections (STIs).2
BV is a polymicrobial disorder marked by the significant reduction in the number of vaginal lactobacilli (FIGURE 1). A recent study showed that BV is associated first with a decrease in Lactobacillus crispatus, followed by increase in Prevotella bivia, Gardnerella vaginalis, Atopobium vaginae, and Megasphaera type 1.3 The polymicrobial load is increased by a factor of up to 1,000, compared with normal vaginal flora.4 BV should be considered a biofilm infection caused by adherence of G vaginalis to the vaginal epithelium.5 This biofilm creates a favorable environment for the overgrowth of obligate anaerobic bacteria.
BMI factors into epidemiology
BV is the leading cause of vaginal discharge in reproductive-age women. In the United States, the National Health and Nutrition Examination Survey estimated a prevalence of 29% in the general population and 50% in Black women aged 14 to 49 years.6 In 2013, Kenyon and colleagues performed a systematic review to assess the worldwide epidemiology of BV, and the prevalence varied by country. Within the US population, rates were highest among non-Hispanic, Black women.7 Brookheart and colleagues demonstrated that, even after controlling for race, overweight and obese women had a higher frequency of BV compared with leaner women. In this investigation, the overall prevalence of BV was 28.1%. When categorized by body mass index (BMI), the prevalence was 21.3% in lean women, 30.4% in overweight women, and 34.5% in obese women (P<.001). The authors also found that Black women had a higher prevalence, independent of BMI, compared with White women.8
Complications may occur. BV is notable for having several serious sequelae in both pregnant and nonpregnant women. For obstetric patients, these sequelae include an increased risk of preterm birth; first trimester spontaneous abortion, particularly in the setting of in vitro fertilization; intra-amniotic infection; and endometritis.9,10 The risk of preterm birth increases by a factor of 2 in infected women; however, most women with BV do not deliver preterm.4 The risk of endometritis is increased 6-fold in women with BV.11 Nonpregnant women with BV are at increased risk for pelvic inflammatory disease, postoperative infections, and an increased susceptibility to STIs such as chlamydia, gonorrhea, herpes simplex virus, and HIV.12-15 The risk for vaginal-cuff cellulitis and abscess after hysterectomy is increased 6-fold in the setting of BV.16
Continue to: Clinical manifestations...
Clinical manifestations
BV is characterized by a milky, homogenous, and malodorous vaginal discharge accompanied by vulvovaginal discomfort and vulvar irritation. Vaginal inflammation typically is absent. The associated odor is fishy, and this odor is accentuated when potassium hydroxide (KOH) is added to the vaginal discharge (amine or “whiff” test) or after the patient has coitus. The distinctive odor is due to the release of organic acids and polyamines that are byproducts of anaerobic bacterial metabolism of putrescine and cadaverine. This release is enhanced by exposure of vaginal secretions to alkaline substances such as KOH or semen.
Diagnostic tests and criteria. The diagnosis of BV is made using Amsel criteria or Gram stain with Nugent scoring; bacterial culture is not recommended. Amsel criteria include:
- homogenous, thin, white-gray discharge
- >20% clue cells on saline microscopy (FIGURE 2)
- a pH >4.5 of vaginal fluid
- positive KOH whiff test.
For diagnosis, 3 of the 4 Amsel criteria must be present.17 Gram stain with Nugent score typically is used for research purposes. Nugent scoring assigns a value to different bacterial morphotypes on Gram stain of vaginal secretions. A score of 7 to 10 is consistent with BV.18
Oral and topical treatments
Treatment is recommended for symptomatic patients. Treatment may reduce the risk of transmission and acquisition of other STIs. The TABLE summarizes Centers for Disease Control and Prevention (CDC) guidelines for BV treatment,19 with options including both oral and topical regimens. Oral and topical metronidazole and oral and topical clindamycin are equally effective at eradicating the local source of infection20; however, only oral metronidazole and oral clindamycin are effective in preventing the systemic complications of BV. Oral metronidazole has more adverse effects than oral clindamycin—including nausea, vomiting, diarrhea, and a disulfiram-like reaction (characterized by flushing, dizziness, throbbing headache, chest and abdominal discomfort, and a distinct hangover effect in addition to nausea and vomiting). However, oral clindamycin can cause antibiotic-associated colitis and is more expensive than metronidazole.
Currently, there are no single-dose regimens for the treatment of BV readily available in the United States. Secnidazole, a 5-nitroimidazole with a longer half-life than metronidazole, (17 vs 8 hours) has been used as therapy in Europe and Asia but is not yet available commercially in the United States.21 Hiller and colleagues found that 1 g and 2 g secnidazole oral granules were superior to placebo in treating BV.22 A larger randomized trial comparing this regimen to standard treatment is necessary before this therapy is adopted as the standard of care.
Continue to: Managing recurrent disease...
Managing recurrent disease, a common problem. Bradshaw and colleagues noted that, although the initial treatment of BV is effective in approximately 80% of women, up to 50% have a recurrence within 12 months.23 Data are limited regarding optimal treatment for recurrent infections; however, most regimens consist of some form of suppressive therapy. One regimen includes one full applicator of metronidazole vaginal gel 0.75% twice weekly for 6 months.24 A second regimen consists of vaginal boric acid capsules 600 mg once daily at bedtime for 21 days. Upon completion of boric acid therapy, metronidazole vaginal gel 0.75% should be administered twice weekly for 6 months.25 A third option is oral metronidazole 2 g and fluconazole 250 mg once every month.26 Of note, boric acid can be fatal if consumed orally and is not recommended during pregnancy.
Most recently, a randomized trial evaluated the ability of L crispatus to prevent BV recurrence. After completion of standard treatment therapy with metronidazole, women were randomly assigned to receive vaginally administered L crispatus (152 patients) or placebo (76 patients) for 11 weeks. In the intention-to-treat population, recurrent BV occurred in 30% of patients in the L crispatus group and 45% of patients in the placebo group. The use of L crispatus significantly reduced recurrence of BV by one-third (P = .01; 95% confidence interval [CI], 0.44–0.87).27 These findings are encouraging; however, confirmatory studies are needed before adopting this as standard of care.
Should sexual partners be treated as well? BV has not traditionally been considered an STI, and the CDC does not currently recommend treatment of partners of women who have BV. However, in women who have sex with women, the rate of BV concordance is high, and in women who have sex with men, coitus can clearly influence disease activity. Therefore, in patients with refractory BV, we recommend treatment of the sexual partner(s) with metronidazole 500 mg orally twice daily for 7 days. For women having sex with men, we also recommend consistent use of condoms, at least until the patient’s infection is better controlled.28
CASE Resolved
The patient’s clinical findings are indicative of BV. This condition is associated with an increased risk of preterm delivery and intrapartum and postpartum infection. To reduce the risk of these systemic complications, she was treated with oral metronidazole 500 mg twice daily for 7 days. Within 1 week of completing treatment, she noted complete resolution of the malodorous discharge. ●
CASE Pregnant woman with abnormal vaginal discharge
A 26-year-old woman (G2P1001) at 24 weeks of gestation requests evaluation for increased frothy, whitish-gray vaginal discharge with a fishy odor. She notes that her underclothes constantly feel damp. The vaginal pH is 4.5, and the amine test is positive.
- What is the most likely diagnosis?
- What obstetrical complications may be associated with this condition?
- How should her condition be treated?
Meet our perpetrator
Bacterial vaginosis (BV) is one of the most common conditions associated with vaginal discharge among women of reproductive age. It is characterized by a polymicrobial alteration of the vaginal microbiome, and most distinctly, a relative absence of vaginal lactobacilli. This review discusses the microbiology, epidemiology, specific obstetric and gynecologic complications, clinical manifestations, diagnosis, and treatment of BV.
The role of vaginal flora
Estrogen has a fundamental role in regulating the normal state of the vagina. In a woman’s reproductive years, estrogen increases glycogen in the vaginal epithelial cells, and the increased glycogen concentration promotes colonization by lactobacilli. The lack of estrogen in pre- and postmenopausal women inhibits the growth of the vaginal lactobacilli, leading to a high vaginal pH, which facilitates the growth of bacteria, particularly anaerobes, that can cause BV.
The vaginal microbiome is polymicrobial and has been classified into at least 5 community state types (CSTs). Four CSTs are dominated by lactobacilli. A fifth CST is characterized by the absence of lactobacilli and high concentrations of obligate or facultative anaerobes.1 The hydrogen peroxide–producing lactobacilli predominate in normal vaginal flora and make up 70% to 90% of the total microbiome. These hydrogen peroxide–producing lactobacilli are associated with reduced vaginal proinflammatory cytokines and a highly acidic vaginal pH. Both factors defend against sexually transmitted infections (STIs).2
BV is a polymicrobial disorder marked by the significant reduction in the number of vaginal lactobacilli (FIGURE 1). A recent study showed that BV is associated first with a decrease in Lactobacillus crispatus, followed by increase in Prevotella bivia, Gardnerella vaginalis, Atopobium vaginae, and Megasphaera type 1.3 The polymicrobial load is increased by a factor of up to 1,000, compared with normal vaginal flora.4 BV should be considered a biofilm infection caused by adherence of G vaginalis to the vaginal epithelium.5 This biofilm creates a favorable environment for the overgrowth of obligate anaerobic bacteria.
BMI factors into epidemiology
BV is the leading cause of vaginal discharge in reproductive-age women. In the United States, the National Health and Nutrition Examination Survey estimated a prevalence of 29% in the general population and 50% in Black women aged 14 to 49 years.6 In 2013, Kenyon and colleagues performed a systematic review to assess the worldwide epidemiology of BV, and the prevalence varied by country. Within the US population, rates were highest among non-Hispanic, Black women.7 Brookheart and colleagues demonstrated that, even after controlling for race, overweight and obese women had a higher frequency of BV compared with leaner women. In this investigation, the overall prevalence of BV was 28.1%. When categorized by body mass index (BMI), the prevalence was 21.3% in lean women, 30.4% in overweight women, and 34.5% in obese women (P<.001). The authors also found that Black women had a higher prevalence, independent of BMI, compared with White women.8
Complications may occur. BV is notable for having several serious sequelae in both pregnant and nonpregnant women. For obstetric patients, these sequelae include an increased risk of preterm birth; first trimester spontaneous abortion, particularly in the setting of in vitro fertilization; intra-amniotic infection; and endometritis.9,10 The risk of preterm birth increases by a factor of 2 in infected women; however, most women with BV do not deliver preterm.4 The risk of endometritis is increased 6-fold in women with BV.11 Nonpregnant women with BV are at increased risk for pelvic inflammatory disease, postoperative infections, and an increased susceptibility to STIs such as chlamydia, gonorrhea, herpes simplex virus, and HIV.12-15 The risk for vaginal-cuff cellulitis and abscess after hysterectomy is increased 6-fold in the setting of BV.16
Continue to: Clinical manifestations...
Clinical manifestations
BV is characterized by a milky, homogenous, and malodorous vaginal discharge accompanied by vulvovaginal discomfort and vulvar irritation. Vaginal inflammation typically is absent. The associated odor is fishy, and this odor is accentuated when potassium hydroxide (KOH) is added to the vaginal discharge (amine or “whiff” test) or after the patient has coitus. The distinctive odor is due to the release of organic acids and polyamines that are byproducts of anaerobic bacterial metabolism of putrescine and cadaverine. This release is enhanced by exposure of vaginal secretions to alkaline substances such as KOH or semen.
Diagnostic tests and criteria. The diagnosis of BV is made using Amsel criteria or Gram stain with Nugent scoring; bacterial culture is not recommended. Amsel criteria include:
- homogenous, thin, white-gray discharge
- >20% clue cells on saline microscopy (FIGURE 2)
- a pH >4.5 of vaginal fluid
- positive KOH whiff test.
For diagnosis, 3 of the 4 Amsel criteria must be present.17 Gram stain with Nugent score typically is used for research purposes. Nugent scoring assigns a value to different bacterial morphotypes on Gram stain of vaginal secretions. A score of 7 to 10 is consistent with BV.18
Oral and topical treatments
Treatment is recommended for symptomatic patients. Treatment may reduce the risk of transmission and acquisition of other STIs. The TABLE summarizes Centers for Disease Control and Prevention (CDC) guidelines for BV treatment,19 with options including both oral and topical regimens. Oral and topical metronidazole and oral and topical clindamycin are equally effective at eradicating the local source of infection20; however, only oral metronidazole and oral clindamycin are effective in preventing the systemic complications of BV. Oral metronidazole has more adverse effects than oral clindamycin—including nausea, vomiting, diarrhea, and a disulfiram-like reaction (characterized by flushing, dizziness, throbbing headache, chest and abdominal discomfort, and a distinct hangover effect in addition to nausea and vomiting). However, oral clindamycin can cause antibiotic-associated colitis and is more expensive than metronidazole.
Currently, there are no single-dose regimens for the treatment of BV readily available in the United States. Secnidazole, a 5-nitroimidazole with a longer half-life than metronidazole, (17 vs 8 hours) has been used as therapy in Europe and Asia but is not yet available commercially in the United States.21 Hiller and colleagues found that 1 g and 2 g secnidazole oral granules were superior to placebo in treating BV.22 A larger randomized trial comparing this regimen to standard treatment is necessary before this therapy is adopted as the standard of care.
Continue to: Managing recurrent disease...
Managing recurrent disease, a common problem. Bradshaw and colleagues noted that, although the initial treatment of BV is effective in approximately 80% of women, up to 50% have a recurrence within 12 months.23 Data are limited regarding optimal treatment for recurrent infections; however, most regimens consist of some form of suppressive therapy. One regimen includes one full applicator of metronidazole vaginal gel 0.75% twice weekly for 6 months.24 A second regimen consists of vaginal boric acid capsules 600 mg once daily at bedtime for 21 days. Upon completion of boric acid therapy, metronidazole vaginal gel 0.75% should be administered twice weekly for 6 months.25 A third option is oral metronidazole 2 g and fluconazole 250 mg once every month.26 Of note, boric acid can be fatal if consumed orally and is not recommended during pregnancy.
Most recently, a randomized trial evaluated the ability of L crispatus to prevent BV recurrence. After completion of standard treatment therapy with metronidazole, women were randomly assigned to receive vaginally administered L crispatus (152 patients) or placebo (76 patients) for 11 weeks. In the intention-to-treat population, recurrent BV occurred in 30% of patients in the L crispatus group and 45% of patients in the placebo group. The use of L crispatus significantly reduced recurrence of BV by one-third (P = .01; 95% confidence interval [CI], 0.44–0.87).27 These findings are encouraging; however, confirmatory studies are needed before adopting this as standard of care.
Should sexual partners be treated as well? BV has not traditionally been considered an STI, and the CDC does not currently recommend treatment of partners of women who have BV. However, in women who have sex with women, the rate of BV concordance is high, and in women who have sex with men, coitus can clearly influence disease activity. Therefore, in patients with refractory BV, we recommend treatment of the sexual partner(s) with metronidazole 500 mg orally twice daily for 7 days. For women having sex with men, we also recommend consistent use of condoms, at least until the patient’s infection is better controlled.28
CASE Resolved
The patient’s clinical findings are indicative of BV. This condition is associated with an increased risk of preterm delivery and intrapartum and postpartum infection. To reduce the risk of these systemic complications, she was treated with oral metronidazole 500 mg twice daily for 7 days. Within 1 week of completing treatment, she noted complete resolution of the malodorous discharge. ●
- Smith SB, Ravel J. The vaginal microbiota, host defence and reproductive physiology. J Physiol. 2017;595:451-463.
- Mitchell C, Fredricks D, Agnew K, et al. Hydrogen peroxide-producing lactobacilli are associated with lower levels of vaginal interleukin-1β, independent of bacterial vaginosis. Sex Transm Infect. 2015;42:358-363.
- Munzy CA, Blanchard E, Taylor CM, et al. Identification of key bacteria involved in the induction of incident bacterial vaginosis: a prospective study. J Infect. 2018;218:966-978.
- Paavonen J, Brunham RC. Bacterial vaginosis and desquamative inflammatory vaginitis. N Engl J Med. 2018; 379:2246-2254.
- Hardy L, Jespers V, Dahchour N, et al. Unravelling the bacterial vaginosis-associated biofilm: a multiplex Gardnerella vaginalis and Atopobium vaginae fluorescence in situ hybridization assay using peptide nucleic acid probes. PloS One. 2015;10:E0136658.
- Allswoth JE, Peipert JF. Prevalence of bacterial vaginosis: 2001-2004 national health and nutrition examination survey data. Obstet Gynecol. 2007;109:114-120.
- Kenyon C, Colebunders R, Crucitti T. The global epidemiology of bacterial vaginosis: a systematic review. Am J Obstet Gynecol. 2013;209:505-523.
- Brookheart RT, Lewis WG, Peipert JF, et al. Association between obesity and bacterial vaginosis as assessed by Nugent score. Am J Obstet Gynecol. 2019;220:476.e1-476.e11.
- Onderdonk AB, Delaney ML, Fichorova RN. The human microbiome during bacterial vaginosis. Clin Microbiol Rev. 2016;29:223-238.
- Brown RG, Marchesi JR, Lee YS, et al. Vaginal dysbiosis increases risk of preterm fetal membrane rupture, neonatal sepsis and is exacerbated by erythromycin. BMC Med. 2018;16:9.
- Watts DH, Eschenbach DA, Kenny GE. Early postpartum endometritis: the role of bacteria, genital mycoplasmas, and chlamydia trachomatis. Obstet Gynecol. 1989;73:52-60.
- Balkus JE, Richardson BA, Rabe LK, et al. Bacterial vaginosis and the risk of Trichomonas vaginalis acquisition among HIV1-negative women. Sex Transm Dis. 2014;41:123-128.
- Cherpes TL, Meyn LA, Krohn MA, et al. Association between acquisition of herpes simplex virus type 2 in women and bacterial vaginosis. Clin Infect Dis. 2003;37:319-325.
- Wiesenfeld HC, Hillier SL, Krohn MA, et al. Bacterial vaginosis is a strong predictor of Neisseria gonorrhoeae and Chlamydia trachomatis infection. Clin Infect Dis. 2003;36:663-668.
- Myer L, Denny L, Telerant R, et al. Bacterial vaginosis and susceptibility to HIV infection in South African women: a nested case-control study. J Infect. 2005;192:1372-1380.
- Soper DE, Bump RC, Hurt WG. Bacterial vaginosis and trichomoniasis vaginitis are risk factors for cuff cellulitis after abdominal hysterectomy. Am J Obstet Gynecol. 1990;163:1061-1121.
- Amsel R, Totten PA, Spiegel CA, et al. Nonspecific vaginitis. diagnostic criteria and microbial and epidemiologic associations. Am J Med. 1983;74:14-22.
- Nugent RP, Krohn MA, Hillier SL. Reliability of diagnosing bacterial vaginosis is improved by a standardized method of gram stain interpretation. J Clin Microbiol. 1991;29:297-301.
- Bacterial vaginosis. Centers for Disease Control and Prevention website. Updated June 4, 2015. Accessed December 9, 2020. https://www.cdc.gov/std/tg2015/bv.htm.
- Oduyebo OO, Anorlu RI, Ogunsola FT. The effects of antimicrobial therapy on bacterial vaginosis in non-pregnant women. Cochrane Database Syst Rev. 2009:CD006055.
- Videau D, Niel G, Siboulet A, et al. Secnidazole. a 5-nitroimidazole derivative with a long half-life. Br J Vener Dis. 1978;54:77-80.
- Hillier SL, Nyirjesy P, Waldbaum AS, et al. Secnidazole treatment of bacterial vaginosis: a randomized controlled trial. Obstet Gynecol. 2017;130:379-386.
- Bradshaw CS, Morton AN, Hocking J, et al. High recurrence rates of bacterial vaginosis over the course of 12 months after oral metronidazole therapy and factors associated with recurrence. J Infect. 2006;193:1478-1486.
- Sobel JD, Ferris D, Schwebke J, et al. Suppressive antibacterial therapy with 0.75% metronidazole vaginal gel to prevent recurrent bacterial vaginosis. Am J Obstet Gynecol. 2006;194:1283-1289.
- Reichman O, Akins R, Sobel JD. Boric acid addition to suppressive antimicrobial therapy for recurrent bacterial vaginosis. Sex Transm Dis. 2009;36:732-734.
- McClelland RS, Richardson BA, Hassan WM, et al. Improvement of vaginal health for Kenyan women at risk for acquisition of human immunodeficiency virus type 1: results of a randomized trial. J Infect. 2008;197:1361-1368.
- Cohen CR, Wierzbicki MR, French AL, et al. Randomized trial of lactin-v to prevent recurrence of bacterial vaginosis. N Engl J Med. 2020;382:906-915.
- Barbieri RL. Effective treatment of recurrent bacterial vaginosis. OBG Manag. 2017;29:7-12.
- Smith SB, Ravel J. The vaginal microbiota, host defence and reproductive physiology. J Physiol. 2017;595:451-463.
- Mitchell C, Fredricks D, Agnew K, et al. Hydrogen peroxide-producing lactobacilli are associated with lower levels of vaginal interleukin-1β, independent of bacterial vaginosis. Sex Transm Infect. 2015;42:358-363.
- Munzy CA, Blanchard E, Taylor CM, et al. Identification of key bacteria involved in the induction of incident bacterial vaginosis: a prospective study. J Infect. 2018;218:966-978.
- Paavonen J, Brunham RC. Bacterial vaginosis and desquamative inflammatory vaginitis. N Engl J Med. 2018; 379:2246-2254.
- Hardy L, Jespers V, Dahchour N, et al. Unravelling the bacterial vaginosis-associated biofilm: a multiplex Gardnerella vaginalis and Atopobium vaginae fluorescence in situ hybridization assay using peptide nucleic acid probes. PloS One. 2015;10:E0136658.
- Allswoth JE, Peipert JF. Prevalence of bacterial vaginosis: 2001-2004 national health and nutrition examination survey data. Obstet Gynecol. 2007;109:114-120.
- Kenyon C, Colebunders R, Crucitti T. The global epidemiology of bacterial vaginosis: a systematic review. Am J Obstet Gynecol. 2013;209:505-523.
- Brookheart RT, Lewis WG, Peipert JF, et al. Association between obesity and bacterial vaginosis as assessed by Nugent score. Am J Obstet Gynecol. 2019;220:476.e1-476.e11.
- Onderdonk AB, Delaney ML, Fichorova RN. The human microbiome during bacterial vaginosis. Clin Microbiol Rev. 2016;29:223-238.
- Brown RG, Marchesi JR, Lee YS, et al. Vaginal dysbiosis increases risk of preterm fetal membrane rupture, neonatal sepsis and is exacerbated by erythromycin. BMC Med. 2018;16:9.
- Watts DH, Eschenbach DA, Kenny GE. Early postpartum endometritis: the role of bacteria, genital mycoplasmas, and chlamydia trachomatis. Obstet Gynecol. 1989;73:52-60.
- Balkus JE, Richardson BA, Rabe LK, et al. Bacterial vaginosis and the risk of Trichomonas vaginalis acquisition among HIV1-negative women. Sex Transm Dis. 2014;41:123-128.
- Cherpes TL, Meyn LA, Krohn MA, et al. Association between acquisition of herpes simplex virus type 2 in women and bacterial vaginosis. Clin Infect Dis. 2003;37:319-325.
- Wiesenfeld HC, Hillier SL, Krohn MA, et al. Bacterial vaginosis is a strong predictor of Neisseria gonorrhoeae and Chlamydia trachomatis infection. Clin Infect Dis. 2003;36:663-668.
- Myer L, Denny L, Telerant R, et al. Bacterial vaginosis and susceptibility to HIV infection in South African women: a nested case-control study. J Infect. 2005;192:1372-1380.
- Soper DE, Bump RC, Hurt WG. Bacterial vaginosis and trichomoniasis vaginitis are risk factors for cuff cellulitis after abdominal hysterectomy. Am J Obstet Gynecol. 1990;163:1061-1121.
- Amsel R, Totten PA, Spiegel CA, et al. Nonspecific vaginitis. diagnostic criteria and microbial and epidemiologic associations. Am J Med. 1983;74:14-22.
- Nugent RP, Krohn MA, Hillier SL. Reliability of diagnosing bacterial vaginosis is improved by a standardized method of gram stain interpretation. J Clin Microbiol. 1991;29:297-301.
- Bacterial vaginosis. Centers for Disease Control and Prevention website. Updated June 4, 2015. Accessed December 9, 2020. https://www.cdc.gov/std/tg2015/bv.htm.
- Oduyebo OO, Anorlu RI, Ogunsola FT. The effects of antimicrobial therapy on bacterial vaginosis in non-pregnant women. Cochrane Database Syst Rev. 2009:CD006055.
- Videau D, Niel G, Siboulet A, et al. Secnidazole. a 5-nitroimidazole derivative with a long half-life. Br J Vener Dis. 1978;54:77-80.
- Hillier SL, Nyirjesy P, Waldbaum AS, et al. Secnidazole treatment of bacterial vaginosis: a randomized controlled trial. Obstet Gynecol. 2017;130:379-386.
- Bradshaw CS, Morton AN, Hocking J, et al. High recurrence rates of bacterial vaginosis over the course of 12 months after oral metronidazole therapy and factors associated with recurrence. J Infect. 2006;193:1478-1486.
- Sobel JD, Ferris D, Schwebke J, et al. Suppressive antibacterial therapy with 0.75% metronidazole vaginal gel to prevent recurrent bacterial vaginosis. Am J Obstet Gynecol. 2006;194:1283-1289.
- Reichman O, Akins R, Sobel JD. Boric acid addition to suppressive antimicrobial therapy for recurrent bacterial vaginosis. Sex Transm Dis. 2009;36:732-734.
- McClelland RS, Richardson BA, Hassan WM, et al. Improvement of vaginal health for Kenyan women at risk for acquisition of human immunodeficiency virus type 1: results of a randomized trial. J Infect. 2008;197:1361-1368.
- Cohen CR, Wierzbicki MR, French AL, et al. Randomized trial of lactin-v to prevent recurrence of bacterial vaginosis. N Engl J Med. 2020;382:906-915.
- Barbieri RL. Effective treatment of recurrent bacterial vaginosis. OBG Manag. 2017;29:7-12.
2021 Update on fertility
In this Update, we discuss several aspects of infertility and emerging technologic advances in treatment. We review an important infertility fact sheet recently issued by the World Health Organization (WHO) that provides a succinct overview of infertility causes, the rights of infertility patients, treatment challenges, and advocacy efforts. In addition, we discuss what the infertility literature reveals about reducing multiple birth rates and the technologic, financial, and social factors involved. Finally, we look at the molecular progress made in germline-editing technology and the myriad complications involved in its potential future translation to clinical phenotyping.
WHO recognizes the burden of infertility and addresses fertility care needs
World Health Organization (WHO). Infertility fact sheet. September 14, 2020. https://www.who.int/news-room/fact-sheets/detail/infertility. Accessed January 24, 2021.
The WHO published its first comprehensive infertility fact sheet in September 2020. This document is important because it validates infertility as a high-burden disease and disability that diminishes quality of life for up to 186 million individuals globally. The infertility fact sheet is a comprehensive yet focused quick read that addresses the causes of infertility, why infertility is important, challenges, and the WHO response.
Factors in infertility
Infertility is caused by different factors in women and men, yet sometimes it is unexplained, and its relative importance can vary from country to country. For women, tubal disorders (for example, postinfectious), uterine problems (fibroids, congenital), endometriosis, ovarian disorders (polycystic ovary syndrome, ovulation disorders), and endocrine imbalances are the most common factors.
For men, causes of infertility include obstruction of the reproductive tract (as after injuries or infection); hormonal disorders in the hypothalamus, pituitary, and/or testicles (for example, low testosterone); testicular failure to produce sperm (such as after cancer treatment); and abnormal sperm function and quality (low count, motility, or morphology).
Environmental and lifestyle factors— including smoking, obesity, alcohol, or toxins—can affect fertility.
Continue to: Recognizing all individuals’ fertility rights...
Recognizing all individuals’ fertility rights
The WHO infertility fact sheet makes strong statements, recognizing that individuals and couples have the right to decide the number, timing, and spacing of their children. Addressing infertility is therefore an important part of realizing the right of individuals and couples to found a family. This includes heterosexual couples, same-sex partners, older persons, individuals not in sexual relationships who might require infertility management and fertility care services, and notably marginalized populations.
Addressing infertility also can help mitigate gender inequality, which has significant negative social impacts on the lives of infertile individuals, especially women. Fertility education is important to reduce the fear of infertility and contraception use in those wanting pregnancy in the future.
In most countries the biggest challenges are availability, access, and quality of interventions to address infertility. This includes the United States, where only 1 in 4 individuals receive the fertility care they need. Lack of prioritization, ineffective public health strategies, inadequate funding, and costs are barriers. Health policies need to recognize that infertility is a disease that often can be prevented, thereby reducing future costs. Comprehensive awareness and education programs, laws and policies that regulate and ensure access and the human rights of all involved, are essential.
Advocacy efforts
To address infertility and fertility care, the WHO is committed to:
- collaborate with partners on epidemiologic and etiologic research
- facilitate policy dialogue globally to frame infertility within a legal and policy framework
- support generation of data on the burden of infertility
- develop guidelines
- produce other documents of standards
- collaborate with all stakeholders to strengthen political commitment and health system capacity, and
- provide country-level technical support to develop or strengthen policies and services.
For your practice, this means that infertility is recognized as a disease that should receive its appropriate share of health care resources. Infertility and fertility care are the right of every individual according to their desires to found a family. Besides providing the best care you can to all your patients, including referring them when necessary, all health care clinicians should advocate on behalf of their patients to payors, policy makers, and the public the need to provide equitable laws, resources, and funding for infertility and fertility care.
Every person has the right to infertility and fertility care as endorsed by the recent WHO infertility fact sheet. To address this high-burden disease, all women’s health care clinicians should be aware of, equitably diagnose and treat, refer as necessary, and advocate for infertile individuals.
Continue to: Lessons learned in reducing multiple pregnancy rates in infertility treatment...
Lessons learned in reducing multiple pregnancy rates in infertility treatment
Views and reviews section. Fertil Steril. 2020;114:671- 672; 673-679; 680-689; 690-714; 715-721.
In the October 2020 issue of Fertility and Sterility, the Views and Reviews section included 5 articles on avoiding multiple live birth rates (LBRs) in assisted reproductive technologies (ART).1-5 International experts provided a comprehensive review of global multiple LBRs and their associated negative impact on maternal and perinatal outcomes, reasons for global variability, strategies to reduce multiples, single embryo transfer, and implications of funding and reporting. These international comparisons and recommendations are helpful and applicable to infertility care in the United States.3
The rise of multiple birth rates
During the first decade of in vitro fertilization (IVF), live birth rates were low, increasing to 14% in 1990. Multiple embryos needed to be transferred so that even these LBRs could be obtained. In the 1990s, however, laboratory technology improved rapidly, with increased implantation rates and subsequent rapid increases in LBR, but also with increased multiple birth rates (MBRs).
In the United States, clinic-specific reporting helped create competition among clinics for the best LBRs, and this led to MBRs of 30% and higher. Numerous studies documented the associated significantly increased morbidity and mortality of both mothers and babies. Similar situations occurred in many other countries while some, especially Nordic nations, Australia, New Zealand, and Japan, had twin rates of less than 10% or even 5% since the early 2000s. So why the difference?
The higher MBR is due largely to the transfer of more than one embryo. The immediate solution is therefore always to perform elective single embryo transfer (eSET). However, numerous factors affect the decision to perform eSET or not, and this ideal is far from being achieved. Older women, those with longer duration of infertility and/or failed treatment, often feel a time pressure and want to transfer more embryos. Of course, biologically this is reasonable because the number and quality of their embryos is lower. While attempts have been made to assess embryo quality with preimplantation genetic testing for aneuploidy, evidence that this increases the LBR is controversial except possibly in women aged 35 to 38 years. This is especially true when the cumulative LBR, that is, the number of live births after transfer of all embryos from an egg retrieval cycle, is the measured outcome.
The major factor that determines the frequency of eSET is financial. Affordability is the out-of-pocket cost (after insurance or other subsidy) as a percentage of disposable income, and it is the most important factor that determines whether eSET is performed. Less affordable treatment creates a financial incentive to transfer more than one embryo to maximize the pregnancy rates in fewer cycles.5 Other factors include whether the effectiveness of treatment, that is, LBR, is emphasized over safety, that is, MBR. In the United States, the Society for Assisted Reproductive Technology now reports cumulative LBR, singleton and multiple LBR, and preterm births as outcomes, thereby increasing the emphasis on eSET.
Sociologic, cultural, and religious factors also can affect the frequency of eSET. Even within the United States, great variation exists in values and beliefs regarding infertility treatment. It can be challenging to determine who makes decisions: the patient alone, the physician, the payor, professional guidelines, or laws. In many countries, including the United States, it is an amalgam of these.
Setting new goals
If the goal is to reduce the MBR, what should that rate be? In the past few years, the MBR in the United States has been reduced to approximately 10%. It is reasonable now to set a goal of 5% in the next several years. To do this, we can learn from countries that have been successful. The United States already has very high-quality clinical and laboratory services, knowledgeable physicians, and a reasonable regulatory environment. Improved technology, specifically embryo selection for transfer, and focus on adherence to established embryo transfer guidelines could help.
Many would argue that eSET essentially should be performed always in women younger than age 40 and in all women of any age with a known euploid embryo. The major problem that drives multiples is the lack of affordability, which can be addressed by increased subsidies from payors. Increased subsidies can result from legislative mandates or societal pressures on employers, either of which could be associated with requirements for eSET and/or reduced MBRs.
In your practice, you can now reassure your infertility patients that cumulative LBRs are excellent in the United States and that the risk of multiple pregnancy has been reduced dramatically. This should encourage more patients to accept and take advantage of this successful technology that has resulted in the birth of millions of babies globally. Further reduction of the MBR to 5% should be possible within a few years through education and advocacy by women’s health care clinicians that results in increased subsidies and more affordable IVF.
The multiple birth rate in ART has been reduced to 10% in the United States through an increased understanding of the complex factors that affect embryo transfer practices globally. Further progress will depend primarily on increased subsidies that make ART more affordable.
Continue to: Genetics and ART...
Genetics and ART: Selection versus correction
Adashi EY, Cohen IG. The case for remedial germline editing—the long-term view. JAMA. 2020;323:1762-1763.
Rosenbaum L. The future of gene editing—toward scientific and social consensus. N Engl J Med. 2019;380:971-975.
Cyranoski D. The CRISPR-baby scandal: what’s next for human gene-editing. Nature. 2019;566:440-442.
de Wert G, Pennings G, Clarke A, et al; European Society of Human Genetics and European Society of Human Reproduction and Embryology. Human germline gene editing: recommendations of ESHG and ESHRE. Hum Reprod Open. 2018;hox025.
Following the completion of the Human Genome Project in 2003 and major technologic advancements in the subsequent years, the field of human genetics became the focal point of convergence for several distinct but interrelated disciplines: bioinformatics, computational biology, and sequencing technologies. As the result, individual human genomes can now be sequenced at a single base pair level, and with higher fidelity, at a fraction of the original cost and at a much faster speed.
This molecular progress, however, has not been accompanied by an equivalent clinical progress, because in a significant number of cases a defined and predictable clinical phenotype cannot be attributed to a detected molecular genotype. This has resulted in an overabundance of variants of uncertain significance. Variable expressivity, incomplete penetrance, epigenetics, mosaicism, and the polygenic nature of many human traits further complicate reliable interpretation and prognostication of the colossal amount of molecular genetic data that are being generated by the above-mentioned technologic advances.
Considering these limitations, at this juncture it is crucial to acknowledge that any attempts to prematurely commercialize these preclinical and research studies (such as polygenic risk scores for embryos) are perilous and have the potential to cause significant harm in terms of unnecessary stress and anxiety for intended parents as well as the potential for yet-unmapped societal and legal implications.
However, it is just a matter of time until more accurate clinical phenotyping catches up with molecular genotyping. As we get closer to this next historic milestone, precision medicine in the postnatal life (with regard to both diagnostics and therapeutics) and preimplantation genetic testing (PGT) at the prenatal stage for a much wider spectrum of conditions—including both monogenic and polygenic traits—may indeed become a reality.
The potential of germline editing
Specifically regarding PGT (which requires IVF), it is important to recognize that due to the limited and nonrenewable endowment of human oocytes (ovarian reserve), combined with the detrimental impact of advancing age on the quality of the remaining cohort as manifested by a higher risk of aneuploidy, the current clinical practice of trying to “select” a nonaffected embryo can be very inefficient. As a result, the intended parents pursuing such treatments may need to undergo multiple cycles of ovarian stimulation and oocyte retrieval.
A potential solution for genes associated with known diseases is the prospect of remedial germline editing by CRISPR–Cas9 technology or its future descendants. This would take advantage of the existing embryos to try to “correct” the defective gene instead of trying to “select” a normal embryo. These technologies are still in the early stages of development and are remotely distant from clinical applications. On the other hand, although germline gene editing, if actualized, would be a monumental breakthrough in the history of genetics and medicine, we must be cognizant of its serious legal, societal, and ethical ramifications, which are currently unknown. Furthermore, even at the biologic and technical level, the technology still is not advanced enough to reliably rule out off-target modifications, and the unintended clinical consequences of the on-target corrections have not been studied either.
Regulation of genetic modifications
Due to these myriad concerns and the lack of an existing appropriate regulatory framework and oversight for such interventions, current US law (since December 2015, through provisions in annual federal appropriations laws passed by Congress and renewed annually thereafter) bars the US Food and Drug Administration from considering any clinical trial application “in which a human embryo is intentionally created or modified to include a heritable genetic modification.” Notably, this moratorium also prohibits mitochondrial replacement technology (MRT), which is a less controversial and relatively better-studied innovation.
Mitochondrial genetic disorders caused by the mutations in mitochondrial DNA (versus nuclear DNA) are amenable to a specific treatment strategy aimed at substituting the defective maternal mitochondrial genome with the mitochondrial genome of an unaffected donor oocyte. This can be achieved via either pronuclear transfer, which involves isolation and transfer of the male and female pronuclei from an affected embryo to an enucleated normal donor embryo, or maternal spindle transfer, which involves isolation and transfer of the metaphase II spindle complex of an affected oocyte to an enucleated disease-free donor egg. It is noteworthy that in 2015 in the United Kingdom, Parliament expanded the definition of “permitted eggs and embryos” to include those “where unhealthy mitochondrial DNA is replaced by healthy mitochondrial DNA from a donor.” This thereby allows the UK Human Fertilisation and Embryology Authority to formally direct and oversee clinical trials in MRT.
Summing up
Although the future of assisted human reproduction cannot be clearly outlined at this time, it is likely to be radically different from the current state given these emerging applications at the intersection of ART and diagnostic and therapeutic genetics. To ensure that exploring this uncharted territory will ultimately be in the interest of humankind and civilization, proper regulatory oversight—after careful consideration of all ethical, societal, and legal implications—needs to be developed for all preclinical and clinical research in this field. Participatory public engagement must be an integrated part of this process. ●
The field of human genetics has already transformed medicine. However, the convergence of the interrelated disciplines of bioinformatics, computational biology, sequencing technologies, and CRISPR–Cas9 technology is creating incredible new advances that will bring great benefits but also major societal challenges.
- Farquhar C. Avoiding multiple pregnancies in assisted reproductive technologies: transferring one embryo at a time should be the norm. Fertil Steril. 2020;114:671-672.
- Bergh C, Kamath MS, Wang R, et al. Strategies to reduce multiple pregnancies during medically assisted reproduction. Fertil Steril. 2020;114:673-679.
- Adamson GD, Norman RJ. Why are multiple pregnancy rates and single embryo transfer rates so different globally, and what do we do about it? Fertil Steril. 2020;114:680-689.
- Eapen A, Ryan GL, Ten Eyck P, et al. Current evidence supporting a goal of singletons: a review of maternal and neonatal outcomes associated with twin versus singleton pregnancies after in vitro fertilization and intracytoplasmic sperm injection. Fertil Steril. 2020;114: 690-714.
- Chambers GM, Keller E, Choi S, et al. Funding and public reporting strategies for reducing multiple pregnancy from fertility treatments. Fertil Steril. 2020;114:715-721.
In this Update, we discuss several aspects of infertility and emerging technologic advances in treatment. We review an important infertility fact sheet recently issued by the World Health Organization (WHO) that provides a succinct overview of infertility causes, the rights of infertility patients, treatment challenges, and advocacy efforts. In addition, we discuss what the infertility literature reveals about reducing multiple birth rates and the technologic, financial, and social factors involved. Finally, we look at the molecular progress made in germline-editing technology and the myriad complications involved in its potential future translation to clinical phenotyping.
WHO recognizes the burden of infertility and addresses fertility care needs
World Health Organization (WHO). Infertility fact sheet. September 14, 2020. https://www.who.int/news-room/fact-sheets/detail/infertility. Accessed January 24, 2021.
The WHO published its first comprehensive infertility fact sheet in September 2020. This document is important because it validates infertility as a high-burden disease and disability that diminishes quality of life for up to 186 million individuals globally. The infertility fact sheet is a comprehensive yet focused quick read that addresses the causes of infertility, why infertility is important, challenges, and the WHO response.
Factors in infertility
Infertility is caused by different factors in women and men, yet sometimes it is unexplained, and its relative importance can vary from country to country. For women, tubal disorders (for example, postinfectious), uterine problems (fibroids, congenital), endometriosis, ovarian disorders (polycystic ovary syndrome, ovulation disorders), and endocrine imbalances are the most common factors.
For men, causes of infertility include obstruction of the reproductive tract (as after injuries or infection); hormonal disorders in the hypothalamus, pituitary, and/or testicles (for example, low testosterone); testicular failure to produce sperm (such as after cancer treatment); and abnormal sperm function and quality (low count, motility, or morphology).
Environmental and lifestyle factors— including smoking, obesity, alcohol, or toxins—can affect fertility.
Continue to: Recognizing all individuals’ fertility rights...
Recognizing all individuals’ fertility rights
The WHO infertility fact sheet makes strong statements, recognizing that individuals and couples have the right to decide the number, timing, and spacing of their children. Addressing infertility is therefore an important part of realizing the right of individuals and couples to found a family. This includes heterosexual couples, same-sex partners, older persons, individuals not in sexual relationships who might require infertility management and fertility care services, and notably marginalized populations.
Addressing infertility also can help mitigate gender inequality, which has significant negative social impacts on the lives of infertile individuals, especially women. Fertility education is important to reduce the fear of infertility and contraception use in those wanting pregnancy in the future.
In most countries the biggest challenges are availability, access, and quality of interventions to address infertility. This includes the United States, where only 1 in 4 individuals receive the fertility care they need. Lack of prioritization, ineffective public health strategies, inadequate funding, and costs are barriers. Health policies need to recognize that infertility is a disease that often can be prevented, thereby reducing future costs. Comprehensive awareness and education programs, laws and policies that regulate and ensure access and the human rights of all involved, are essential.
Advocacy efforts
To address infertility and fertility care, the WHO is committed to:
- collaborate with partners on epidemiologic and etiologic research
- facilitate policy dialogue globally to frame infertility within a legal and policy framework
- support generation of data on the burden of infertility
- develop guidelines
- produce other documents of standards
- collaborate with all stakeholders to strengthen political commitment and health system capacity, and
- provide country-level technical support to develop or strengthen policies and services.
For your practice, this means that infertility is recognized as a disease that should receive its appropriate share of health care resources. Infertility and fertility care are the right of every individual according to their desires to found a family. Besides providing the best care you can to all your patients, including referring them when necessary, all health care clinicians should advocate on behalf of their patients to payors, policy makers, and the public the need to provide equitable laws, resources, and funding for infertility and fertility care.
Every person has the right to infertility and fertility care as endorsed by the recent WHO infertility fact sheet. To address this high-burden disease, all women’s health care clinicians should be aware of, equitably diagnose and treat, refer as necessary, and advocate for infertile individuals.
Continue to: Lessons learned in reducing multiple pregnancy rates in infertility treatment...
Lessons learned in reducing multiple pregnancy rates in infertility treatment
Views and reviews section. Fertil Steril. 2020;114:671- 672; 673-679; 680-689; 690-714; 715-721.
In the October 2020 issue of Fertility and Sterility, the Views and Reviews section included 5 articles on avoiding multiple live birth rates (LBRs) in assisted reproductive technologies (ART).1-5 International experts provided a comprehensive review of global multiple LBRs and their associated negative impact on maternal and perinatal outcomes, reasons for global variability, strategies to reduce multiples, single embryo transfer, and implications of funding and reporting. These international comparisons and recommendations are helpful and applicable to infertility care in the United States.3
The rise of multiple birth rates
During the first decade of in vitro fertilization (IVF), live birth rates were low, increasing to 14% in 1990. Multiple embryos needed to be transferred so that even these LBRs could be obtained. In the 1990s, however, laboratory technology improved rapidly, with increased implantation rates and subsequent rapid increases in LBR, but also with increased multiple birth rates (MBRs).
In the United States, clinic-specific reporting helped create competition among clinics for the best LBRs, and this led to MBRs of 30% and higher. Numerous studies documented the associated significantly increased morbidity and mortality of both mothers and babies. Similar situations occurred in many other countries while some, especially Nordic nations, Australia, New Zealand, and Japan, had twin rates of less than 10% or even 5% since the early 2000s. So why the difference?
The higher MBR is due largely to the transfer of more than one embryo. The immediate solution is therefore always to perform elective single embryo transfer (eSET). However, numerous factors affect the decision to perform eSET or not, and this ideal is far from being achieved. Older women, those with longer duration of infertility and/or failed treatment, often feel a time pressure and want to transfer more embryos. Of course, biologically this is reasonable because the number and quality of their embryos is lower. While attempts have been made to assess embryo quality with preimplantation genetic testing for aneuploidy, evidence that this increases the LBR is controversial except possibly in women aged 35 to 38 years. This is especially true when the cumulative LBR, that is, the number of live births after transfer of all embryos from an egg retrieval cycle, is the measured outcome.
The major factor that determines the frequency of eSET is financial. Affordability is the out-of-pocket cost (after insurance or other subsidy) as a percentage of disposable income, and it is the most important factor that determines whether eSET is performed. Less affordable treatment creates a financial incentive to transfer more than one embryo to maximize the pregnancy rates in fewer cycles.5 Other factors include whether the effectiveness of treatment, that is, LBR, is emphasized over safety, that is, MBR. In the United States, the Society for Assisted Reproductive Technology now reports cumulative LBR, singleton and multiple LBR, and preterm births as outcomes, thereby increasing the emphasis on eSET.
Sociologic, cultural, and religious factors also can affect the frequency of eSET. Even within the United States, great variation exists in values and beliefs regarding infertility treatment. It can be challenging to determine who makes decisions: the patient alone, the physician, the payor, professional guidelines, or laws. In many countries, including the United States, it is an amalgam of these.
Setting new goals
If the goal is to reduce the MBR, what should that rate be? In the past few years, the MBR in the United States has been reduced to approximately 10%. It is reasonable now to set a goal of 5% in the next several years. To do this, we can learn from countries that have been successful. The United States already has very high-quality clinical and laboratory services, knowledgeable physicians, and a reasonable regulatory environment. Improved technology, specifically embryo selection for transfer, and focus on adherence to established embryo transfer guidelines could help.
Many would argue that eSET essentially should be performed always in women younger than age 40 and in all women of any age with a known euploid embryo. The major problem that drives multiples is the lack of affordability, which can be addressed by increased subsidies from payors. Increased subsidies can result from legislative mandates or societal pressures on employers, either of which could be associated with requirements for eSET and/or reduced MBRs.
In your practice, you can now reassure your infertility patients that cumulative LBRs are excellent in the United States and that the risk of multiple pregnancy has been reduced dramatically. This should encourage more patients to accept and take advantage of this successful technology that has resulted in the birth of millions of babies globally. Further reduction of the MBR to 5% should be possible within a few years through education and advocacy by women’s health care clinicians that results in increased subsidies and more affordable IVF.
The multiple birth rate in ART has been reduced to 10% in the United States through an increased understanding of the complex factors that affect embryo transfer practices globally. Further progress will depend primarily on increased subsidies that make ART more affordable.
Continue to: Genetics and ART...
Genetics and ART: Selection versus correction
Adashi EY, Cohen IG. The case for remedial germline editing—the long-term view. JAMA. 2020;323:1762-1763.
Rosenbaum L. The future of gene editing—toward scientific and social consensus. N Engl J Med. 2019;380:971-975.
Cyranoski D. The CRISPR-baby scandal: what’s next for human gene-editing. Nature. 2019;566:440-442.
de Wert G, Pennings G, Clarke A, et al; European Society of Human Genetics and European Society of Human Reproduction and Embryology. Human germline gene editing: recommendations of ESHG and ESHRE. Hum Reprod Open. 2018;hox025.
Following the completion of the Human Genome Project in 2003 and major technologic advancements in the subsequent years, the field of human genetics became the focal point of convergence for several distinct but interrelated disciplines: bioinformatics, computational biology, and sequencing technologies. As the result, individual human genomes can now be sequenced at a single base pair level, and with higher fidelity, at a fraction of the original cost and at a much faster speed.
This molecular progress, however, has not been accompanied by an equivalent clinical progress, because in a significant number of cases a defined and predictable clinical phenotype cannot be attributed to a detected molecular genotype. This has resulted in an overabundance of variants of uncertain significance. Variable expressivity, incomplete penetrance, epigenetics, mosaicism, and the polygenic nature of many human traits further complicate reliable interpretation and prognostication of the colossal amount of molecular genetic data that are being generated by the above-mentioned technologic advances.
Considering these limitations, at this juncture it is crucial to acknowledge that any attempts to prematurely commercialize these preclinical and research studies (such as polygenic risk scores for embryos) are perilous and have the potential to cause significant harm in terms of unnecessary stress and anxiety for intended parents as well as the potential for yet-unmapped societal and legal implications.
However, it is just a matter of time until more accurate clinical phenotyping catches up with molecular genotyping. As we get closer to this next historic milestone, precision medicine in the postnatal life (with regard to both diagnostics and therapeutics) and preimplantation genetic testing (PGT) at the prenatal stage for a much wider spectrum of conditions—including both monogenic and polygenic traits—may indeed become a reality.
The potential of germline editing
Specifically regarding PGT (which requires IVF), it is important to recognize that due to the limited and nonrenewable endowment of human oocytes (ovarian reserve), combined with the detrimental impact of advancing age on the quality of the remaining cohort as manifested by a higher risk of aneuploidy, the current clinical practice of trying to “select” a nonaffected embryo can be very inefficient. As a result, the intended parents pursuing such treatments may need to undergo multiple cycles of ovarian stimulation and oocyte retrieval.
A potential solution for genes associated with known diseases is the prospect of remedial germline editing by CRISPR–Cas9 technology or its future descendants. This would take advantage of the existing embryos to try to “correct” the defective gene instead of trying to “select” a normal embryo. These technologies are still in the early stages of development and are remotely distant from clinical applications. On the other hand, although germline gene editing, if actualized, would be a monumental breakthrough in the history of genetics and medicine, we must be cognizant of its serious legal, societal, and ethical ramifications, which are currently unknown. Furthermore, even at the biologic and technical level, the technology still is not advanced enough to reliably rule out off-target modifications, and the unintended clinical consequences of the on-target corrections have not been studied either.
Regulation of genetic modifications
Due to these myriad concerns and the lack of an existing appropriate regulatory framework and oversight for such interventions, current US law (since December 2015, through provisions in annual federal appropriations laws passed by Congress and renewed annually thereafter) bars the US Food and Drug Administration from considering any clinical trial application “in which a human embryo is intentionally created or modified to include a heritable genetic modification.” Notably, this moratorium also prohibits mitochondrial replacement technology (MRT), which is a less controversial and relatively better-studied innovation.
Mitochondrial genetic disorders caused by the mutations in mitochondrial DNA (versus nuclear DNA) are amenable to a specific treatment strategy aimed at substituting the defective maternal mitochondrial genome with the mitochondrial genome of an unaffected donor oocyte. This can be achieved via either pronuclear transfer, which involves isolation and transfer of the male and female pronuclei from an affected embryo to an enucleated normal donor embryo, or maternal spindle transfer, which involves isolation and transfer of the metaphase II spindle complex of an affected oocyte to an enucleated disease-free donor egg. It is noteworthy that in 2015 in the United Kingdom, Parliament expanded the definition of “permitted eggs and embryos” to include those “where unhealthy mitochondrial DNA is replaced by healthy mitochondrial DNA from a donor.” This thereby allows the UK Human Fertilisation and Embryology Authority to formally direct and oversee clinical trials in MRT.
Summing up
Although the future of assisted human reproduction cannot be clearly outlined at this time, it is likely to be radically different from the current state given these emerging applications at the intersection of ART and diagnostic and therapeutic genetics. To ensure that exploring this uncharted territory will ultimately be in the interest of humankind and civilization, proper regulatory oversight—after careful consideration of all ethical, societal, and legal implications—needs to be developed for all preclinical and clinical research in this field. Participatory public engagement must be an integrated part of this process. ●
The field of human genetics has already transformed medicine. However, the convergence of the interrelated disciplines of bioinformatics, computational biology, sequencing technologies, and CRISPR–Cas9 technology is creating incredible new advances that will bring great benefits but also major societal challenges.
In this Update, we discuss several aspects of infertility and emerging technologic advances in treatment. We review an important infertility fact sheet recently issued by the World Health Organization (WHO) that provides a succinct overview of infertility causes, the rights of infertility patients, treatment challenges, and advocacy efforts. In addition, we discuss what the infertility literature reveals about reducing multiple birth rates and the technologic, financial, and social factors involved. Finally, we look at the molecular progress made in germline-editing technology and the myriad complications involved in its potential future translation to clinical phenotyping.
WHO recognizes the burden of infertility and addresses fertility care needs
World Health Organization (WHO). Infertility fact sheet. September 14, 2020. https://www.who.int/news-room/fact-sheets/detail/infertility. Accessed January 24, 2021.
The WHO published its first comprehensive infertility fact sheet in September 2020. This document is important because it validates infertility as a high-burden disease and disability that diminishes quality of life for up to 186 million individuals globally. The infertility fact sheet is a comprehensive yet focused quick read that addresses the causes of infertility, why infertility is important, challenges, and the WHO response.
Factors in infertility
Infertility is caused by different factors in women and men, yet sometimes it is unexplained, and its relative importance can vary from country to country. For women, tubal disorders (for example, postinfectious), uterine problems (fibroids, congenital), endometriosis, ovarian disorders (polycystic ovary syndrome, ovulation disorders), and endocrine imbalances are the most common factors.
For men, causes of infertility include obstruction of the reproductive tract (as after injuries or infection); hormonal disorders in the hypothalamus, pituitary, and/or testicles (for example, low testosterone); testicular failure to produce sperm (such as after cancer treatment); and abnormal sperm function and quality (low count, motility, or morphology).
Environmental and lifestyle factors— including smoking, obesity, alcohol, or toxins—can affect fertility.
Continue to: Recognizing all individuals’ fertility rights...
Recognizing all individuals’ fertility rights
The WHO infertility fact sheet makes strong statements, recognizing that individuals and couples have the right to decide the number, timing, and spacing of their children. Addressing infertility is therefore an important part of realizing the right of individuals and couples to found a family. This includes heterosexual couples, same-sex partners, older persons, individuals not in sexual relationships who might require infertility management and fertility care services, and notably marginalized populations.
Addressing infertility also can help mitigate gender inequality, which has significant negative social impacts on the lives of infertile individuals, especially women. Fertility education is important to reduce the fear of infertility and contraception use in those wanting pregnancy in the future.
In most countries the biggest challenges are availability, access, and quality of interventions to address infertility. This includes the United States, where only 1 in 4 individuals receive the fertility care they need. Lack of prioritization, ineffective public health strategies, inadequate funding, and costs are barriers. Health policies need to recognize that infertility is a disease that often can be prevented, thereby reducing future costs. Comprehensive awareness and education programs, laws and policies that regulate and ensure access and the human rights of all involved, are essential.
Advocacy efforts
To address infertility and fertility care, the WHO is committed to:
- collaborate with partners on epidemiologic and etiologic research
- facilitate policy dialogue globally to frame infertility within a legal and policy framework
- support generation of data on the burden of infertility
- develop guidelines
- produce other documents of standards
- collaborate with all stakeholders to strengthen political commitment and health system capacity, and
- provide country-level technical support to develop or strengthen policies and services.
For your practice, this means that infertility is recognized as a disease that should receive its appropriate share of health care resources. Infertility and fertility care are the right of every individual according to their desires to found a family. Besides providing the best care you can to all your patients, including referring them when necessary, all health care clinicians should advocate on behalf of their patients to payors, policy makers, and the public the need to provide equitable laws, resources, and funding for infertility and fertility care.
Every person has the right to infertility and fertility care as endorsed by the recent WHO infertility fact sheet. To address this high-burden disease, all women’s health care clinicians should be aware of, equitably diagnose and treat, refer as necessary, and advocate for infertile individuals.
Continue to: Lessons learned in reducing multiple pregnancy rates in infertility treatment...
Lessons learned in reducing multiple pregnancy rates in infertility treatment
Views and reviews section. Fertil Steril. 2020;114:671- 672; 673-679; 680-689; 690-714; 715-721.
In the October 2020 issue of Fertility and Sterility, the Views and Reviews section included 5 articles on avoiding multiple live birth rates (LBRs) in assisted reproductive technologies (ART).1-5 International experts provided a comprehensive review of global multiple LBRs and their associated negative impact on maternal and perinatal outcomes, reasons for global variability, strategies to reduce multiples, single embryo transfer, and implications of funding and reporting. These international comparisons and recommendations are helpful and applicable to infertility care in the United States.3
The rise of multiple birth rates
During the first decade of in vitro fertilization (IVF), live birth rates were low, increasing to 14% in 1990. Multiple embryos needed to be transferred so that even these LBRs could be obtained. In the 1990s, however, laboratory technology improved rapidly, with increased implantation rates and subsequent rapid increases in LBR, but also with increased multiple birth rates (MBRs).
In the United States, clinic-specific reporting helped create competition among clinics for the best LBRs, and this led to MBRs of 30% and higher. Numerous studies documented the associated significantly increased morbidity and mortality of both mothers and babies. Similar situations occurred in many other countries while some, especially Nordic nations, Australia, New Zealand, and Japan, had twin rates of less than 10% or even 5% since the early 2000s. So why the difference?
The higher MBR is due largely to the transfer of more than one embryo. The immediate solution is therefore always to perform elective single embryo transfer (eSET). However, numerous factors affect the decision to perform eSET or not, and this ideal is far from being achieved. Older women, those with longer duration of infertility and/or failed treatment, often feel a time pressure and want to transfer more embryos. Of course, biologically this is reasonable because the number and quality of their embryos is lower. While attempts have been made to assess embryo quality with preimplantation genetic testing for aneuploidy, evidence that this increases the LBR is controversial except possibly in women aged 35 to 38 years. This is especially true when the cumulative LBR, that is, the number of live births after transfer of all embryos from an egg retrieval cycle, is the measured outcome.
The major factor that determines the frequency of eSET is financial. Affordability is the out-of-pocket cost (after insurance or other subsidy) as a percentage of disposable income, and it is the most important factor that determines whether eSET is performed. Less affordable treatment creates a financial incentive to transfer more than one embryo to maximize the pregnancy rates in fewer cycles.5 Other factors include whether the effectiveness of treatment, that is, LBR, is emphasized over safety, that is, MBR. In the United States, the Society for Assisted Reproductive Technology now reports cumulative LBR, singleton and multiple LBR, and preterm births as outcomes, thereby increasing the emphasis on eSET.
Sociologic, cultural, and religious factors also can affect the frequency of eSET. Even within the United States, great variation exists in values and beliefs regarding infertility treatment. It can be challenging to determine who makes decisions: the patient alone, the physician, the payor, professional guidelines, or laws. In many countries, including the United States, it is an amalgam of these.
Setting new goals
If the goal is to reduce the MBR, what should that rate be? In the past few years, the MBR in the United States has been reduced to approximately 10%. It is reasonable now to set a goal of 5% in the next several years. To do this, we can learn from countries that have been successful. The United States already has very high-quality clinical and laboratory services, knowledgeable physicians, and a reasonable regulatory environment. Improved technology, specifically embryo selection for transfer, and focus on adherence to established embryo transfer guidelines could help.
Many would argue that eSET essentially should be performed always in women younger than age 40 and in all women of any age with a known euploid embryo. The major problem that drives multiples is the lack of affordability, which can be addressed by increased subsidies from payors. Increased subsidies can result from legislative mandates or societal pressures on employers, either of which could be associated with requirements for eSET and/or reduced MBRs.
In your practice, you can now reassure your infertility patients that cumulative LBRs are excellent in the United States and that the risk of multiple pregnancy has been reduced dramatically. This should encourage more patients to accept and take advantage of this successful technology that has resulted in the birth of millions of babies globally. Further reduction of the MBR to 5% should be possible within a few years through education and advocacy by women’s health care clinicians that results in increased subsidies and more affordable IVF.
The multiple birth rate in ART has been reduced to 10% in the United States through an increased understanding of the complex factors that affect embryo transfer practices globally. Further progress will depend primarily on increased subsidies that make ART more affordable.
Continue to: Genetics and ART...
Genetics and ART: Selection versus correction
Adashi EY, Cohen IG. The case for remedial germline editing—the long-term view. JAMA. 2020;323:1762-1763.
Rosenbaum L. The future of gene editing—toward scientific and social consensus. N Engl J Med. 2019;380:971-975.
Cyranoski D. The CRISPR-baby scandal: what’s next for human gene-editing. Nature. 2019;566:440-442.
de Wert G, Pennings G, Clarke A, et al; European Society of Human Genetics and European Society of Human Reproduction and Embryology. Human germline gene editing: recommendations of ESHG and ESHRE. Hum Reprod Open. 2018;hox025.
Following the completion of the Human Genome Project in 2003 and major technologic advancements in the subsequent years, the field of human genetics became the focal point of convergence for several distinct but interrelated disciplines: bioinformatics, computational biology, and sequencing technologies. As the result, individual human genomes can now be sequenced at a single base pair level, and with higher fidelity, at a fraction of the original cost and at a much faster speed.
This molecular progress, however, has not been accompanied by an equivalent clinical progress, because in a significant number of cases a defined and predictable clinical phenotype cannot be attributed to a detected molecular genotype. This has resulted in an overabundance of variants of uncertain significance. Variable expressivity, incomplete penetrance, epigenetics, mosaicism, and the polygenic nature of many human traits further complicate reliable interpretation and prognostication of the colossal amount of molecular genetic data that are being generated by the above-mentioned technologic advances.
Considering these limitations, at this juncture it is crucial to acknowledge that any attempts to prematurely commercialize these preclinical and research studies (such as polygenic risk scores for embryos) are perilous and have the potential to cause significant harm in terms of unnecessary stress and anxiety for intended parents as well as the potential for yet-unmapped societal and legal implications.
However, it is just a matter of time until more accurate clinical phenotyping catches up with molecular genotyping. As we get closer to this next historic milestone, precision medicine in the postnatal life (with regard to both diagnostics and therapeutics) and preimplantation genetic testing (PGT) at the prenatal stage for a much wider spectrum of conditions—including both monogenic and polygenic traits—may indeed become a reality.
The potential of germline editing
Specifically regarding PGT (which requires IVF), it is important to recognize that due to the limited and nonrenewable endowment of human oocytes (ovarian reserve), combined with the detrimental impact of advancing age on the quality of the remaining cohort as manifested by a higher risk of aneuploidy, the current clinical practice of trying to “select” a nonaffected embryo can be very inefficient. As a result, the intended parents pursuing such treatments may need to undergo multiple cycles of ovarian stimulation and oocyte retrieval.
A potential solution for genes associated with known diseases is the prospect of remedial germline editing by CRISPR–Cas9 technology or its future descendants. This would take advantage of the existing embryos to try to “correct” the defective gene instead of trying to “select” a normal embryo. These technologies are still in the early stages of development and are remotely distant from clinical applications. On the other hand, although germline gene editing, if actualized, would be a monumental breakthrough in the history of genetics and medicine, we must be cognizant of its serious legal, societal, and ethical ramifications, which are currently unknown. Furthermore, even at the biologic and technical level, the technology still is not advanced enough to reliably rule out off-target modifications, and the unintended clinical consequences of the on-target corrections have not been studied either.
Regulation of genetic modifications
Due to these myriad concerns and the lack of an existing appropriate regulatory framework and oversight for such interventions, current US law (since December 2015, through provisions in annual federal appropriations laws passed by Congress and renewed annually thereafter) bars the US Food and Drug Administration from considering any clinical trial application “in which a human embryo is intentionally created or modified to include a heritable genetic modification.” Notably, this moratorium also prohibits mitochondrial replacement technology (MRT), which is a less controversial and relatively better-studied innovation.
Mitochondrial genetic disorders caused by the mutations in mitochondrial DNA (versus nuclear DNA) are amenable to a specific treatment strategy aimed at substituting the defective maternal mitochondrial genome with the mitochondrial genome of an unaffected donor oocyte. This can be achieved via either pronuclear transfer, which involves isolation and transfer of the male and female pronuclei from an affected embryo to an enucleated normal donor embryo, or maternal spindle transfer, which involves isolation and transfer of the metaphase II spindle complex of an affected oocyte to an enucleated disease-free donor egg. It is noteworthy that in 2015 in the United Kingdom, Parliament expanded the definition of “permitted eggs and embryos” to include those “where unhealthy mitochondrial DNA is replaced by healthy mitochondrial DNA from a donor.” This thereby allows the UK Human Fertilisation and Embryology Authority to formally direct and oversee clinical trials in MRT.
Summing up
Although the future of assisted human reproduction cannot be clearly outlined at this time, it is likely to be radically different from the current state given these emerging applications at the intersection of ART and diagnostic and therapeutic genetics. To ensure that exploring this uncharted territory will ultimately be in the interest of humankind and civilization, proper regulatory oversight—after careful consideration of all ethical, societal, and legal implications—needs to be developed for all preclinical and clinical research in this field. Participatory public engagement must be an integrated part of this process. ●
The field of human genetics has already transformed medicine. However, the convergence of the interrelated disciplines of bioinformatics, computational biology, sequencing technologies, and CRISPR–Cas9 technology is creating incredible new advances that will bring great benefits but also major societal challenges.
- Farquhar C. Avoiding multiple pregnancies in assisted reproductive technologies: transferring one embryo at a time should be the norm. Fertil Steril. 2020;114:671-672.
- Bergh C, Kamath MS, Wang R, et al. Strategies to reduce multiple pregnancies during medically assisted reproduction. Fertil Steril. 2020;114:673-679.
- Adamson GD, Norman RJ. Why are multiple pregnancy rates and single embryo transfer rates so different globally, and what do we do about it? Fertil Steril. 2020;114:680-689.
- Eapen A, Ryan GL, Ten Eyck P, et al. Current evidence supporting a goal of singletons: a review of maternal and neonatal outcomes associated with twin versus singleton pregnancies after in vitro fertilization and intracytoplasmic sperm injection. Fertil Steril. 2020;114: 690-714.
- Chambers GM, Keller E, Choi S, et al. Funding and public reporting strategies for reducing multiple pregnancy from fertility treatments. Fertil Steril. 2020;114:715-721.
- Farquhar C. Avoiding multiple pregnancies in assisted reproductive technologies: transferring one embryo at a time should be the norm. Fertil Steril. 2020;114:671-672.
- Bergh C, Kamath MS, Wang R, et al. Strategies to reduce multiple pregnancies during medically assisted reproduction. Fertil Steril. 2020;114:673-679.
- Adamson GD, Norman RJ. Why are multiple pregnancy rates and single embryo transfer rates so different globally, and what do we do about it? Fertil Steril. 2020;114:680-689.
- Eapen A, Ryan GL, Ten Eyck P, et al. Current evidence supporting a goal of singletons: a review of maternal and neonatal outcomes associated with twin versus singleton pregnancies after in vitro fertilization and intracytoplasmic sperm injection. Fertil Steril. 2020;114: 690-714.
- Chambers GM, Keller E, Choi S, et al. Funding and public reporting strategies for reducing multiple pregnancy from fertility treatments. Fertil Steril. 2020;114:715-721.